pharmacy patient case study examples

Login into your account

Please enter username and password bellow!

Forgotten Password

Don't have an account? Register here

Our Pharmacy Blog

Mastering pharmacy case studies.

pharmacy case studies

Introduction

If you are training to become a pharmacist, you will have had experience with pharmacy case studies. But why are pharmacy case studies so important?

As a qualifying pharmacist, case studies bring together the threads of study over the past four years. This includes your study of subjects such as:

  • Pharmacology
  • Pharmaceutical chemistry
  • Pharmaceutics
  • Clinical pharmacy practice

In practice, pharmacists are expected to draw on this knowledge and clinically apply it where necessary. These subjects feed into one another where knowledge of one subject became necessary to advance in a second subject and so forth. University staff overseeing the course structure put that structure together with these factors in mind. Pharmacy case studies are an important component, often toward the end of your pharmacy degree, that aim to establish the most relevant details that play a role in the career of a qualified pharmacist.

Case studies give pharmacy students an opportunity to test their understanding of a specialist topic. This may be anything from the formulation and dosing of medicines; to a drug’s mechanism of action, drug interactions, and clinical appropriateness for a medicine in a given scenario for a patient with specific factors to keep in mind. Evidently, this takes practice. There are many possible case study scenarios to consider. It can be difficult to always get things right.

Case studies are, then, a special kind of barometer through which we measure the professional competency of pharmacy students .

That is why pharmacy case studies are popular in degree programs – forcing students to think critically about a given topic – whether it be blood diagnostics, epidemiology, treatment options, or drug monitoring – tying together their past year’s study and how to apply this knowledge to (potentially) real-life situations.

Below, we’ve put together an introductory case study to provide you with a clear example of what kinds of questions can be asked and how best you should approach each question. With enough practice, clinical case studies become that much easier. And with time, students learn to enjoy case studies – as they are often your first direct experience of learning real and relevant facts that have an impact on your long-term professional career.

Pharmacy Case Study – Osteoporosis

A 49-year old woman with osteoporosis has been taking Fosamax for 6-months. She visits her GP complaining of acid reflux and pain radiating down her esophagus.

  • What is the active ingredient of Fosamax?
  • What is the mechanism of action of this medicine?
  • Suggest a reason why this patient is taking Fosamax.
  • How should the GP respond to the patient’s symptoms?
  • What foods and/or medicines should the patient avoid?

Explanation

The questions ask more about the medicine – how it works, what it’s indicated for, how the GP should respond to patient symptoms and what interactions, from both food and drug sources, the prescriber and pharmacist must consider.

A – The active ingredient of Fosamax is alendronate; a bisphosphonate drug.

B – Alendronate works by inhibiting osteoclast-mediated bone resorption (the process whereby bone is broken down and minerals are released into the blood).

C – As a 49-year old woman, the patient is likely post-menopausal. Bisphosphonates are routinely prescribed to prevent osteoporosis in these patients.

D – The patient may be improperly administering the medicine. Patients who do not follow the correct protocol of administering bisphosphonates are likely to experience specific symptoms, particularly relating to the esophagus and GI tract. Patients should be counseled to take the medicine in the morning on an empty stomach, whilst remaining upright, and taken with a full glass of water. This eases the bisphosphonate through the digestive tract without irritating the esophageal wall. Patients should avoid taking and food or medicines, both before and for at least 30-minutes after taking the bisphosphonate.

E – Two groups of medicines should be avoided. First, NSAIDs should be avoided; as they increase the risk of gastrointestinal side effects. Second, patients should avoid foods or supplements that contain multivalent ions such as magnesium, aluminum, or calcium. This category includes dairy products and antacids. As we learned above, bisphosphonates should be avoided with these medicines/foods for at least 30-minutes after the bisphosphonate has been taken (on an empty stomach).

Practice More Pharmacy Case Studies

The more pharmacy case studies you practice , the better prepared you are for the needs and demands that present during the licensing end of your pharmacy program. Pharmacy case studies help guide students through the must-know clinical facts about drugs and medicines; both theoretical and practical knowledge.

Clinical case studies are one of the ways in which students make the transition between an experienced, knowledgeable student and a clinical professional whose expertise can be trusted in the real world. Case studies bring pharmacy students to the next level. The more practice you put in, the better results you can expect as you progress through the licensing stage of your nascent career. That, in the end, is what matters.

That’s about it for our discussion of case studies! Check back to our pharmacy blog soon for more exclusive content to help you master the science of drugs and medicines and build your long-term career.

  • Anticancer Pharmacology
  • Antimicrobial Drugs
  • Cardiovascular Pharmacology
  • Clinical Case Study
  • Clinical Pharmacy
  • General Pharmacology
  • GI Pharmacology
  • Immune System Pharmacology
  • Nervous System Pharmacology
  • Respiratory Pharmacology
  • Study Tips and Tricks

Join Our Mailing List For Even More Facts!

Don't stop learning now, you may also like, loop diuretics pharmacology, triptans pharmacology, cyp 3a4 enzyme | what is it.

PharmD

Clinical Case Studies with Answers

This section includes clinical case studies* with answers for the theoretical practice of Doctor of Pharmacy ( Pharm.D ) students. Also, it is equally useful for Pharm.D PB students. These Pharm.D case studies are regularly discussed in the telegram group .   You can join there and participate in active case discussions. Case studies help you revise your Pharmacotherapy syllabus and increase knowledge of disease in an innovative way apart from practical knowledge. 

You can access these clinical case studies with answers with no charge, which are contributed by various seniors from Doctor of Pharmacy stream only. In some cases practical references are also taken from certified & specialized practitioner from the respective field along with standard treatment guidelines.

*Clinical Case Studies included here are collected from various sources and are only for study purpose while maintaining the privacy of patients.

1. Pharm.D Case Studies for second year and Pharm.D PB first year 

A) cardiovascular system case studies with answers: .

  • Hypertension    :   Case Study 1      and Case Study 2  
  • Congestive Heart Failure   :  Case Study 3 and Case Study 4
  • Angina Pectoris  :   Case study 5 and Case Study 6
  • Myocardial Infarction :  Case Study 7 and Case Study 8
  • Hyperlipidaemias :   Case Study 9 and Case Study 10  
  • Electrophysiology of heart and Arrhythmia :   Case Study 11 and Case Study 12

b) Respiratory System Case Studies with answers:

  • Asthma:   Case Study 13 and Case Study 14
  • COPD : Case Study 15 and Case Study 16

c) Endocrine System Case Studies with answers:

  • Diabetes:   Case Study 17    and  Case Study – 18 
  • Hypothyroidism :  Case Study -19 and Case Study -20
  • Other Thyroid Disorders :  Case Study -21    and Case Study -22
  • Oral Contraceptive Use : Case Study- 23 and Case Study- 24
  • Hormone Replacement Therapy:   Case Study- 25    and  Case Study- 26
  • Osteoporosis: Case Study – 27 and Case Study – 28

d) Opthalmology Case Studies with answers: 

  • Glaucoma: Case Study – 29    and Case Study – 30 
  • Conjunctivitis: Case Study – 3 1 and Case Study – 32

e) General prescribing guidelines, Small Case Study with answers:

  • Pediatrics : Case Study – 33

Case Studies for Doctor of Pharmacy students :

Case study-1 (hypertension with cardiovascular comorbidities).

JR is a 58-year-old man with a medical history of elevated low-density lipoprotein levels and well-controlled chronic stable angina (experiencing <1 angina attack per month) secondary to coronary artery disease (CAD). He presents to his primary care physician for a follow-up appointment after his blood pressure (BP) was found to be 165/94 mm Hg at his annual physical exam. At today’s visit, JR’s BP is found to be 166/93 mm Hg, resulting in a diagnosis of hypertension. JR is currently on  Atorvastatin 40 mg daily  and  Metoprolol Tartrate 100 mg twice daily.  And he reports no adverse effects from either medication. He has no other medical history of note, and his resting heart rate is 65 to 70 beats per minute.

Questions :

  • What is the target goal for BP in this patient?
  • What are the main classes of anti-hypertensives that can be used in this case ?
  • Prepare a therapeutic regimen for this patient.

For Case Study-1 Answer, Click here.

Case Study-2 (Hypertension with Type-2 Diabetes Mellitus)

Mr. MK a 55-year-old man, having history of  hypertension and type-2 diabetes mellitus  for past 10 years with non-compliance to medication and poor diet control referred to the clinic for further management of poorly controlled diabetes and hypertension.

Socio-demographics:

Age :55                                   Sex : Male BMI : 35kg/m²                     Weight : 98 kg Occupation : Salesman

Family History :

Mother : Diabetic ( on dialysis). Father : Stroke ( residual left hemiparesis).

Subjective and Objective Evidence :

  • Minimal bilateral leg edema.
  • Bilateral proliferative retinopathy. Vitals –         Blood Pressure : 160/90 mm/Hg                      Pulse rate. : 88 /min

Investigation results :

A1c                                       :      9.2% FBS.                                     :      11.8 mmol/L Serum creatinine               :      1.2 mg/dl eGFR                                    :      88 ml/min/1.73m² 24 hr urinary protein         :      200mg/24 hr. ECG                                      :       Left Ventricular Hypertrophy.

Past medication :

Metformin          1000 mg   BD Gliclazide           160 mg      BD Amlodipine        10 mg        OD

  • What might be reasons for his poorly controlled diabetes and hypertension?
  • What would be the A1c and BP target?
  • How would you manage both HTN and T2DM ?
  • Is there any Drug interaction, that should be taken in consider?
  • What are the major patient counselling points?

For Case Study-2 Answer,  Click here

For more Pharm.D Case studies. Click here.

Case Study-3 ( Congestive Cardiac Failure)

Mrs. JE a 70-year-old woman, was admitted to the medical unit with complaints of increasing dyspnea on exertion.

Subjective data:

  • Had a severe MI at 58 years of age
  • Has experienced increasing dyspnea on exertion during the last 2 years
  • Recently had a respiratory tract infection, frequently cough and edema in legs 2 weeks ago
  • Shortness of breathe while having average walking
  • Has to sleep with head elevated on 3 pillows
  • Does not always remember to take medication

Objective Evidence :

  • In respiratory distress, use of accessory muscles.
  • Heart murmur.
  • Moist cracle in both lungs.
  • Cyanotic lips and extremeties.
  • Skin cool and diaphoretic. Vitals –  Blood Pressure : 130/80 mm/Hg                Pulse rate. : 70 /min                Respiratory rate : 36 / min

Chest X-Ray : Cardiomegaly with right and left ventricular hypertrophy, fluid in lower lung fields.

Current Medication/ Collaborative Care

Digoxin                               0.25 mg            PO            qd Furosemide                        40 mg               IV              bid Potassium                         40 mEq             PO            bid Enalapril                             5 mg                 PO            qd Sodium diet                       2 gm Oxygen                               6 L/min

  • What is the significance of the findings of the chest X- Ray?
  • Are there any collaborative problems that has been not considered?
  • Is Mrs. JE provided with appropriate drug regimen? Justify.

For  Case Study-3 Answer, Click here .

For more Pharm.D Case studies. Click here.

Case Study-4 ( Case of Congestive Heart Failure in Pediatrics patient)

A 6 weeks old female presented to the emergency room with the chief complaints of lethargy, poor feeding, and respiratory distress. Her parents reported that she sweats a lot on her forehead when feeding. Her parents have also noted her to be increasingly lethargic, with tachypnea, and retractions.

  • She is the product of a G3P2, full term, uncomplicated pregnancy. Delivery was unremarkable except for meconium stained fluid. Her pediatric follow-up has been poor.
  • Developed a febrile illness with cough, rhinorrhea, and emesis prior 2 weeks
  • Subsequently developed progressive respiratory distress.

Objective Evidence:

  • Acyanotic, Lethargic, tachypneic, mildly cachectic
  • Mild to moderate subcoastal & intercoastal Retractions

                                    Heart rate                : 160 / min  

                                    Respiratory rate     : 72 / min   

                                    Temp.                       : 98.24 ᵒF

HEENT exam : unremarkable

  • Neck is supple without lymphadenopathy.
  • Skin is clear with no rashes or other significant skin lesions.
  • Lungs have scattered crackles with slightly decreased aeration in the left lower lobe.
  • Precordium is mildly active.
  • Heart is of regular rate and rhythm, with a Grade II/VI holosystolic murmur at the mid lower left sternal border with radiation to the cardiac apex.
  • S1 is normal and the S2 is prominent. An S4 gallop is noted at the cardiac apex. There are no rubs or valve clicks.
  • Her abdomen is soft, non-distended, and non-tender.
  • The liver edge is palpable 3 to 4 cm below the right costal margin. Bowel sounds are hypoactive.
  • Capillary refill is 4 to 5 seconds (delayed).
  • Chest x-ray:  Moderate cardiomegaly with a moderate degree of pulmonary edema.  No pleural effusions.
  • 12 lead ECG:    Sinus tachycardia, normal PR and QTc intervals, and a left axis deviation. Voltage evidence of biventricular hypertrophy is present. No significant Q-waves or ST segment changes are noted.

A large peri-membranous ventricular septal defect with non-restrictive left to right shunting. All cardiac chambers are dilated. Left ventricular contractility is at the lower range of normal. There is no pericardial effusion.

  • Suggest the best therapeutic regimen to control/overcome the current situation.
  • Should she be referred for surgical correction of Ventricular Septal Defect after the drug regimen has been started?

Go to  Case Study-4 Answers  |  Other  Case Studies

Case Study-5 ( Case of Angina Pectoris)

A 62-year-old male smoker with Type-2 Diabetes Mellitus and Hypertension presents with a 4-month history of exertional chest pain. 

Physical examination shows a  blood pressure  of 152/90 mm Hg, but is otherwise unremarkable. 

The  ECG  is normal, and laboratory tests show a  fasting blood glucose  value of 110 mg/dL,  glycosylated hemoglobin  6.0%,  creatinine  1.1 mg/dL,  total cholesterol  160,  LDL  120,  HDL  38, and  triglycerides  147 mg/dL. 

He exercises for 8 minutes, experiences chest pain, and is found to have a 2-mm  ST-segment depression  in the inferolateral leads at the end of exercise. 

The patient is diagnosed with  chronic stable angina .

  • What is the treatment goal and strategy for this case?
  • Suggest the best follow-up for this case.
  • What are the conditions which worsens the symptoms of angina (in general)?

Case Study-6 (Case of Angina Pectoris)

A Mr SW is a 48-year-old man going through a stressful time at work, who for the past 6 months, has been increasingly short of breath while walking to the bus. He has put this down to his ‘unhealthy ‘ lifestyle. Although he has cut down from two packets to one packet of cigarettes a week, reduced his alcohol intake from about 40 units to 25 units a week and is trying to lose weight (currently 1.8 m tall, weighing 100 kg). He sometimes finds himself short of breath with mild chest tightness, especially when he is running late. He has a strong family history of cardiovascular disease with his father having a stroke in his early 50s and his older brother having had a CABG (coronary artery bypass graft) 2 years ago. Both have encouraged Mr. SW to see his general physician due to his worsening symptoms.

Mr. SW was seen by a physician 2 years ago who prescribed  aspirin  75 mg daily,  atenolol  50 mg daily and a  GTN spray  to be used if he experiences chest pain. Since this time, he has stopped taking the aspirin, because he feels that he does not need it and he has stopped the atenolol for more than a year because he was feeling tired and read that it can cause impotence. He has used the GTN once but, after experiencing a headache and facial flushing, he has not used it since and he does not carry it with him.

After his current physician visit, he is prescribed with:

Aspirin                                              75 mg                                    daily

Simvastatin                                     40 mg                                    at night

Amlodipine                                      5 mg                                      daily

GTN spray                                                                                      when required

  • What information and counselling points would you include?
  • How is stable angina managed?
  • What options are there, if Mr. SW experiences further symptoms despite the use of amlodipine?

 Go For  Case Study-6 Answer  | Go back to  Case Studies | Go to  Home

Case Study-7 (Myocardial Infarction)

A 50 year-old male ( Height -154 cm , weight – 70 kg ) who was auto driver visited to the clinic with chief complaints of chest pain. The patient was apparently alright till 3 h back, when he suddenly felt a vague chest pain present at the center of the chest. Pain was located in the substernal location and was radiating to the right side of the shoulder. The quality of the pain was dull aching, which was increasing in severity rapidly over few hours. Chest pain was aggravated by exertion. Pain was not relieved even during rest. There was profuse sweating associated with the chest pain, and also a sense of doom or impending death. There were mild dyspnea and palpitation associated with the chest pain. There was no history of pedal edema, abdominal distension and facial puffiness, loss of appetite/fullness, right hypochondriac pain, or increased neck pulsations. Chest pain  was  diffuse in nature and  not  localized. There is no relation of the chest pain to food intake. There were no associated vomiting and hematemesis. There was no history of trauma and no history of any psychological disorders in the past. Review of other systems was normal.

Past Medical History:

The patient is not a known diabetic, or hypertensive. He has had no similar history in the past.

Past Medication History:

The patient is not on any medications. There was no history of any intake of any cardiotoxic drugs (cancer chemotherapy or prolonged steroids).

Personal History:

The patient takes mixed diet and smokes 1 packet cigarette/day for last 15 years. There was no alleged history of any alcohol or illicit drug abuse.

Pulse rate                             : 120/min      with    regular rhythm

Blood pressure                    : Left hand – 138/94 mm Hg

                                                 Left leg   -. 144/90 mm Hg.

Lab Findings:

Hb                                           : 15 g/dL

WBC                                       : 10,000 u/mcL

Creatinine clearance        : 90 ml/min

ECG Finding :

There is ST elevation >2 mm in v2–v6, and >1 mm ST-elevation in lead 1 and aVL with some minimal reciprocal changes seen in lead 3 suggestive of anterior wall + lateral wall MI due to complete left anterior descending (LAD) occlusion. Likely in the proximal LAD.

Diagnosis : Acute Myocardial infarction

Treatment :

T. Aspirin                               75 mg            OD

T. Ticagrelor                        90 mg           BD (after initial loading dose of 180 mg)

T. Atorvastatin                    80 mg            OD HS

T. Metoprolol                      50 mg            OD

T. Ramipril                            2.5 mg           BD

T. Lasix + Spironolactone (20/50) mg   OD

1. What is the latest definition of STEMI?

2. What are the classical ECG criteria for diagnosing STEMI ?

3. What are the different types of MI ?

4. Justify the treatment given to this patient.

5. Determine the duration of DAPT In this patient ?

For Case Study-7 Answers | Go to Guidelines | Go to other Case Studies

Case Study-8 (Myocardial Infarction)

Mr TR, a 54-year-old man, presented to his general physician with sudden-onset epigastric pain that started the previous night and ‘felt like trapped wind’ . This radiated through his back, up to his neck and into both shoulders/arms. He was extremely flatulent.

The patient was referred for coronary angiogram/percutaneous coronary intervention (PCI), which was carried out on site the same day. It showed the presence of a thrombus in the OM (obtuse marginal coronary artery). The distal LAD (distal left anterior descending coronary artery) and the RCA (right coronary artery) were 80% and 60- 70% stenosed, respectively.

Echocardiogram: normal LV function.

Diagnosis: NSTEMI

Two drug- eluting stents were inserted into the OM. The following drugs were prescribed post-PCI:

  • Aspirin                                              75 mg                        daily
  • Clopidogrel                                     75 mg                        daily
  • Bisoprolol                                        2.5 mg                      daily
  • Ramipril                                           1.25 mg                     at night
  • Atorvastatin                                   40 mg                        at night
  • Lansoprazole                                  30 mg                        daily

Mr TR was discharged after 3 days, after an uneventful outpatient stay, with the drugs listed above and a GTN spray to be used sublingually. Arrangements were made for an exercise tolerance test (ETT) in 6 weeks to determine whether further PCI was indicated, and for Mr TR to enter the local cardiac rehabilitation programme.

  • Is the diagnosis & treatment given to Mr TR justified? Explain.
  • Is there a need of counselling?  

Go for Case Study-8 Answers | Explore more Case Studies | Go to Guidelines

Case Study-9 (Hyperlipidemia)

A 62-year-old male is referred for management of elevated cholesterol. He has history of obesity, hypertension, and hyperlipidemia. He had a non–ST-segment elevation myocardial infarction (NSTEMI) one year ago with drug-eluting stent placement in his right coronary artery. His current medications include aspirin 81 mg daily, lisinopril 20 mg daily, and metoprolol XL 50 mg daily. His physical exam is notable for a body mass index (BMI) of 32 kg/m 2  but is otherwise unremarkable. His blood pressure is 135/85 mm Hg.

A recent lipid panel shows the following:

  • Total Cholesterol: 226 mg/dL
  • Triglycerides: 154 mg/dL
  • High-Density Lipoprotein Cholesterol (HDL-C): 39 mg/dL
  • Low-Density Lipoprotein Cholesterol (LDL-C): 190 mg/dL
  • He has a normal creatinine and normal liver enzymes. His TSH and vitamin D levels are within normal limits.
  • What would be the target goal for LDL-C in this patient?
  • What is the drug of choice in this patient to treat LDL-C?
  • What would be the treatment plan, if the patient’s LDL-C goal is not reached even after initiation of statin therapy?
  • What would be choice of drug in this patient, if he is intolerant to statin therapy?

Go for Case Study-9 Answers | Explore more Case Studies | Refer Guidelines

Case Study-10 ( Hyperlipidemias)

Four months ago, a 46-year-old man was admitted to hospital with acute chest pain. A subendocardial inferior MI was diagnosed and he was treated with thrombolytics and aspirin. After discharge, he complained of angina, and coronary angiography was performed. This showed severe triple-vessel disease not suitable for stenting, and coronary artery bypass grafting was performed. He is attending a cardiac rehabilitation clinic and he has had no further angina since his surgery.

Family History:

He has a strong family history of ischaemic heart disease, with his father and two paternal uncles having died of myocardial infarctions in their 50s; his 50-year-old brother has angina. He is married with two children.

Social History:

He smokes 25 cigarettes per day and drinks at least 40 units of alcohol per week.

Medication History:

He is taking atenolol and aspirin.

Examination :

He is slightly overweight (85 kg; body mass index ) 28). He has tar-stained nails. He has bilateral corneal arcus, xanthelasmata around his eyes and xanthomata on his Achilles tendons. He has a well-healed midline sternotomy scar. His pulse is 64/min regular, blood pressure 150/84 mmHg. He has no palpable pedal pulses. His respiratory, gastrointestinal and neurological systems are normal.

Lab Investigation:

pharmacy patient case study examples

Urinalysis : no abnormality detected.

  • What is the metabolic abnormality present? 
  • Discuss the Patient counselling for this case?

Go to Case Study-10 Answers | Explore more Case Studies | Refer Guidelines

Case Study-11 ( Arrthymias )

(Arrhythmia) A 2 month-old male who presents to the emergency room with a chief complaint of fever, lethargy, and poor feeding for the past 36 hours. His parents began noticing increasing lethargy and tiring with feeding and increased work of breathing for about 12 hours prior to presentation.

He is the product of a G2P1, full term, uncomplicated pregnancy and spontaneous vaginal delivery. Nursery course was uneventful.

Vital Sign: Temp: 99.32 ᵒF,

        Heart Rate: 240 per min,

        Respiratory Rate: 72 per min,

        BP: 87/64mmHg, oxygen saturation 98% in room air.

He is well developed, well nourished, but pale, lethargic and tachypneic, with mild subcostal retractions. HEENT exam is normal . Neck is supple without adenopathy.

Lungs have good aeration with fine crackles and mild retractions. His heart is tachycardic with a regular rhythm. No murmur, rub, or valve clicks are heard. His abdomen is soft, non-distended, non-tender, and without masses. His liver is 2 to 3 cm below right costal margin. His feet and hands are cool.

His peripheral pulses are 1+ to 2+ (out of 4+) throughout. Capillary refill time is 3 to 4 seconds. He has no rashes or other significant lesions.

Chest x-ray shows mild cardiomegaly and mild pulmonary edema. A 12 lead electrocardiogram shows a narrow complex tachycardia (rate of 240 bpm) with no visible P-waves (rhythm strip below).

The patient is felt to be in supraventricular tachycardia and mild congestive heart failure.

A peripheral IV is started and he is given a rapid IV bolus dose of adenosine. The patient immediately becomes briefly bradycardic followed by resumption of a normal sinus rhythm at a rate of 140 beats per minute. He is admitted for overnight observation and initiation of an anti-arrhythmic medication.

A 12-lead electrocardiogram (ECG) following conversion shows no evidence of a delta-wave, so he is started on digoxin .

  • What medicine used to treat Supraventricular tachycardia is contraindicated specifically in Wolff-Parkinson-White syndrome?
  • What would be the possible differential diagnosis?

Go For Case Study-11 Answers | Explore More Case study | Go to Guidelines

Case Study-12 ( Atrial Fibrillation )

A 67-year-old man presents to the emergency department with palpitations and dyspnea which began approximately 4 hours ago. He has a history of hypertension, diabetes, and gastroesophageal reflux disease, and LVH. On further questioning, he reports drinking 1 cup of coffee daily and 1-2 beers on the weekends.

He denies binge drinking and the use of herbal or alternative medications. He quit smoking 10 years ago. His urine drug screen is negative. His body mass index is 36 and he admits to snoring and daytime sleepiness. He is at high risk for obstructive sleep apnea (OSA). His current medications include lisinopril, metformin, and omeprazole. He has no history of congestive heart failure, stroke, or transient ischemic attack (TIA).

He appears to be in mild respiratory distress.

Blood pressure is 88/60 mmHg, pulse rate is 140 bpm, respiratory rate is 24/min, and temperature is normal. Oxygen saturation is 90% on 40% oxygen by face mask. Cardiac exam reveals tachycardia with an irregularly irregular tachycardic rhythm. There are crackles in the lower lung fields.

Electrocardiogram demonstrates atrial fibrillation (AF) with rapid ventricular rate.

  • What would be the most appropriate initial management in this patient?
  • Calculate the risk of thromboembolism and risk of bleeding in this patient?

Go for Case Study-12 Answers | Explore more Case Study | Go to G uidelines

Case Study-13 ( Case study of Asthma )

A 29-year-old man with mild persistent asthma presented to an outpatient office for a follow-up visit.  He was originally referred 6 months ago by his primary care provider after having an asthma exacerbation which required treatment in an emergency room.

At his initial visit, he reported wheeze and cough 4 days a week and nocturnal symptoms three times a month.  Spirometry revealed forced vital capacity (FVC) 85% predicted, forced expiratory volume in 1 second (FEV 1 ) 75% predicted, FEV 1 /FVC 65%, and an increase in FEV 1  of 220 ml or 14% following an inhaled short-acting bronchodilator.  He was placed on a low-dose  inhaled corticosteroid  twice a day and a short-acting  inhaled beta-agonist  as needed. 

He returned 4 weeks later improved, but with continued daytime symptoms 2 days a week.  He also had symptoms of rhinitis; therefore he was referred to an allergist for evaluation. Skin testing was positive for trees, ragweed, dust mites, and cats, and he was prescribed a nasal steroid spray and nonsedating oral antihistamine.   He presents today and reports no asthma exacerbations since his last visit. 

Furthermore, during the past 4 weeks, he has not been awakened by his asthma, experienced morning breathing symptoms, missed work, had any limitations in activities due to asthma, or required the use of rescue albuterol.  He currently denies shortness of breath or wheezing.  He performs aerobic exercise 4 days a week for 45 minutes per session without symptoms, provided he premedicates with a short-acting inhaled beta-agonist.  His review of symptoms is otherwise unremarkable. 

His current medications include low-dose inhaled corticosteroid, 1 puff twice a day; steroid nasal spray, 2 puffs each nostril daily; a nonsedating antihistamine, 1 tablet daily; and inhaled beta-agonist, 2 puffs as needed.   

His  past medical history is significant for intermittent asthma diagnosed at age 13 and frequent “colds.”  He has never required hospitalization for an asthma exacerbation.  He works as a hospital microbiologist and does not smoke, drink alcohol, or use illicit drugs.  He recently moved to a pet-free apartment complex and instituted dust mite protective barriers for his bedding.  His family history is noncontributory.

Physical Exam

On physical exam, he is an age-appropriate man in no acute distress.  His height and weight are proportionate and resting oxygen saturation as measured by a pulse (SpO 2 ) is 98% on room air.  A Head and neck exam revealed mild erythema of the nasal mucosa.  A heart exam revealed normal heart tones, no murmurs, gallops or rubs, and the lungs were clear to auscultation.  Extremities were free of oedema, cyanosis, or clubbing.

Lab:  In the office, spirometry is completely normal. He states he feels great and inquires about stopping his inhalers, particularly his inhaled steroid.

1. Based on current evidence, which of the following would be the most appropriate recommendation regarding his asthma medication regimen?

A. Maintain current medication regimen; no adjustment is indicated.

B. Discontinue the inhaled corticosteroid; maintain on an Inhaled beta-agonist as needed.

C. Decrease the inhaled corticosteroid to 1 puff daily.

D. Discontinue the inhaled corticosteroid; start a leukotriene modifier at bedtime.

E. Discontinue the inhaled corticosteroid; start low-dose inhaled corticosteroid/long-acting beta-agonist, 1 inhalation at bedtime.

2. Which of the following should be done routinely with each follow-up visit?

A. Methacholine challenge testing

B. Sputum for eosinophils

C. In-office peak flow recordings

D. Review of proper inhaler technique and adherence

E. Measurement of exhaled nitric oxide (NO)

3. What findings would suggest that the patient requires a step-up in asthma medication?

A. Two or more nighttime awakenings per month due to Asthma.

B. Two or more interruptions in daytime activities per month due to asthma .

C. Peak flow readings 85% of personal best

D. Short-acting beta-agonist for rescue once a week

E. Short-acting beta-agonist 4 days/week prior to exercise

4. The patient was provided with an asthma action plan to follow at home. Which component of the asthma action plan is considered the most critical element for improving asthma outcomes?

A. A list of the patient’s controller and rescue medications

B. A list of symptoms indicative of worsening asthma

C. Medication changes based on personal-best peak flow readings

D. Instructions describing when, how, and how long to increase medications when symptomatic

E. Medication changes based on symptoms

5.  How often is spirometry testing recommended if the previous readings are normal and the patient’s asthma is well controlled?

A. Every 1 to 2 years

B. Only if asthma controller medications are changed

C. Only if symptomatic

D. Every 4 to 6 months

E. Every follow-up visit

Image result for asthma

Explore more  Case Studies  | Go to  Case Study-13  Answers  |  Guidelines

Case Study-14 ( Case Study - II of Asthma )

A 14-year-old girl with a history of asthma requiring daily inhaled corticosteroid therapy and allergies to house dust, mites, cats, grasses, and ragweed presents to the emergency department in mid-September, reporting a recent “cold” complicated by worsening shortness of breath and audible inspiratory and expiratory wheezing.

She appears frightened and refuses to lie down but is not cyanotic.

Her pulse is 120 bpm, and respirations are 32/min.

Her mother states that she has used her albuterol inhaler several times a day for the past 3 days and twice during the previous night. She took an additional two puffs on her way to the emergency department, but her mother states that “the inhaler didn’t seem to be helping so I told her not to take any more.

1. What emergency measures are indicated?

2. How should her long-term management be altered?

Case Study-14 Answer | More Case Studies | Guidelines

Case Study-15 ( Chronic Obstructive Pulmonary Disease : COPD Case Study)

Mrs. Glenda is a 60-year-old and has recently retires from her job working for a firm that manufactures fabrics. She is a thin lady who appears older than her stated age. Glenda visits her GP as she beginning to get short of breath while climbing the stairs and is struggling to walk to the end of her road. She explains to the GP that for the past six months she has become increasingly short of breath while carrying out daily activities.

Glenda takes no regular medications and has no history of drug allergies .

She says she has not had any acute changes in her breathing, but she does have a chronic cough that produces around one or two tablespoons of clear sputum daily. Her cough has not changed recently, and the colour and volume of her sputum have also remained unaltered. She says she is not suffering from chest pains or wheezing and has not been coughing up blood.

She has smoked at least ten cigarettes a day since she was 20 but has recently cut down to five a day because of her shortness of breath. In the last few years, she has had at least two chest infections each year requiring treatment with antibiotics.

The GP suspects COPD and conducts spirometry testing, the results of which are:

1. What clinical features and risk factors of COPD does glenda exhibit?  What grade of severity does glenda’s COPD fall into?

2. What initial treatment would you recommend for glenda?

3. Glenda continues to report that her breathlessness is getting worse. Her medical research council dyspnea score is now four and in the last few days she has been producing more sputum than usual. Her sputum has turned a yellow green colour. What does these changes indicate & what treatment would you recommend?

Explore more Case Studies | Guidelines | Case Answers of this COPD Case Study-15

Case Study-16 ( Case of COPD with comorbidity )

A 63-year-old woman, 67 kg, is admitted to hospital with chest pain, shortness of breath and sweating. She is seen in casualty and treated using a salbutamol nebuliser. She looks obese. She has been a life-long smoker who stopped one day ago.

Her previous medical history includes chronic obstructive pulmonary disease (COPD) for 10 years, the last admission to hospital was two weeks ago; ischaemic heart disease since 1995, myocardial infarction 4 years ago; osteoporosis diagnosed 3 years ago; hypertension diagnosed 9 years ago; and pulmonary embolism two months ago.

On examination:

Blood pressure                    105/90 mmHg

Heart rate                             90 bpm

Respiratory rate                 20 breaths per minute.

Arterial blood gases on admission:

pH                                           7.388 on 35% O2

PCO2                                      9.67 kPa

PO2                                        6.5 kPa.

Oxygen saturation: SpO2 89%. Lungs were hyperinflated, no wheeze, few right base crepitations.

Laboratory tests at admission  :

WCC                 16.5 × 109/L                    (4–11 × 109/L)

Na+                   140 mmol/L                     (135–145 mmol/L)

K+                      4.4 mmol/L                      (3.5–5 mmol/L)

Creatinine     75 micromol/L            (59–104 micromol/L)

Urea                 7.8 mmol/L                       (1.7–8.3 mmol/L)

Hb                     11.6 g/dL                            (13–17 g/dL)

Medication on admission:

Prednisolone                   10 mg                    o.d.

Fluticasone inhaler         500 micrograms  b.d.

Aspirin                              75 mg                    o.d.

Bumetanide                    1 mg                        o.d.

Combivent nebs             2                              q.d.s.

Enalapril                          5 mg                        o.d.

Uniphyllin Continus       200 mg                    bd

Senna                              2 tablets                  nocte

Warfarin                          5 mg                        o.d.

Zopiclone                        7.5 mg                    nocte

Diclofenac                       50 mg                     p.r.n.

Oxygen  (O2)                                     2 L nasal specs.

1.  How the sign, symptoms & pathophysiology of COPD, relates to the patient?

2.  Comment on the current drug therapy and describe the role of O2 in this patient.

3.  What are the social issues in treating this patient at home?

Case Study-16 Answer  | Explore more  Case Studies  |  Guidelines

Case Study-17 ( Type-1 Diabetes in Pregnancy )

Mrs Jaya is a 36-year-old married lady who has type 1 diabetes. She undertook a home pregnancy test because she was feeling particularly nauseated in the mornings and her period was late. The  test was positive  confirming that she was pregnant.

However, at 8 weeks, she experienced  vaginal bleeding  and  abdominal pain . She attended the Accident & Emergency department, where a miscarriage was confirmed.

Upon questioning, it was discovered that she had been taking  folic acid 400 μcg daily  for the previous 6 months but had not received any pre-conception diabetes care.

Her most recent HbA1c was 7.3% (56 mmol/mol). Her regular medications are  ramipril 10 mg daily ,  simvastatin 40 mg daily ,  insulin glargine at night  and  insulin aspart three times daily  with meals.

  • Why should women of childbearing age be offered advice about pregnancy?
  • Was she taking appropriate dietary supplements prior to conception?
  • What advice should she be given with respect to her regular medication?

Go to Case Study-17 Answers

Case Study-18 (Type-2 Diabetes)

Case scenario:.

Mk a 68-yr-old woman who has previously diagnosed with type 2 diabetes 15 years ago came to the clinic for her regular follow-up. she reports paraesthesia’s in her feet at night but her sleep was not disturbed by these symptoms. She checked her blood sugars at home two to four times each day and found them to range from 100-250 mg/dl.

She reported compliance with her medications that included lisinopril 20 mg OD, hydrochlorothiazide 25 mg OD, diltiazem 180 mg OD, glargine insulin 0.4 units/kg , atorvastatin 40 mg , and aspirin 81 mg OD.

Past medical history & medication history :

  • Type 2 diabetes, hyperlipidaemia and hypertension diagnosed at the age of 47. For these conditions she was treated with metformin, atorvastatin, and hydrochlorothiazide and maintained Hb A1c below 7.0 %, LDL below 100 mg/dl, and blood pressure below 135/80 mmHg for many years.
  • At the age 59, she was noted to have an increase in her urine albumin to 54 mg/g of creatinine and lisinopril was added to her regimen.
  • Three years later at the age 62, her serum creatinine increased to 1.56 mg/dl, and her GFR was estimated to be 36 ml/min. Her Hb A1c increased to 7.2% and she began to experience paraesthesia’s in her feet. The metformin was discontinued, and she was started on glargine insulin.
  • At age 63, she developed proliferative retinopathy in her right eye and underwent laser photocoagulation. Bilateral macular edema developed at age 65.

Examination – Present visit:

  • BMI – 38.2 kg/m² (height – 151 cm, weight – 87 kg)
  • Blood pressure – 142/90 mmHg
  • Heart rate – 68 bpm
  • Retinal exam – significant for panretinal photocoagulation changes with scattered dot haemorrhages and macular edema in both eyes.
  • Heart, lung and abdominal exams were unremarkable.
  • Extremities – Pulses in her feet were reduced, but her feet were warm and without ulcers. She was unable to detect a Semmes Weinstein 5.07 monofilament on the soles of her feet.

Lab parameters:

  • Hb A1c : 7.9%.
  • Serum creatinine : 2.45 mg/dl.
  • GFR : 20 ml/min.
  • Calcium : 9.8 mg/dl. Phosphorus                                                  : 4.6 mg/dl.
  • PTH : 75 pg/ml
  • Total cholesterol : 188 mg/dl,
  • Triglycerides :82 mg/dl,
  • High-density lipoprotein cholesterol :42 mg/dl,
  • LDL cholesterol  : 131 mg/dl
  • Hb  : 10.8 g/dl.
  • Prepare pharmaceutical care plan for this case study.

Go to Case Study-18 Answers

Case Study-19 (Thyroid Disorder)

A 55-year-old man presents to his general practitioner, complaining of lack of energy. He has become increasingly tired over the past 18 months. He works as a solicitor and describes episodes where he has fallen asleep in his office.

He is unable to stay awake after 9:30 pm, and sleeps through until 7:30 am. He finds it difficult to concentrate at work, and has stopped playing his weekly game of tennis. He had an episode of depression 10 years ago related to the break-up of his first marriage. He has no current personal problems. He has had no other major illnesses.

His brother developed type 1 diabetes mellitus at the age of 13. On direct questioning, he has noticed that he has become more constipated but denies any abdominal pain or rectal bleeding. He has put on 8kg in weight over the past year.

Examination:

On examination he is overweight. His facial skin is dry and scaly. His pulse is 56/min, regular and blood pressure 146/88 mmHg. Examination of his cardiovascular, respiratory and abdominal systems is unremarkable. Neurological examination was not performed.

Haemoglobin                                               10.3 g/dL                              13.3–17.7 g/dL

Mean corpuscular volume (MCV)          92 fL                                       80–99 fL

White cell count                                         4.3 x 10⁹/L                            3.9–10.6 x 10⁹/L

Platelets                                                        154 x 10⁹/L                           150–440 x 10⁹/L

Sodium                                                          140 mmol/L                         135–145 mmol/L

Potassium                                                    4.4 mmol/L                          3.5–5.0 mmol/L

Urea                                                               6.4 mmol/L                          2.5–6.7 mmol

Creatinine                                                    125 μmol/L                        70–120 μmol/L

Glucose                                                         4.7 mmol/L                          4.0–6.0 mmol/L

Calcium                                                         2.48 mmol/L                        2.12–2.65 mmol/L

Phosphate                                                    1.20 mmol/L                        0.8–1.45 mmol/L

Cholesterol                                                  6.4 mmol/L                          3.9–6.0 mmol/L

Triglycerides                                                1.4 mmol/L                          0.55–1.90 mmol/L

Urinalysis: nothing abnormal detected (NAD)

  • What is the likely diagnosis? Main differential diagnosis?
  • How would you further manage this patient?

Go to Case Study-19 Answers

Case Study-20 (Thyroid Disorder)

A 40-year-old woman (51kg) presents with complaints of left side chest pain since 4 months that last for 1-2 hours and radiates to back, increase on exertion; associated with palpitation, perspiration. Headache since 1 month in occipital region that lasts 30-40 min. Generalised weakness, easy fatigue, facial swelling, peri-orbital swelling, bilateral lower limb swelling.

Past medication history :

Blood transfusion 2 times 4 months back.

Temp : Afebrile                               Pulse: 86 beats/min

BP: 130/84mmHg (pattern for hypotension seen for 2 days)

Lab details:

Hb                                           10.1 g%                                 13-17g%

WBC                                      4600 cumm                        4000-11,000 cumm

PCV                                        31.3%                                    36-47%

MCV                                       85.2fL                                   82-92fL

MCH                                       27.6pg                                   27-31pg

MCHC                                    32.3%                                    32-36%

Platelet                                1.55 lac/cumm                    1.5-4.5 lakh/cumm

RBC                                        3.67 millions/ μ L              4.4-5.9 millions/ μ L

RDW                                      17.7%                                    13-15 %

Pus & epithelial cells        1-2/hpf

Reticulocyte count            0.5%                                       0.5-2.5%

Na                                           135 mmol/L                        135-145mmol/L

K                                              4.2mmol/L                           3.5-5.5mmol/L

Cl                                            100mmol/L                          98-110mmol/L

Creatinine                           0.8mg%                                0.6-1.3 mg%

Glucose : RBS                      103mg%                               70-140mg%

RBC Smear : Mild anisocytosis                                                                                  

Stool Test ( Occult Blood) : Positive

TSH                                         >100 miu/ml                       >15miu/ml

Total T4                                0.10 ng /dL                          0.82- 2ng/dL                      

Free T4                                  0.10ng /dL                           0.9- 2.3 ng/dL        

ECG : normal sinus rhythm , poor P-wave progression

LVEF: 60%; Grade 1 diastolic function; mild MR; mild TR; mild PAH; Concentric LVH

            Local part

Findings: Left lobe P/O  — hyperplastic nodule

                  Both lobe P/O – inflammatory etiology thyroiditis

Inj pantoprazole    40mg                                                                         12hrly

Infusion 0.45% NS 500ml + 2 ampoule Optineuron at 60cc/hr

Tab Fdson MP Forte                                                                                0-1-0

Tab febac XT                                                                                             1-0-1

Tab Thyronorm      25mg                                                                        1-0-0

  • What is the likely diagnosis in this case? And are lab finding clinically justified?
  • What are the Pharmacist Intervention points in this case?
  • Patient counselling regarding drug & disease?
  • Write the Generic Names of above advised drugs.

Go to Case Study-20 Answers

Case Study-21 (Thyroid Disorder)

A 34-year-old pregnant woman came to the emergency department with complaints of fetal movements not felt since 1 day ago.

At present the patient is the third pregnant at the age of 23-24 weeks, so far the regular control to the midwife and specialist doctors.

The patient has never experienced bleeding or trauma before. A history of previous labour was normal with a healthy baby. The patient has been suffering from Grave’s disease for the past 10 years and has not been treated for one year.

During pregnancy, the patient feels palpitations and tightness during activity. The patient also complaint about weight loss and fatigue.

The patient’s general condition appeared weak with a blood pressure of 197/87 mmHg and pulse 148 times per minute regular.

The exophthalmos and the thyroid gland feel soft in the neck without pain.

Laboratory tests results:

 An increase in FT4 levels of 75.62 pmol / L and low TSHs levels of 0.005 µIU / mL. Wayne index with a value of 23 or found signs of toxicity and the Burch Wartofsky scale with a value of 45 or impending thyroid storm.

Q1 . What is the likely diagnosis ?

The patient was taken to the endocrine section and given methimazole 30 mg twice daily, propranolol 30 mg twice daily, and Lugol 5 drops per 6 hours .

The termination of pregnancy is carried out by vaginal delivery and administration of oxytocin postpartum.

The patient complained of shortness of breath and anxiety after two hours postpartum.

Patient was consulted in the Cardiology division with pulmonary oedema and advised giving diuretics and vasodilators .

Patient was given nitroglycerin at a dose of 5 meq per hour and furosemide 30 mg per hour intravenously to improve the condition of pulmonary oedema.

Close monitoring is carried out on the patient for several days until the patient’s condition is stable.

Q2. Was the Treatment given to the patient justified according to clinical condition?

Q3. Widely used anti-thyroid drugs in pregnant women with hyperthyroidism?

Patient is planned to go home for outpatient care after monitoring side effects and postpartum complications. Hemodynamic condition is stable with blood pressure 110/70 mmHg and pulse 92 times per minute. Patient was given anti-thyroid therapy and beta-blockers on discharge. During treatment at home, patient is advised to monitor thyroid function to the clinic every once a month.

Q4. What should be the Patient Counselling points ?

Go to  Case Study-21 Answers

Case Study- 22 (Thyroid Disorder)

A 55-year-old woman presents with complaints of pus discharge per vagina since 8-9 months. Decreased urine output. Palpitation 3-4 months. Shortness of breath 4-6 months

Past medical history :

Hysterectomy 10-12 years ago

Social History :

“ Chulha ” user

Temp : Afebrile                               Pulse: 140 beats/min

BP: 126/80mmHg

Provisional Diagnosis :

Atrial Fibrillation w/ hemorrhoids w/ hyperthyroidism w/ perianal fissure.

Hb                                           13.2 g%                                 13-17g%

WBC                                       8100 cumm                         4000-11,000 cumm

Platelet                                  2.11 lac/cumm                    1.5-4.5 lakh/cumm

Pus cell                                  5-6

Epithelial cells                     1-2/hpf

Na                                           142 mmol/L                        135-145mmol/L

K                                              3.8mmol/L(3 rd day: 2.9)          3.5-5.5mmol/L

Cl                                             106mmol/L                         98-110mmol/L

Creatinine                             0.6mg%                                0.6-1.3 mg%

Urea                                       24

Mg                                          2.1

Ca                                          8.6

Protein                                 6.1 g %                                    6.0-8.0 g %

Albumin                                3.4g%                                    2.7-5.0 g %

Globulin                                2.7 g%                                   2.5-4.0 g %

SGOT                                     59

SGPT                                      27

Bilirubin                                1.6

TSH                                         0.01                                        0.39-5.0

Free T3                                  7.01                                        2.1-3.8

Free T4                                  4.28ng /dL                            0.9- 2.3 ng/dL         

CRP                                         <10                                         0-6.0

ECG : Flat T- wave (V5) atrial fibrillation.

ECO : Mild MR; Mild TR; LVEF: 60%

Inj  Dilzem                            12.5mg                                 stat w/ 10ml NS

Tab Dilzem                           30mg                                     1-1-1

Inj Lasix                                20mg                                     1-0-0

Inj PAN                                  40mg                                     24 hrly

Tab Metro                           400mg                                  1-1-1             

Syp Duphalac                     2tsp                                       0-0-1             

Tab Ciplox T2                      500/600                               1-0-1             

Tab Neomercazole            10mg                                                 1-1-1             

Tab Propanolol                  10mg                                    1-0-1

Syp Potklor                          15ml                                     1-1-1 

Metrogyl ointment                                                           1-1-1-1 

Inj NS + KCl 1 amp                                                             12hrly                       

  • Are there any drug-drug interactions?
  • Is the dosing of medication for hyperthyroidism, according to standard treatment guidelines?
  • What should be Patient counselling in this case, regarding drug & disease?

Go to Case Study-22 Answers

Case Study-23 (OB/GYN: Oral Contraceptive Use)

An 18-year-old female presents with an absence of periods for 6 months. This has occurred twice before in the past but on both occasions menstruation returned so she was not too concerned. Her periods started at the age of 12 years and were initially regular.

She has no medical history of note and denies any medication. She is currently in her first year at university. She sometimes follows inconsistent diet plan. However, She runs most days and reports a ‘healthy ‘ diet avoiding carbohydrate foods and meat. She is the oldest of three siblings and her parents separated when she was 12 years. She has minimal contact with her father and lives mainly with her mother who she says she gets on well with. She has had a boyfriend in the past but has veered away from any sexual relationships.

The woman is tall and thin with a body mass index (BMI) of 15.5 kg/m².

There is evidence of fine downy hair growth on her arms.

Heart rate is 86/min and blood pressure 100/65 mmHg.

Abdominal examination reveals no scars or masses, and genital examination is not performed.

INVESTIGATIONS:

Follicle-stimulating hormone                  1.0 IU/L                     Day 2-5

 Luteinizing hormone                                0.8 IU/L                     Day 2-5

0.5-14.5 IU/L

Oestradiol                                                    52 pmol/L                 70-600 pmoI/L

Prolactin                                                       630 mu/L                  90-520 mu/L

Testosterone                                               1.6 nmol/L                0.8-3.1 nmol/L

Diagnosis : Hypogonadotrophic hypogonadism

  • What should be the further investigation ?
  • How, this patient would be managed ?
  • What Patient counselling points should be included in this case?

Go to Case Study-23 Answers

Case Study-24 (OB/GYN: Oral Contraceptive Use)

A 19-year-old female was referred by her general practitioner with increased hair. She first noticed the problem when she was about 16 years old and it has progressively worsened such that she now feels very self-conscious. It also affects her forming relationships.

The hair growth is noticed mainly on her arms, thighs and abdomen. Hair has developed on the upper lip more recently. She has tried shaving but this seems to make the problem worse. She feels depilation creams are ineffective. Waxing is helpful but very expensive and she has bleached her upper-lip hair.

Her GP has not prescribed any medication in the past.

There is no significant previous medical history of note. Her periods started at the age of 13 years and she bleeds every 30-35 days. The periods are not painful or heavy and there is no intermenstrual bleeding or discharge. She has never been sexually active.

Examination

On examination she has an increased body mass index (BMI) of 29 kg/m². The blood pressure is 118/70 mmHg. There is excessive hair growth on the lower arms, legs and thighs and in the midline of the abdomen below the umbilicus. There is a small amount of growth on the upper lip too.

The abdomen is soft and no masses are palpable. Pelvic examination is not indicated.

INVESTIGATIONS

Follicle-stimulating hormone (FSH)                  7 IU/L                         Day 2-5

                                                                                                                        1-11 IU/L

Luteinizing hormone (LH)                                    12 IU/L                      Day 2-5

                                                                                                                        0.5-14.5 IU/L

Prolactin                                                                   780 mu/L                  90-520 mu/L

Testosterone                                                           3.2 nmol/L                0.8-3.1 nmol/L

Thyroid-stimulating hormone                            4.9mu/L                    0.5-5.7 mu/L

Free thyroxine                                                         14.7pmol/L              10-40pmol/L

Provisional Diagnosis : Polycystic ovarian syndrome(PCOS)

Clinical features: Hirsutism , acne, increased BMI and slight menstrual irregularity

Go to Case Study-24 Answers

Case Study-25 (Hormone Replacement Therapy)

History of present illness :.

A 51-year-old newly postmenopausal woman suffering from intense hot flashes and night sweats, as well as anxiety, insomnia, and stress, presents to the hospital.

The doctor presents a breakdown of the patient’s self-reported symptoms before going into the details of her test results – where she highlights the biochemical factors that explain both her symptoms and her observed results.

Symptoms she rated as severe , included, depression, anxiety, and sleep disturbances , moderate symptoms included hot flashes, night sweats, foggy thinking, vaginal dryness, and mood swings. In addition, she had multiple symptoms that she rated as mild in severity.

The Patient had already tried multiple supplements in an attempt to address her own symptoms – these included elements like magnesium, calcium, selenium, zinc, and copper, as well as L-theanine and Rhodiola, vitamin D, fish oil, and some probiotics.

Investigation:

Saliva Hormone Test Results:

Name                                                      Lab value                     Normal range

Estrogen hormone                                     0.3                                0.9-3.1 pg/ml

Progesterone hormone                            5                                   12-100 pg/ml

DHEA                                                           1                                   2-23 ng/ml

Testosterone                                              90                                 16-55 pg/ml

Neurotransmitter Test revealed:

LOW levels of:

Nor Epinephrine

  • What should be the further investigation for this patient ?
  • Give the provisional diagnosis for this case.

Go to Case Study-25 Answers

Case Study-26 (Hormone Replacement Therapy)

A 25 year-old woman with menarche at 13 years and menstrual periods until about 1 year ago.

Complains of hot flushes, skin and vaginal dryness, weakness , poor sleep and scanty and infrequent menstrual periods of a year duration.

She visits her gynecologist, who obtains plasma levels of follicle-stimulating hormone and luteinizing hormone, both of which are moderately elevated.

She is diagnosed with premature ovarian failure, and estrogen and progesterone replacement therapy is recommended.

A dual energy absorptiometry scan (DEXA) reveals bone density t-score f <2.5 SD, ie., Frank osteoporosis.

  • How should the ovarian hormones she lacks be replaced?
  • What extra measures should she take for her osteoporosis while receiving treatment?

Go to Case Study-26 Answers

Case Study-27 (Osteoporosis)

History of present illness:.

     A 73-year- old woman presenting for a physical examination who looks and feels healthy and well.

Medication history:

Takes a multivitamin daily plus a calcium tablet

History of low-trauma Colles’ fracture (11 years ago)

Physical Examination:

Weight:  55 kg (121 lbs.)

Height: 157 cm (5’2”)

Body Mass Index (BMI): 22.3 kg/m2

*Changes in height and weight can be signs of vertebral fractures

INVESTIGATION:

BMD :                 Value                       

Spine                -3.6;

Hip                     -2.0

Provisional Diagnosis : Osteoporosis with moderate risk of fracture

  • What all investigations are needed for the further management?
  • How this patient would be managed ?
  • How will you do the Patient counselling in this case?
  • Mention the indications for BMD testing.

Case Study-28 (Osteoporosis)

A 64-year-old retired firefighter Retired nine years ago; now doing contract carpentry Presents for physical examination, complaining his back has been “worse than usual” the past three weeks.

On no medications

Prior smoker (45 pack/year history)

Quit smoking one year ago

Height: 180 cm (5’11”)

Patient recalls being 185.5 cm (6’1”)

Weight: 80 kg (up 5 kg from one year ago)

Body mass index (BMI): 24.7 kg/m2

*Other indicators of vertebral fracture in physical examination: Rib-pelvis distance and occiput-wall distance

INVESTIGATION:      

  Screening for osteoporosis with dual energy X-ray absorptiometry (DXA) is  T-score -1.9 at femoral neck

Lateral thoraco-lumbar spine X-ray is ordered to rule out vertebral compression deformities

*The radiologist makes note of two vertebrae being wedge shaped and just meeting the criteria for vertebral compression fracture

  • What is the significance of T- Score?

Go to Case Study – 28 Answers

Case Study-29 (Glaucoma)

The patient, a 61 year-old female retired school teacher, has not had an eye exam in 10 years. She reports no problems with driving, watching TV, computer or reading. She uses over-the-counter readers for close work. She admits to mild eye burning feeling after reading for long periods of time or in the afternoons. She denies any flashes, floaters, pain, redness or double vision.

Past Ocular History:

Presbyopia. No prior eye surgeries, hx of eye trauma, amblyopia or strabismus.

Ocular Medications:

Hypertension

Surgical History:

Cesarean delivery x 1

Past Family Ocular History:

Cataract surgery in her mother and father. Negative for macular degeneration, glaucoma or blindness.

Never smoked

Medications :

Hydrochlorothiazide

Allergies :

Denies any recent illness or any new CNS, heart, lungs, GI, skin or joint symptoms.

Ocular Exam:

Visual Acuity (cc):

IOP (tonoapplantation):

OD: 21 mmHg

OS: 23 mmHg

Equal, round and reactive to light, no APD

Extraocular Movements:

Full OU, no nystagmus

Confrontational Visual Fields:

Full to finger counting OU

Normal, both sides

Lids and Lashes:      Normal OU

Conjunctiva/Sclera:         Normal OU

Cornea:                 Clear OU; no krukenberg spindle or embryotoxon

Anterior Chamber: Deep and quiet OU

Iris:              Normal, no neovascularization or atrophy

Lens: 1+ nuclear sclerotic cataracts OU

Anterior Vitreous: Clear OU

Dilated Fundus Examination:

OD:    Clear view, CDR 0.7 with sharp optic disc margins (no obvious rim thinning or disc hemorrhage); flat macula with normal foveal light reflex; normal vessels and peripheral retina.

OS:    Clear view, CDR 0.8 with sharp optic disc margins (no obvious rim thinning or disc hemorrhage); flat macula with normal foveal light reflex; normal vessels and peripheral retina

Gonioscopy: Open angles with minimal pigmentation in the trabecular meshwork, no synechiae OU

Automated visual field test: superior arcuate defect in both eyes

Pachymetry: 560 OD; 551 OS (within normal range)

  • What findings are needed for the diagnosis of POAG?
  • Explain the result of the above mentioned investigations of the ocular exam?
  • Enlist the Patient counselling points in this case?

Go to Case Study – 29 Answers

Case Study-30 (Galucoma)

The patient is a 51 year-old stay-at-home mother who presented to the ED with severe R brow, R cheek and R eye pain/pressure that started 2-3 hrs prior. She also noticed blurry vision from that eye and rainbow-colored halos around lights around the same time. 

Accompanying symptoms include acute nausea. She has vomited twice since feeling eye pain. Denies prior episodes. Denies flashes, floaters or diplopia. There is mild redness in the R eye.

Hx myopia OU No prior eye surgeries, trauma, amblyopia or strabismus

Degenerative disc disease – lower back

Father: chronic angle-closure glaucoma. No FHx of macular degeneration or other blinding diseases.

30 pack/year smoking history Drinks alcohol on occasion No illicit drug use

Medications:

Multivitamins Vicodin prn (uses few days/month for back pain)

Denies recent illnesses, new medications, CNS, lungs, GI, skin, joint problems except for above.

Ocular Exam

Visual Acuity (cc): OD: 20/70 OS: 20/20 IOP (tonoapplantation): OD: 62 mmHg OS: 11 mmHg Pupils: OD: pupil mid dilated, sluggish to respond to light. OS: pupil round and reactive to light No obvious APD Extraocular Movements: Full OU No nystagmus Confrontational Visual Fields: Full to finger counting OU External: Normal, both sides

Lids and Lashes: Normal OU Conjunctiva/Sclera: Mild diffused injected conjunctiva OD, Normal OS Cornea: Hazy cornea OD, Normal OS Anterior Chamber: Shallow anterior chamber 360 OD – hazy view, Deep and quiet OS Iris: Mid dilated iris OD, Normal OS Lens: Trace nuclear sclerosis OU Anterior Vitreous: Clear OU

OD: Dilation not performed, examination through undilated pupil showed hazy view, CDR 0.5 with sharp optic disc margins; flat macula OS: Dilation not performed, examination through undilated pupil showed clear media, CDR 0.4 with sharp optic disc margins; flat macula

Gonioscopy: Performed with Abraham 4 mirrored lens, shallow angle and no view of the angle structures 360 degrees OD, shallow angle with view of the trabecular meshwork 360 degrees OS without synechiae, mild pigmentation 360 degrees OS

1. What is the Diagnosis in this case? 2. What is the mechanism of angle closure in an episode of acute angle closure glaucoma resulting from a pupillary block? 3. Mention in brief the surgical options for the disease? 

Go to Case Study – 30 Answers

Case Study - 31 (Conjunctivitis)

The patient is an 31 year-old male who reports a 4-day history of irritation and itching, first in the L eye followed by the R eye one day later. Both eyes have also had a mild yellow discharge and mattering of the eyelids making it difficult to open the eyes in the morning. There is minimal eye redness but no foreign body sensation, flashes, floaters, decreased vision or diplopia. Not using any drops. No environmental exposures to the eyes.

No history of eye trauma, surgery, amblyopia or strabismus. No history of contact lens use.

Born at term without complications

No history of glaucoma, macular degeneration or other blinding diseases

No smokers at home

Exposure to the common cold (neighbor friend). No history of environmental allergies, recent cold, CNS, heart, lung, GI, skin or joint problems.

OD: 17 mmHg

OS: 14 mmHg

Equal, round and reactive OU, no APD

Normal-appearing orbital structures; no redness or swelling either eye

Lids and Lashes:           Crusted dry flaky material on eyelashes OU, no follicles in inferior or superior fornix OU. No foreign body in fornices OU

Conjunctiva/Sclera               Mild conjunctival injection OU, no chemosis

Cornea                           Clear OU, no infiltrates

Anterior Chamber               Deep and quiet OU

Iris                       Normal OU

Lens                   Normal OU

Anterior Vitreous            Clear OU

Dilated Fundus Examination :

OD     Clear view, CDR 0.2 with sharp optic disc margins, flat macula with normal foveal light reflex, normal vessels and peripheral retina

OS      Clear view, CDR 0.2 with sharp optic disc margins, flat macula with normal foveal light reflex, normal vessels and peripheral retina

No preauricular or submandibular lymph node enlargement

  • What are the patient counselling points in these case?
  • What is not a typical exam finding of conjunctivitis?

Go to Case Study-31 Answers

Case Study - 32 (Conjunctivitis)

History: A 35 year old female presented with complaints of sticky eyelids, watery and green ocular discharge, redness, soreness and slightly blurred vision in both eyes.

The symptoms started 2 weeks ago; the right eye was affected first.

Initially prescribed with Chloramphenicol (0.5%), no improvement was reported.

Chloramphenicol was replaced with Tobramycin, improvement was noticed only on 1 st day.

·         Ocular swab taken for PCR and was advised to stop tobramycin.

Again, on the visit, she reported that the left eye now felt worse and noticed an increase in green discharge, however a decrease in redness since the drop discontinuation.

Past medical history : A metal foreign body removal from the right eye – 10years ago.

She had flu, 3 weeks ago but is in good health now (was not atopic and she was not taking any medications.)

Vision: OD :    6/6                                                      OS :     6/5

Slit lamp examination:

OD: Mild conjunctival injection. Mild follicular change of the inferior palpebral conjunctiva. No corneal surface involvement. No anterior chamber inflammation.

OS: 360 degree conjunctival injection grade 3. Multiple white nodules at the limbus. Mild follicular change of the inferior palpebral conjunctiva. No anterior chamber inflammation.

Other Investigation:

PCR test : negative for Adenovirus, Varicella-Zoster virus, Herpes Simplex virus and Chlamydia trachomatis.

Provisional Diagnosis: Bacterial Conjunctivitis

For left eye:

Instill one drop of ofloxacin                     q2h

* reduce the dosage to “qid” the next day if the condition improves.

For right eye:

Ofloxacin                                                      qid     

Revisit after 1 week.

  •   What are the possible differential diagnosis in this case?
  •   What is the role of Ofloxacin in this Patient’s case?
  •   Why Tobramycin wasn’t readvised after the PCR result, is there any evidence suggestive for the change?

 Go to Case Study -32 Answers

Case Study - 33 ( Pediatrics)

Leanne is a 6-year-old girl whose teachers have suggested that her parents take to her GP. They have noticed that she seems to have problems listening and to be daydreaming a lot in class. The GP asks whether her parents have also seen her daydream. Her mother has, but has not thought much about it. However, more recently, it seems to have been happening more frequently. On direct questioning by the GP, Leanne’s mother thinks that these daydreams or ‘trances’ as she calls them sometimes occur when Leanne is in the middle of doing or saying something, and they interrupt her activity. Leanne’s birth and early medical history, including her development, have been normal. There was a history of epilepsy on her father’s side of the family. Her younger brother and older sister are well.

1.What should Leanne’s GP consider as a possible diagnosis?

2. What further investigations should the paediatrician or paediatric neurologist Request, and what considerations should be taken into account?

3. What should the discussion around medication include, and what medications may be Prescribed?

Go to Case Study- 33 Answers

pharmacy patient case study examples

Answers for Case Studies for Pharm.D / PharmD students:

Case study-1 answers.

  • According to AHA guidelines the target BP goal for hypertensive patients with CAD is <130/80 mmHg.
  • Beta blockers, ACE inhibitors, ARB’s, thiazide diuretics, and calcium channel blockers (CCB). ( Beta – blocker , first line agent to treat HTN in patients with Angina).

Explanation : stepwise                                 Go to CASE STUDY 1

Step 1:  In HTN patients with angina as comorbidity Beta blockers are the first line agents. Here metoprolol ( beta- blocker is already prescribed). Check whether target BP goal is achieved or not. Here target BP goal is not achieved in this patient (noted as 166/93 mmHg on his today’s visit to clinic). Follow step-2 if target Bp goal is not achieved. Step-2 :  Prescribe the maximum dose of metoprolol (100 mg twice daily). In this case already maximum dose is prescribed. Follow step 3 if target BP goal is not achieved. Step-3 :  Add another anti-hypertensive agent from ACEI’s / ARB’s / Thiazide diuretics/ CCB’s to the Beta -blocker. Note :  Here the patient is tolerant to METOPROLOL , however if the patient is intolerant to METOPROLOL ,Consider ACEi/ARB as first line agent and add CCB/thiazide as adjuvant.

       Go to CASE STUDY 1  

Case Study-2 Answers

Poor diet control, non-compliance to medication and obesity were considered to be the main reasons for poorly controlled hypertension and diabetes in Mr. MK

  • HbA1C target value should be less than 6.5%
  • Target value for BP should be 130/80 mm Hg.
  • Achieving these targets found to be beneficial in decreasing the Cardiovascular morbidities and prevention or delay of CKD progression.

3.Ans)   FOR HTN:                                                         Go to  Case Study 2.

For diabetic patients whose blood pressure levels are ≥160/100 start with two anti-hypertensive agents. Diabetic and hypertensive Patients with albuminuria consider ACEI /ARB as first line agent and add adjunctive drug from classes calcium channel blockers or Diuretic. Eg : Telmisartan (ARB) + Furosemide (diuretic)

FOR DIABETES: Metformin is the preferred initial pharmacologic agent for the treatment of type 2 diabetes. Once initiated, metformin should be continued as long as it is tolerated and not contraindicated; If the A1c levels are not achieved within the target levels, consider other class of agents to metformin. LIRAGLUTIDE (GLP1 AGONISTS) has the benefit of decreasing both the HBA1c values as well as body weight. So replacing GLICLAZIDE with LIRAGLUTIDE increases the effectiveness of treatment. Rx : Metformin + LIRAGLUTIDE .

4. Ans) DRUG INTERACTION:

AMLODIPINE+METFORMIN  – Amlodipine decreases effect of Metformin by pharmacodynamic antagonism. Management  – Monitor closely after withdrawal of Amlodipine whether the blood sugar levels are decreased or any hypoglycemic condition is observed.

5.Ans) Patient counselling:

  • Counsel the patient regarding importance of medication adherence.
  • Suggest to maintain regular exercise or at least do walking for 1 hour as it will be more beneficial for the weight loss.
  • Follow  DASH  diet to control HTN and reduce cardiovascular comorbidities.
  • Reduce intake of saturated fat and trans-fat; increase of dietary n-3 fatty acids, viscous fiber, and plant stanols/sterols.
  • Counsel the patient for regular checkup of A1c levels at least four times yearly.

Considerations:                                         Go to  Case Study 2.

  • LIRAGLUTIDE -1.2MG SC as GLP 1 agonists mainly decrease the HbA1c value (~0.8% -1.6%)and it also decreases body weight to (1-3kgs). Monitor closely while using liraglutide as it is contraindicated in medullary thyroid carcinoma and Monitor the calcitonin levels .
  • Metformin is contraindicated in patients with an eGFR ,30 mL/min/1.73 m2;
  • eGFR should be monitored while taking metformin;
  • The benefits and risks of continuing treatment should be reassessed when eGFR falls ,45 mL/min/1.73 m2;
  • Metformin should not be initiated for patients with an eGFR ,45 mL/min/1.73 m2; and
  • Metformin should be temporarily discontinued at the time of or before iodinated contrast imaging procedures in patients with eGFR 30 60 mL/min/1.73m2

Within these constraints, metformin should be considered the first-line treatment for all patients with type 2 diabetes, including those with CKD.

  • If  cost of medication  was the reason for  non-compliance  replace Liraglutide with insulin therapy basal insulin with lowest acquisition cost. If replaced with insulin there must be strict diet control and exercise that helps the patient to loose weight.
  • If Furosemide was given as an adjunctive drug advice the patient to observe closely for hypoglycemia, if needed switch to other anti-hypertensive agent.

Go to  Case Study 2.  | For more  CASE STUDIES .

Case Study-5 Answers

1.   what is the treatment goal and strategy for this case.

Ans :  Goals of therapy of CSA are :

  • Amelioration of anginal symptoms and improved angina-free exertion capability
  • Prevention or reduction of subsequent acute MI, UA, or ISD and there by increasing quality of life.

1.  Vasodilating – Beta-blockers as initial therapy:

Vaso-dilating beta blockers like Carvedilol and nebivolol can be used as first line agents, as they don’t have negative metabolic effects as compared to non-vasodilating beta-blockers like metoprolol.

Usually non-vasodilating  beta-blockers like metoprolol, propranolol etc., have high risk for new onset diabetes or masking of hypoglycemia.

{ Usually beta-blockers are preferred as initial therapy in the absence of any contraindications or chances for severe side effects.

In case of contraindications to beta blockers → use long acting calcium channel blockers  as initial therapy. }

2.  Lipid lowering agents  :

Lipid lowering agents like atorvastatin can be used as prophylactic therapy to prevent or minimizing risk of CAD.

3. Anti-platelet agents :

Adding  Aspirin  to the treatment shows a good evidence of Preventing MI and Death and Reducing Symptoms.

2. Suggest the best follow-up for this case.

Ans:     Monitoring of Symptoms and Antianginal Therapy:

During the first year of therapy, evaluations every 4 to 6 months are recommended. After the first year of therapy, annual evaluations are recommended if the patient is stable and reliable enough to call or make an appointment when anginal symptoms become worse or other symptoms occur.

  • Has the patient decreased the level of physical activity since the last visit?
  • Have the patient’s anginal symptoms increased in frequency and become more severe since the last visit? If the symptoms have worsened or the patient has decreased physical activity to avoid precipitating angina, then he or she should be evaluated and treated according to either the unstable angina or chronic stable angina guidelines, as appropriate.
  • How well is the patient tolerating therapy?
  • How successful has the patient been in reducing modifiable risk factors and improving knowledge about ischemic heart disease?
  • Has the patient developed any new comorbid illnesses or has the severity or treatment of known comorbid illnesses worsened the patient’s angina.

3.What are the conditions which worsens the symptoms of angina (in general)?

  • Tobacco use,
  • high blood pressure,
  • high cholesterol,

Go to  Case 5  | Explore more  Case Studies  | Go to  guidelines

Case Study-6 Answers

1 .  what information and counselling points would you include.

Ans:  Mr SW  has already been taking steps  to address his unhealthy lifestyle and reduce his cardiovascular risk.

  • Furthermore, he can be counselled for risk factors for cardiovascular disease that are commonly distinguished as modifiable and non-modifiable. Modifiable risk factors are those that can be controlled, treated or modified. These include  smoking, diet  (especially cholesterol and lipids),  weight and obesity, physical exercise, blood sugar levels and hypertension .

Non-modifiable risk factors are those that cannot be changed, such as age, gender, ethnicity and family history.

  • However, Mr SW should be counselled about how primary and secondary prevention helps in delaying the progression of disease by early detection of any further onset of disease .
  • He should be given proper information about the side effects (like muscle spasm, headache, flushing, breathlessness on physical work) & use, storage of medications; and how the benefits of treatment are much more as compared to side effects. So that, Mr SW will understand the rational use of medication and develop better medication adherence.
  • Also, if Mr SW founds any side effects that he is concerned about ( like impotence in previous ), rather quitting the drug by himself, consult his physician for the same. Drugs can be changed/further treatment option will be suggested based on the situation, the physician finds.

2. How is stable Angina managed ?                          Go to CASE STUDY-6

Ans: Managing stable Angina:

Treatments can be divided into those that reduce the risk of future cardiovascular events, thereby reducing mortality ( secondary prevention ) and those that prevent symptoms ( which can be further subdivided into short- and   long-term relief ).

For Secondary Prevention:

Aspirin  75 mg daily   (reduction in non-fatal MI & vascular events)

Simvastatin  40 mg daily  (starting dose)

  • Alternative / lower dose can be used if contraindication / interactions found.
  • If total cholesterol <4 mmol/L or LDL <2 mmol/L not achieved at initial dose, titration of simvastatin or alternative should be used.

For Short-Acting relief:

GTN      sublingually  (rapid onset of action: within 1-5 min)                                            Go to CASE STUDY-6

  • Minimizes the discomfort
  • Choice of dosage form should be discussed with patient
  • Pt should be counselled for side effects, use & storage
  • Can cause difficulties in Pt with significant arthritis or reduced hand dexterity

For Long-Acting symptom control:

  • 1 st  line treatment should be with either beta-blocker or CCBs.
  • If symptoms are not controlled, the next step is to swap or add the other (usually avoiding the combination of  beta – blocker & verapamil ).
  • Long-acting nitrate
  • Response to treatment should be reviewed 2-4 weeks after starting or changing drug treatment.               Go to CASE STUDY-6

3. What options are there, if Mr. SW experiences further symptoms despite the use of amlodipine?

Ans :   If further symptoms are experienced, i.e., symptoms not adequately

controlled with medication, the relative merits and risks of coronary artery bypass grafts (CABG) versus percutaneous coronary intervention to alleviate symptoms should be discussed. A multidisciplinary discussion should take place when the coronary artery disease is more complex.

REFERENCES:

1. European Society of Cardiology (2006). Guidelines on the management of stable Angina Pectoris.

2. national institute for health and clinical excellence (2011). management of stable angina. clinical guideline 126. london.

Go to CASE STUDY-6

Explore more  Case Study  |  Guidelines  |  Home

Case Study-7 Answers

Ans. the fourth universal definition of mi (udmi) was released in 2018. according to the udmi, mi is defined as myocardial injury in the clinical setting of myocardial ischemia. there are two components:.

  • Myocardial injury which is defined as raise of troponins above the 99th percentile
  • Clinical Setting of myocardial ischemia-symptoms and signs of myocardial ischemia, ECG changes with new ischemic changes or pathological Q waves, imaging evidence like loss of viability, regional wall motion abnormality (RWMA), thrombus in angiography, evidence of thrombus in autopsy, sudden cardiac death.

2. What are the classical ECG criteria for diagnosing STEMI ? Go to Case Study-7  

Ans.  There should be two contiguous leads with ST elevation (measured at J point).

>2.5 mm for men <40 years

>2 mm for men >40 years           in leads V2, V3

>1.5 mm for women.

And/or       >1 mm in other leads in absence of LVH, LBBB.

For posterior MI, in leads v7–v9, 0.5 mm is itself enough to diagnose posterior wall MI in inferior MI.

3. What are the different types of MI ? Go to Case Study-7  

Ans.  UDMI classifies MI in five types:

a) Type 1 MI due to thrombosis of an atherosclerotic plaque

b) Type 2 MI due to myocardial oxygen supply demand imbalance in the context of another acute illness.

c) Type 3 MI presenting as sudden death

d) Type 4 post-percutaneous coronary intervention (PCI)

e) Type 5 postcoronary artery bypass grafting.

4. Justify the treatment given to this patient.  Go to Case Study-7  

  • Aspirin  – ISIS 2 was the landmark trial in 1988 after which aspirin was considered as mainstay in ACS patients
  • DAPT  – benefit of DAPT in ACS setting and post-PCI setting were shown in CURE and PCI CURE trials, where they used Clopidogrel as the p2y12 inhibitor of choice. There was a significant reduction in composite primary endpoint of cardiovascular (CVS) mortality, non-fatal MI and stroke (9.3vs. 11.4%) with number needed to treat (NNT=48)

Ticagrelor was compared to clopidogrel in PLATO trial. Addition of ticagrelor to aspirin was able to further reduce composite primary endpoint of cardiovascular mortality, non-fatal MI, and stroke from 11.7% to 9.8% (P < 0.001) with an insignificant increase in major bleeding (11.6% vs. 11.2%)

  • In this case BMI of the patient was found to be 29.2 which is overweight. This puts patient at high risk for increased LDL, Cardiovascular comorbidities etc.

Statins  have a huge body of evidence both for early initiation and intensive treatment strategy mainly from meta-analysis data by cholesterol treatment trialist. Most of the trials utilized atorvastatin in the STEMI setting. With high intensity statin defined as the atorvastatin 40-80 mg with an ability to decrease LDL cholesterol by >50

  • Beta blockers  found to have robust benefit in people with MI and post MI reduced LVEF.
  • ACE inhibitor  trials are the SAVE, AIRE and TRACE trials which demonstrated mortality benefit of starting ACE inhibitors within first 24 h. AIRE trial used the drug ramipril.

#   Discontinue  Lasix + Spironolactone  :

Early initiation of Lasix and Spironolactone was not was not found to beneficial in Acute Myocardial infarction. ( As MI is one of the main cause for reduced ejection fraction or systolic heart failure consider using Mineralocorticoid receptor antagonists when reduced ejection fraction is less than 40% ).

(These can be used if the patient is having post-MI heart failure or LV dysfunction or Ejection fraction < 40% or diabetic with symptoms of heart failure.)

5. Determine the duration of DAPT In this patient ? Go to Case Study-7  

Ans.  In this case patient has had an acute coronary event (STEMI). He is a 56-year-old male, with only smoking as risk factor. He does not have any features of high bleeding risk. He is a candidate with a Low Bleeding risk with a moderate ischemic risk. PRECISE DAPT SCORE in this patient is <25. Hence, strategy for DAPT duration will be till 12 months with Aspirin 75 mg dose and ticagrelor 90 mg BD dose.  Reassess the patient at the end of 12 months , with DAPT score. If the patient is found to have high ischemic risk features, prefer continuing the patient on ticagrelor 60 mg BD till 3 years’ duration if cost is not an issue for greater benefit .

( Refer : high bleeding risk criteria and Precise DAPT scoring)

Go to Case Study-7  

Explore more  Clinical Case Studies  |  Guidelines  |  Home

Case Study-8 Answers

1. is the diagnosis & treatment given to mr tr justified explain.  case 8  .

Ans:  The diagnosis and treatment given to patient is  appropriate .

             Justification:

Diagnosis  :  The most likely diagnosis was confirmed by angiogram, which identified thrombosis and stenosis in the coronary arteries obtuse marginal, LAD& RCA, requiring deployment of drug-eluting stents. Also, Echocardiogram shows normal LV function.

Treatment  :

  • Drug eluting stents(DES) – for stenosed coronary artery to restore blood flow.
  • Dual antiplatelet therapy (Aspirin+Clopidogrel) – required as DES is inserted, until re-endothelisation occurs. However, treatment courses of 2 nd  antiplatelet agent(clopidogrel) should be kept to a minimum(1-3 months or 1 year)
  • Beta blocker & ACEI started with minimum required dose which is required.
  • High-intensity statins are recommended after an acute coronary syndrome.
  • GTN helps in venodilatation and arterial dilatation & reduction of cardiac ischaemia.

***Major and intermediate drugs interactions are there. However, drugs dosage is already  maintained minimum as recommended , if  used together .

2. Is there a need of counselling?  Case 8  

Ans:  Approx. 50% or more cardiovascular patients do not take their medications as intended. Poor adherence can lead to increase in recurrence, hospitalization, mortality. So, counselling is required to increase positive adherence.

Pt should be informed of drugs effects and side effects and the benefits of medication adherence.

Also, he should be counselled for lifestyle modification.

The patient should be encouraged to report any adverse effects from the drugs.

**He may have to take Aspirin lifelong to prevent secondary cardiovascular events.

Case 8   |  Clinical Case Studies  |  Guidelines  |  Home

Case Study-9 Answers

  • As the patient is at High risk with known CVD, the target goal for LDL-C would be < 70 mg/dl.
  • High-intensity statins (Atorvastatin 40-80 mg , Rosuvastatin 20 – 40 mg ) would be choice of treatment in this patient.
  • If on maximal statin & LDL-C ≥70 mg/dL (≥1.8 mmol/L), add ezetimibe.
  • Using non-statin therapy like niacin would be helpful in the patients who are intolerant to statins. (prior to withdrawing statin therapy check whether the patient is having true intolerance to statin or not).

References :

  • https://www.acc.org/~/media/Non-Clinical/Files-PDFs-Excel-MS-Word-etc/Guidelines/2018/Guidelines-Made-Simple-Tool-2018-Cholesterol.pdf
  • https://www.lipid.org/sites/default/files/6-_lipid_u-_guidelines-_jones.pdf

Case 9 | Clinical Case Studies | Guidelines | Home

Case Study-10 Answers

1. what is the metabolic abnormality present  case 10.

Ans: The obvious abnormal investigation is a very high serum cholesterol with high

LDL and low HDL levels. He has many clinical features to go with the high cholesterol and premature vascular disease. The patient has familial hypercholesterolaemia . He has presented with premature coronary artery disease. His absent pedal pulses suggest peripheral vascular disease.

** The metabolic defect is a result of a reduced number of high-affinity cell-surface LDL receptors. This leads to increased LDL levels. Increased uptake of LDL by macrophage scavenger receptors leads to increased oxidized LDL, which is particularly atherogenic. Triglyceride and VLDL levels are normal or mildly elevated. HDL levels are low. The other major causes of hypercholesterolaemia are familial combined hyperlipidaemia and polygenic hypercholesterolaemia. Familial combined hyperlipidaemia differs from familial hypercholesterolaemia by patients having raised triglycerides. Patients with polygenic hypercholesterolaemia have a similar lipid profile to familial hypercholesterolaemia but they do not develop xanthomata.

2. Discuss the Patient counselling for this case?  Case 10

Ans: The patient is at extremely high risk for further vascular events and especially

occlusion of his coronary artery bypass grafts. His risk depends on the combination of his risk factors, and all of these need attentions.

He should be counselled about following points:

  • To stop smoking,
  • Reduce his alcohol intake
  • Take more exercise and
  • Eat a strict low-cholesterol diet.
  • He should be suggested that diet alone will not control this level of cholesterol. So, a proper adherence with the pharmacological treatment with a statin or combined treatment for this level of hyperlipidaemia, is needed.
  • Importance of periodic lab check-ups.
  • His children should have their lipid profile measured so that they can be treated to prevent premature coronary artery disease.

Case 10 | Clinical Case Studies | Guidelines | Home  

Case Study-11 Answers

1. what would be the possible differential diagnosis  case 11  .

Ans:  The differential diagnosis of a pediatric patient who presents with a  narrow complex tachycardia  includes atrioventricular reentrant tachycardia  (AVRT ), AV nodal reentrant tachycardia ( AVNRT ),  sinus tachycardia ,  atrial flutter ,  atrial fibrillation ,  junctional ectopic tachycardia ,  atrial ectopic tachycardia , and  multifocal atrial tachycardia/chaotic atrial tachycardia .

Provisional Diagnosis: SVT ( supraventricular tachycardia)

2. What is the treatment goal and strategy for this case?   Case 11  

Ans: Treatment goals include:

  • Reducing the symptoms and stabilizing the patient,
  • Terminating the SVT and
  • Stablishing a mechanism to prevent any cardiac emergency condition.

Treatment Strategy:

If the patient is clinically stable , vagal maneuvers may be initially attempted to convert the tachycardia. Such vagal maneuvers may include bearing down (as though having a bowel movement (i.e., Valsalva maneuver)), blowing in a straw or inducing the diving reflex using an ice bag to the face for infants. Other vagal maneuvers such as eyeball pressure and unilateral carotid massage are harmful and should not be performed.

If the patient appears clinically unstable , urgent electrical cardioversion is indicated using 0.5-1 J/kg. If an intravenous line is already in place (antecubital preferred over a hand vein), an  IV bolus of 0.1-0.2 mg/kg adenosine  may be given prior to cardioversion. Adenosine causes a transient AV block and sinus bradycardia, thus interrupting the reentrant circuit involving the AV node and accessory pathway. It must be remembered that this medication is metabolized by the red blood cells and has a very short half-life (approximately 5 seconds), therefore it should be administered via bolus injection followed by an immediate bolus of saline (rapid push and flush). A 12-lead ECG should be obtained before and after conversion, if possible, and a rhythm strip should be continuously run during attempted conversion.  External pacing equipment should be available since some patients go into sinus arrest following administration of adenosine .

If adenosine initially fails to convert the SVT , but the patient is hemodynamically stable, they may be started on a medication such as  propranolol, digoxin or verapamil  (digoxin should be avoided in WPW, verapamil should be avoided in infants) followed by a repeat dose of adenosine. 

3. What medicine used to treat Supraventricular tachycardia(SVT) is contraindicated specifically in Wolff-Parkinson-White syndrome(WPW)?  Case 11  

Ans:    Digoxin .

** In the setting of WPW, digoxin can facilitate impulse conduction via accessory pathway and increase risk for ventricular arrhythmias (i.e., ventricular fibrillation).

Case 11  |  Clinical case Studies  |  Guidelines  |  Home 

Case Study-12 Answers

1. acute management of af with hemodynamic instability     case 12.

Given his presentation of AF with hypotension and pulmonary edema, considering AF with hemodynamic instability, sinus rhythm must be rapidly restored by synchronised electrical cardioversion. After achieving normal sinus rhythm, consider anti-coagulation as the patient is at risk for thromboembolism. Anticoagulation must be initiated and should be continued for at least 4 weeks.

(The decision to continue anticoagulation beyond the initial 4 weeks is based on the long-term risk for thromboembolism associated with nonvalvular AF. This risk is estimated by determining his CHA 2 DS 2 -VASc score, which is calculated by assigning points for congestive heart failure (1 point), hypertension (1 point), age (1 point for 65-74 years and 2 points for >75), diabetes (1 point), stroke, TIA, or thromboembolism (2 points), vascular disease defined as history of myocardial infarction, peripheral vascular disease, or aortic atherosclerosis (1 point), and female gender (1 point) )

2. CHA 2 DS 2 -VASc score (estimating risk of stroke & thromboembolism):    3 points       Case 12

Stroke risk was 3.2% per year in >90000 patients ( the Swedish atrial fibrillation cohort study) and 4.6%risk of stroke/TIA/systemic embolism.

  HAS-BLED score ( for estimating major bleeding risk) : 2 points

Risk was 4.1% in one validation study (Lip 2011) and 1.88 bleeds per 100 patient-years in another validation study      (Pisters 2010).Anticoagulation can be considered, however patient does have moderate risk for major bleeding (~2/100 patient-years).

Case 12 | Clinical Case Studies | Guidelines | Home

Case Study-13 Answers

1. based on current evidence, which of the following would be the most appropriate    recommendation regarding his asthma medication regimen   case study-13.

A. Maintain current medication regimen; no adjustment is indicated

B. Discontinue the inhaled corticosteroid; maintain on an inhaled beta-agonist as needed

C. Decrease the inhaled corticosteroid to 1 puff daily

D. Discontinue the inhaled corticosteroid; start a leukotriene modifier at bedtime

E. Discontinue the inhaled corticosteroid; start low-dose inhaled corticosteroid/long-acting beta-agonist, 1 inhalation at bedtime

ANS:  correct answer = option E

EXPLANATION : The goal of asthma therapy is to minimize risk and maintain asthma control with the least amount of medication (1).  In patients with mild persistent asthma, recent studies have demonstrated several options for “step-down therapy.”  The American Lung Association Asthma Clinical Research Centers network study found that patients who stepped down from twice daily low-dose fluticasone to once daily combination therapy with fluticasone/salmeterol had equivalent asthma control scores, FEV 1 , and frequency of exacerbations compared with continued therapy with twice daily fluticasone (2).  Once-daily montelukast demonstrated a slightly higher treatment failure compared with either of the regimens containing inhaled steroids.  Despite the slight increase in treatment failure with montelukast, each of the treatment groups had equivalent symptom-free days and rates of clinically significant asthma exacerbations.  Thus, while either regimen would be appropriate, stepping down to once-daily combination therapy with fluticasone/salmeterol appears to be more beneficial.

Recent studies also suggest that those with mild persistent asthma taking inhaled corticosteroids in combination with either a long-acting beta-agonist or a short-acting beta-agonist when symptomatic, had no increase in adverse outcomes compared with those taking scheduled daily inhaled doses.  Boushey et al. (3) compared patients with mild persistent asthma using twice-daily budesonide versus twice-daily zafirlukast verses placebo. All three groups used budesonide as-needed following a symptom-based action plan. The study found that in comparison with patients on a daily controller (budesonide or zafikulast), participants using only as-needed budesonide had no significant difference in morning peak expiratory flow, postbronchodilator FEV 1 , quality of life, or frequency of asthma exacerbations. Results of this study raise the possibility of treating mild persistent asthmatics with as-needed inhaled corticosteroids. More recently, Papi et al. (4) found as-needed use of an inhaler containing both beclomethasone and albuterol for symptom relief was associated with fewer exacerbations and higher morning peak flow readings than using an inhaler with albuterol alone.  The morning peak flow readings in the as-needed combination beclomethasone/albuterol group was equivalent to those taking scheduled daily doses of beclomethasone alone, or scheduled daily doses of beclomethasone/albuterol combined.  The combination of an inhaled steroid and a short-acting beta-agonist in a single inhaler is not currently available in the United States.

In the mild persistent asthmatic there is now strong evidence to support multiple treatment approaches which provide good asthma control.  Matching the drug regimen with the patient’s preferences, lifestyle, comorbidities, and financial limitations will help ensure drug adherence and maintain asthma control.

2. Which of the following should be done routinely with each follow-up visit?   Case Study-13

ANS: correct answer = option D

EXPLANATION: Assessing inhaler technique at each office visit allows the provider an opportunity to assess compliance, reinforce proper use, and identify motor or physical limitations affecting technique (8).

When studied, only approximately 25% of patients are able to properly demonstrate use of a meter dose inhaler when asked.  The remaining 75% improved with specific instruction and practice which reinforces the need to incorporate proper inhaler use during the office visit (9,10).  The use of a spacer significantly improves accuracy and dose delivery, particularly in patients with poor coordination skills (9,10).

Assessing patient adherence is best approached with a non-judgmental attitude.  Adherence to inhaled corticosteroids is estimated at < 50% (11).  Causes of nonadherence are multifactorial but may be improved by providing asthma education, encouraging self management through use of an asthma action plan, and facilitating open communication (11).  Financial barriers often transcend all other efforts to improve adherence and must be taken into account when prescribing asthma therapy (11).

Methacholine challenge testing is useful to demonstrate airway hyperresponsiveness in those with normal spirometry and a suspicion of asthma, but is not recommended as a serial procedure.  Biomarkers for inflammation such as eosinophils or nitric oxide are being investigated in clinical trials but currently have no indication in routine asthma care (1).  Peak flow monitoring is useful for long-term home assessment of asthma control and medication response, but is not indicated for regular office assessment or diagnostic purposes (1).

3. What findings would suggest that the patient requires a step-up in asthma medication?   Case Study-13

A. Two or more night time awakenings per month due to Asthma.

B. Two or more interruptions in daytime activities per month due to asthma

ANS:  correct answer = option A

EXPLANATION: Inadequate asthma control and a need for step-up therapy is based on two or more daytime symptoms per week, two or more nighttime symptoms per month, interference with activities of daily living, use of short-acting beta-agonist > 2 days/week (excluding use for prevention of exercise-induced bronchospasm),or peak flow or FEV 1  <80% predicted/personal best (1). 

Asthma symptoms should be assessed at each office visit to determine asthma control.  Validated self-assessment tools such as the Asthma Control Test (ACT), Asthma Therapy Assessment Questionnaire (ATAQ), or Asthma Control Questionnaire (ACQ) can facilitate consistent measurement and documentation of asthma symptoms during office visits (1, 8).  All asthmatics are at risk for a severe asthma attack regardless of their asthma classification; therefore, providers are encouraged to teach patients to recognize symptoms of inadequate asthma control and provide them with specific instructions for adjusting their medications or seeking medical care (1)

4. The patient was provided with an asthma action plan to follow at home. Which component of the asthma action plan is considered the most critical element for improving asthma outcomes?   Case Study-13

ANS:  correct answer = option D

EXPLANATION : All of the elements are important components of an asthma action plan. However, Gibson and Powell (5) found a 40% reduction in hospital admissions and a 20% reduction in emergency room visits when the plan contained personalized instructions regarding the medications to add, criteria for adding the medication, duration of use, and when to seek medical help when patients are symptomatic. An asthma action plan serves as a patient guide for early recognition and treatment of an exacerbation. Treatment guidelines may be based on symptoms, peak flow readings, or both. When peak flow readings are used, personal-best readings were consistently associated with improved health outcomes compared with percentage-predicted readings (5).

5. How often is spirometry testing recommended if the previous readings are normal and the patient’s asthma is well controlled?   Case Study-13

EXPLANATION: Spirometry is a simple test that can be performed in-office and can be used to assist the provider in determining the degree of airway obstruction (1, 6).  There are no widely accepted data correlating frequency of spirometry with clinical outcomes in asthmatics, thus one must rely on expert opinion and individual patient needs.  Spirometry is recommended during the initial evaluation after treatment is initiated and the patient’s symptoms have stabilized during periods of progressive or prolonged loss of asthma control and at least every 1-2 years (1).

When spirometry is used to diagnose or confirm asthma, testing must include pre- and post-bronchodilator readings (1).  A change in FEV 1  of >200 ml  and  ≥ 12% from the baseline measure following the administration of a short-acting bronchodilator is indicative of significant airway reversibility which has been shown to correlate with airway inflammation (7).  

The Expert Panel (1) classifies asthma severity by FEV 1 , FEV 1 /FVC, short-acting beta-agonist use, or frequency of asthma symptoms.   Parameters are measured at baseline with asthma severity determined by the worse parameter, e.g., daily symptoms with normal FEV 1  is classified as moderate persistent asthma.  Correct identification of asthma severity guides the provider in choosing the appropriate type and amount of therapy

REFERENCES :

  • Expert Panel Report 3 (EPR 3). Guidelines for the Diagnosis and Management of Asthma. Bethesda, Md: National Institutes of Health; 2007. NIH Publication No. 08-4051.
  • The American Lung Association Asthma Clinical Research Centers. Randomized comparison of strategies for reducing treatment in mild persistent asthma. N Engl J Med 2007;356:2027-2039.
  • Boushey HA, Sorkness CA, King TS, et al. Daily versus as-needed corticosteroids for mild persistent asthma. N Engl J Med 2005;352:1519-1528.
  • Papi A, Giorgio GW, Maestrelli P, et al. Rescue use of beclomethasone and albuterol in a single inhaler for mild asthma. N Engl J Med 2007;356:2040-2052.
  • Gibson PG, Powell H. Written action plans for asthma: an evidence-based review of the key components. Thorax 2007;59:94-99.
  • Miller MR, Hankinson J, Brusasco V, et al. Series ATS/ERS Task Force: Standardization of lung function testing. Eur Respir J 2005;26:319-338.
  • Pellegrino R, Viegi G, Brusasco V, et al. Interpretative strategies for lung function tests. N Engl J Med 2005;26:948-968.
  • Global Initiative for Asthma. Pocket guide for asthma management and prevention. Bethesda, Md: National Institutes of Health; 2006.
  • Giraud V, Roche N. Misuse of corticosteroid metered-dose inhaler is associated with decreased asthma stability. Eur Respir J 2002;19(2):246-251.
  • Johnson DH, Robart P. Inhaler technique of outpatients in the home. Respir Care 2000;45(10):1182-1187.
  • Elliott RA. Poor adherence to anti-inflammatory medication in asthma reasons, challenges, and strategies for improved disease management. Dis Manage Health Outcomes 2006;14(4):223-233.Top of Form

Case Study-14 Answers

1. what emergency measures are indicated   case study- 14.

Ans: In this case patient demonstrates the destabilizing effects of a respiratory infection on asthma, and her mother’s comments demonstrate the common (and dangerous) phobia about overuse of bronchodilator or steroid inhalers. The patient has signs of imminent respiratory failure, including her refusal to lie down, her fear and her tachycardia, which cannot be attributed to her minimal treatment with albuterol.

#Critically important immediate steps are

  • to administer high flow oxygen and to start albuterol by nebulisation.
  • Adding ipratropium to the nebulized solution is recommended.
  • A corticosteroid (0.5 – 1.0 mg/kg of methylprednisolone) should be administered intravenously.

#Alert the intensive care unit, because a patient with severe bronchospasm who tires can slip into respiratory failure quickly, and intubation can be difficult.

2. How should her long-term management be altered?    Case Study- 14

Ans: Presuming this patient recovers, she needs adjustments to her therapy before discharge. The strongest predictor of severe attacks of asthma is their occurrence in the past. Thus, this patient therapy needs to be stepped up to a higher level, like a high dose inhaled corticosteroid in combination with a long acting beta agonist. Both the patient and her  parents need instructions on the importance of regular adherence to therapy, with reassurance that it can be stepped down to a lower dose of inhaled corticosteroid( although still in combination with long acting beta agonist) once her condition stabilizes.

They also need instruction on an action plan for managing severe symptoms. This can be as simple as advising that if the patient has a severe, frightening attack, she can take up to four puffs of albuterol every 15 minutes, but if the first treatment doesn’t bring significant relief, she should take next four puffs while on her way to an emergency department or urgent care clinic.

She should also be given a prescription for prednisone, with instructions to take 40-60 mg orally for severe attacks, but not to wait for it to take effect if she remains severely short of breath even after albuterol inhalations.

# Asthma is a chronic disease, and a good care requires close follow up and creation of a provider-patient partnership for optimal management. If she has had several previous exacerbations, she should be a candidate considered for monoclonal anti-IgE antibody therapy with omalizumab, which effectively reduces the rate of asthma exacerbations _ even those associated with viral respiratory infection.

Reference : Text book Bertram G. katzung Basic and clinical pharmacology 14 th edition page number 365.

Go to Case Study- 14

For more Case Studies | Guidelines

All subject, PharmD books in pdf free download

Case Study-15 Answers

1. what clinical features and risk factors of copd does glenda exhibit  what grade of severity does glenda’s copd fall into    case study -15.

Ans: Glenda presents with a clinical scenario suggestive of COPD based on her age, smoking history, weight, frequent chest infections and gradual worsening of respiratory symptoms – breathlessness on exertion, a reduction in exercise tolerance, chronic cough and regular sputum production. Working in a fabric factory is also one of the identified occupational risk factors for COPD. Glenda is considered to have moderate COPD based on her spirometry results and breathlessness score.

(refer NICE classification of severity of airflow obstruction)

2. What initial treatment would you recommend for glenda?     Case Study -15

Ans: Start a short acting beta agonist such as salbutamol or a short acting muscarinic antagonist, such as ipratropium bromide, to alleviate symptoms as required. Short acting bronchodilators should be used as needed; their onset of action ranges from approximately five minutes (beta2 agonists) to 30 minutes (muscarinic antagonists) and effects lasts for between 3 and 6 hours.

Refer: NICE clinical guideline for COPD

3. Glenda continues to report that her breathlessness is getting worse. Her medical research council dyspnea score is now four and in the last few days she has been producing more sputum than usual. Her sputum has turned a yellow green colour. What does these changes indicate & what treatment would you recommend?     Case Study -15

Ans: Glenda’s symptoms suggest that she is experiencing exacerbations of COPD. She must be prescribed an antibiotic for 5 days at a therapeutic dose with Oral prednisolone 30 mg every morning for 7-14 days.

(Systemic corticosteroids are beneficial in the management of exacerbations of COPD. They shorten recovery time and improving lung function (FEV10 and hypoxemia (PaO2).  Antibiotics should be used to treat exacerbations of COPD associated with a history of more purulent sputum. Initial empirical treatment should be amoxicillin, doxycycline or clarithromycin depending on a local resistance pattern.

Go to Case Study -15

Explore more Case Studies | Guidelines

Case Study-16 Answers

1. how the sign, symptoms & pathophysiology of copd, relates to the patient   case study-16.

Ans:   Clinical signs and symptoms of COPD, the pathophysiology and how this relates to the patient.

2. Comment on the current drug therapy and describe the role of O2 in this patient.     Case Study-16

Role of oxygen:

As this patient has a PaO2 of less than 7.3 kPa and oxygen saturation of arterial blood of less than 90%, she is eligible for Long-Term Oxygen Therapy (LTOT). She will need to use oxygen at least 15 hours a day and needs to be counselled on the importance of smoking cessation. Ambulatory and short-burst oxygen therapy should also be considered, as per NICE guidelines (NICE, 2004).

3. What are the social issues in treating this patient at home?    Case Study-16

Ans: Patients may be anxious and refuse such support. The cultural and social setting of the patient needs to be taken into account. As patients lose their mobility and increase dependence on others for help with day-to-day living, anxiety increases. Patients can become hesitant to seek help because of the perception that their condition was self-inflicted. Poor populations tend to have a higher risk of developing COPD, other factors include poor nutrition, crowding, exposure to pollutants, poor access to healthcare and early respiratory infections. Some evidence suggests women are more susceptible to COPD development than men. A multidisciplinary team should be involved in the support of the patient at home. Additional use can be made of nebulisers, compressors, oxygen, visiting respiratory nurses and increased social service input. The patient remains under care of the hospital but the GP is made aware of the extra support. Health status is better in home-treated patients. COPD is linked with other co-morbid conditions. Patients are more likely to have ischaemic heart disease, pneumonia and diabetes, making treatment more complicated and requiring a holistic approach to care.

This patient demonstrates five co-morbidities. These in turn impact on the medication load she has to cope with, so concordance is important.

Book: Pharmacy case studies by Rebekah     Raymond

Go to Case Study-16

Explore more Case Studie s | Guidelines

PharmD 1st year, 2nd year, 3rd year, 4th year books pdf free download

Case Study-17 Answers

1. Why should women of childbearing age be offered advice about pregnancy?  Case Study-17

Ans:  Mrs Jaya or other women like her should have been offered preconception advice prior to becoming pregnant because glucose control needs to be optimal to reduce the risks of miscarriage, congenital malformation, stillbirth and neonatal death associated with diabetes in early pregnancy. Preconception advice should also include information for the patient on how diabetes affects pregnancy and how pregnancy affects diabetes, what dietary supplements to take and advice on diabetes-related medicines that are unsafe to take during pregnancy.

2.  Was she taking appropriate dietary supplements prior to conception?     Case Study-17

Ans:  Mrs Jaya was taking the appropriate dietary supplement; however, the recommended dose for women with diabetes is 5 mg daily, rather than 400 μcg daily. The 5-mg strength tablets are available on prescription.

3.  What advice should she be given with respect to her regular medication?     Case Study-17

Ans:  Mrs Jaya should have been advised to stop her ramipril and simvastatin since both have been associated with an increased risk of birth defects.

Case Study - 18 Answers

You can get Case -18th answers with explanation by this link.

Go back to Case Study-18 

Case Study - 19 Answers

Answers :        (CORRECTED)

1.     What is the likely diagnosis? Main differential diagnosis?          Case Study-19

Ans:   Diagnosis: Hypothyroidism.

The differential diagnosis is extensive.

However, Main Differential Diagnosis : Depression and Hypothyroidism

** Fatigue is a very common symptom of both physical and mental illness. The differential diagnosis is extensive and includes cancer, depression, anaemia, renal failure and endocrine diseases. He has a past history of depression, but currently has no obvious triggers for a further episode of depression. He is not waking early in the morning or having difficulty getting to sleep, which are common biological symptoms of severe depression.

There are a number of clues in this case to the diagnosis of hypothyroidism . Insidious onset of fatigue, difficulty concentrating, increased somnolence, constipation and weight gain are features of hypothyroidism. As in this case there may be a family or past medical history of other autoimmune diseases such as type 1 diabetes mellitus, vitiligo or Addison’s disease. Hypothyroidism typically presents in the fifth or sixth decade, and is about five times more common in women than men. Obstructive sleep apnoea is associated with hypothyroidism and may contribute to daytime sleepiness and fatigue. On examination the facial appearances and bradycardia are consistent with the diagnosis

2.     How would you further manage this patient?                        Case Study-19

Ans:    For the management of the patient in this case, patient should be advised for Thyroid Function Test . Level of TSH, T ₃ , T ₄ will lead to decide the accurate therapy for him.

**However, a starting dose of levothyroxine 50-70 μ g/day will be sufficient. Clinical benefits begin in 3-5 days.

Response is measured clinically and biochemically by the return of TSH to the normal range.

*Elderly patients or those with coronary heart disease should be started cautiously on T4 because of the risk of precipitating myocardial ischaemia.

The most common cause of hypothyroidism is autoimmune thyroiditis and the patient should have thyroid autoantibodies assayed.

Patients should be advised to avoid or do heavy physical labor with caution (hypotonia may be there sometimes).

·         No specific diets are required for hypothyroidism . WHO recommends a daily dietary iodine intake of 150 μ g for adults.

CORRECTION :                    Case Study-19

1.      Starting Dose of Levothyroxine should be 1.6 – 1.8 mcg/kg/day of lean body mass.

2.       If started with levothyroxine, then, soyabean, walnuts, dietary fiber, calcium fortified juices should be avoided, if possible. When levothyroxine is given during continuous enteral nutrition for more than 7 days, the tube should be interrupted for at least one hour before and one hour after the dose of levothyroxine.  

Ref-2: Recommendations for the use of medications with continuous enteral nutrition , “ American journal of health-system pharmacy: AJHP: official journal of the American Society of Health-System Pharmacists ”  

Case Study - 20 Answers

1.     What is the likely diagnosis in this case?  & Are the Lab finding s clinically justified?    Case Study- 20

Ans: Diagnosis: Hypothyroidism with Anemia.

           !  Ix Hashimoto Thyroiditis .

Yes , the lab findings are clinically justified.

·         High TSH & low levels of T4 suggestive of Hypothyroidism.

·         Stool occult blood positive: Reason to justify blood loss, decrease in levels of RBC, Haemglobin.

·         High RDW with normal MCV indication for anemia further evalualtion needed for iron/B12/FA deficiency/anemia of chronic disease.

·         Decrease in reticulocyte:(borderline to decrease) indication for iron deficiency/anemia of chronic disease.

·         USG: indicates Thyroditis further lab values needs for antibody testing(thyroxine peroxidase).

2.     What are the Pharmacist Intervention points in this case?          Case Study- 20

Ans:   Overdose: According to TSH value,  Tab Thyronorm 75 mcg should have been  started instead of Tab Thyronorm 25mg on the basis of wt. of patient (1.6-1.8mcg/kg/day).

Inappropriate Dose:  There is no electrolytic imbalance seen therefore, isotonic (0.90% NS) should have been administered.

Untreated Indication: No medication for headache has been prescribed (antipyretic SOS can be prescribed)

Drug- Drug Interaction: Moderate Interaction

( Pantoprazole : Iron supplements ), ( Levothyroxine: Pantoprazole ), [ Levothyroxin: Iron Supplements ( Theoritical )]

Drug-Food Interaction: Is seen with soyabean products

3.     Patient counselling regarding drug & disease?          Case Study- 20

Ans:    Disease: It is a disorder in which under activity of Thyroid glands occur(in local language).

Signs & Symptoms : related to slow metabolism (functioning) of the body .

·         Fatigueness, dry skin, constipation, feeling cold, slow heart rate, weight gain

     *Avoid soyabean & its products; cruciferous vegetables.

·         Don’t chew the tablet

·         Don’t split the tablet

·         Tightly place the cap of container after use

·         In the case of missed dose, Don’t take double dose; If it is time for next dose, take the single dose only.

·         Don’t take the dose immediately, when you recall anytime, after you missed the dose

·         If case, vomiting occurs immediately after taking the dose, consume it after sometime. If another episode occurs, consult your physician. Don’t stop taking the medication [this further may lead to untreated condition, resulting in myxedema (emergency condition)]

ü The Improvement of symptoms may not be evident for several weeks after the starting of the therapy. So, she should be counselled for proper medication adherence.

Drugs Counselling Points:

·         Take tab Thyronorm on an empty stomach (30min to 1 hour prior to breakfast) once daily.

·         Take tab Thyronorm 4 hours apart from Pantoprazole and Febac XT, Fdson MP Forte.

·         Take tab Fdson MP Forte once daily after lunch

·         Take one tab Febac XT after breakfast and another after dinner.

·         For Tab Fdson MP Forte & Febac XT, you may observe metallic taste in the mouth. Therefore, don’t stop taking the medication.

·         Take Tab Pantoprazole once a day 1 hour before lunch.

4.     Write the Generic Names of above advised drugs.      Case Study- 20

Ans:                                                                                         Generic

Pantoprazole                                                  Pantoprazole

            Optineuron                                                   Vit. B-Complex

            Fdson MP Forte                                          Folic Acid 5mg + Methylcobalamin

        1500mcg + Pyridoxine 20mg

            Febac XT                                                       Ferrous Ascorbat 100mg + Folic Acid

          1.5mg + Zinc Sulphate 22.5 mg

            Thyronorm                                                  Levothyroxine Sodium

Case Study - 21 Answers

Q1 . What is the likely diagnosis ?          Case Study-21

Ans: Diagnosis : Grave disease in pregnancy with impending thyroid storm and

        IUFD at 23-24 weeks gestation.

Q2. Was the Treatment given to the patient justified according to clinical condition?            Case Study-21

Ans: The dosing and the regimen advised to the patient is according to the prescribed guidelines and in the limit according to the lab reports.

***Methimazole 30mg BD,                     recommended 60-80mg/day (impending

thyroid storm cond. )

*** Lugol solution can be given 5 drops 2 times a day, but should not be more than 1 week.

*** The combination of propranolol 40 mg every 6 hours with iodide usually results in clinical improvement within 2 to 7 days.

Q3. Widely used anti-thyroid drugs in pregnant women with hyperthyroidism?              Case Study-21

Ans: Propylthiouracil (PTU) and methimazole

Q4. What should be the Patient Counselling points?                  Case Study-21

·         Pt should be instructed to report any new symptoms occurring.

·         She should be counselled about the necessity of medication adherence.

·         Next optimal time to conceive is once a euthyroid state is reached, should consult the practitioner for the same.

·         Pre- pregnancy counselling for all patients with hyperthyroidism or a history of hyperthyroidism is imperative and use of contraception until the disease is controlled.

·         Prior to conception, ablative therapy (radioiodine or surgery) or medical therapy may be offered when the thyroid gland is overactive.

·         May experience the previous symptoms of hyperthyroidism along with new symptoms, consult the practitioner

·         Avoid caffeine; stress reduction therapy should be suggested to relieve the symptoms of anxiety, nervousness, poor conc. May occur due to the miscarriage.

·         It is normal to women to experience some Vaginal bleeding (light, menstrual-like bleeding) for several weeks after an abortion.

·         Some pain is normal after an abortion, as the uterus is contracting.

·         In this case, she should be specially suggested of the recommended interval to next interval to next pregnancy is at least 6 months (checking the hyperthyroid condition)

·         She should be counselled with care and one to one interactions and the family members should be counselled to maintain a better social environment for her.

ADDITIONAL BASIC POINTS TO CONSIDER IN COUNSELLING:

·         If case, vomiting occurs immediately after taking the dose, consume it after sometime. If another episode occurs, consult your physician. Don’t stop taking the medication [this further may lead to untreated condition , resulting in myxedema (emergency condition)]

Case Study - 22 Answers

1.     Are there any drug-drug interactions?          Case Study – 22

Ans:    Major:

Diltiazem + Propranolol ::  Increases the toxicity of other by unspecified mechanism. Can increase the risk of bradycardia.

**In this case, alternative drugs also have the major/serious interaction (alternatively: closely monitor)

            Moderate:

            Propranolol + Furosemide :: Decreases serum potassium.

            Propranolol + Diltiazem  ::  Both increase anti-hypertensive channel blocking.

            ** Monitor Closely                              *Minor interactions are also there.

2.     Is the dosing of medication for hyperthyroidism, according to standard treatment guidelines?          Case Study – 22

Ans:    Dose of carbimazole in hyperthyroidism; depends on the

FT4 levels. If the levels are raised 2 times higher than the upper limit of normal values then the dose is : 10-20 mg. So, correct in this case.

Given in split doses as duration of action is less than 24 hours, Twice a day is acceptable.

If HR >90 BPM Beta adrenergic blocker are considered, Propranolol dose :10-40 mg 3-4 times / day

For atrial fibrillation: diltiazem IV 15-20 mg is considered.

Maintenance dose is: 5-10 mg Carbimazole

3.     What should be Patient counselling in this case, regarding drug & disease?                Case Study – 22

Ans:  Drugs:

·         If case, vomiting occurs immediately after taking the dose, consume it after sometime. If another episode occurs, consult your physician. Don’t stop taking the medication.

            Disease:

Hyperthyroidism

·         It is a disorder in which thyroid hormones level are increased. So, they may cause some alarming symptoms related to increase in the metabolism of the body such as wt loss, over active bowel movement, heat intolerance, tremors, palpitations, nervousness, fatigue , weakness.

·         Patient should be advised to follow medication adherence, Otherwise may increase the risk of cardiovascular problems.

·         Pt should be informed that improvement in the symptoms may be seen in 3-4 weeks.

·         A sightly elevated liver function test are commonly seen in some hyperthyroidism pts.

·         Patient should be informed of side effects of ATDs and the necessity of informing the physician promptly if they should develop pruritic rash, jaundice, acolic stools or dark urine, arthralgias, abdominal pain, nausea, fatigue, fever, or pharyngitis. Preferably, this information should be in writing.

·         Before starting ATDs and at each subsequent visit, the patient should be alerted to stop the medication immediately and call their physician if there are symptoms suggestive of agranulocytosis or hepatic injury.

·         Patient should be informed about the condition like, Sometimes, after apparently successful treatment, the condition may return, and further treatment may be needed.

Hemorrhoids/ Anal fissure

·         May be advised to sit over Hot water bag/ tub/brick not more than 15 min.

Note: As much as one can tolerate

·          Keep a check on your regular bowel movement, avoid any straining.

·          Pt should be advised to Eat less, oily less spicy easily digested food items

Case Study -23 Answers

1.     What should be the further investigation ?                          Case Study- 23

Ans:     Females with primary ovarian insufficiency-related estrogen deficiency are

at risk of osteopenia, osteoporosis, and fracture, especially if hypoestrogenism occurs early in life and before accrual of peak bone mass. Therefore, Dual-energy X-ray absorptiometry has been recommended for the evaluation of bone mineral density in women diagnosed with primary ovarian insufficiency.

Flow-mediated brachial artery diameter for endothelial dysfunction as there are cardiovascular risks .

[** The woman has evidence of hypogonadotrophic hypogonadism- she has low oestradiol

levels associated with low gonadotrophin stimulation from the anterior pituitary. This

may be due to various pituitary or hypothalamic causes, but in this case clearly relates to

anorexia nervosa and possibly excessive exercise . The raised prolactin is consistent with

stress and does not need to be investigated further. At a BMI below 18 kg/m2, menstru-

ation tends to cease, returning once the BMI increases again.]

-Furthermore, CBC, Liver & renal function test should be

  monitored.

-Cognition, Mood, and Psychosocial Functioning

-Vasomotor Symptoms and Quality of Life

-Also include USG abdomen if needed!

2.     How, this patient would be managed ?                Case Study- 23

Ans: The combined oral contraceptive pill should be prescribed, which will prevent osteoporosis and bring on periods. However, this anorexia and primary ovarian insufficiency condition can be refractory to treatment, albeit pharmacologically induced.

Recommended dose:

* One of the estrogen options to be combined with one of the progestogen options.

Also, vitamin supplementation and calcium should be started (if evident)

3.     What Patient counselling points should be included in this case?      Case Study- 23

Ans: Following Counselling points will help in effective recovering:

·         Encouraging the woman to eat a more normal diet and to avoid exercising is the ideal management.

·         Explanation that her periods will return if she increases her BMI may possibly encourage her to put on weight.

·         Proper counselling should also be given to her parents, to help in stress reduction.

·         She should be advised to consult dietician & nutritionist also, to maintain regular food habits with minimal or moderate physical exercise.

·         Combined oral contraceptive pills are to be taken daily at approximately the same time each day.

·         Avoid taking them greater than 24 hours apart as this could affect efficacy. 

·         When you initiate the contraceptive pills you are not protected from pregnancy prevention in the first 7 days and an alternative method of birth control is recommended during this time period (not required in this case)

·         If she, miss a tablet, just take the missed tablet as soon as she remembers and the next tablet at the usual time (taking 2 tablets in 1 day). 

·         If she, miss 2 tablets in a row in the first or second week, then, take 2 tablets the day she remembers and 2 tablets the next day, then resume 1 per day.

·         The most common adverse effect of combined oral contraceptive pills is break through bleeding. 

·         She may feel of nausea, headaches, abdominal cramping, breast tenderness, and an increase in vaginal discharge or decreased libido. 

·         Nausea can be avoided by taking the medication at night before sleep.

Ref: 1. https://www.acog.org/en/Clinical/Clinical%20Guidance/Committee%20Opinion/Articles/2017/05/Hormone%20Therapy%20in%20Primary%20Ovarian%20Insufficiency

2. Oral contraceptive pills, NCBI, National Library of Medicine,USA

Case Study -24 Answers

1.      What should be the further investigation ?                  Case Study – 24

Ans: Further recommended investigations are :

Lipid panel , 2 hr OGTT, questionnaire for Depression, USG Abdomen (transabdominal)

2.      How, this patient would be managed ?                      Case Study – 24

Ans : For acne: First line: hormonal contraception, and topical cream (benzoyl

         peroxide/ tretinoin/adapalene/ antibiotic cream like clindamycin.

            ** depending upon the staging nd grading of acne

Obesity: First line is Lifestyle Modifications

Insulin resistance: First line METFORMIN (Dose 1500-2250 mg twice daily )

Hirsutism: First line:  Hormonal contraception with or without anti androgen therapy

Menstrual irregularities : Clomiphene 50-100mg / day

3.      What Patient counselling points should be included in this case?                      Case Study – 24

·         Encouraging the woman to eat a more normal diet and to avoid leg exercising is the ideal management.

·         Explanation that how increased BMI can also affect the normal menstrual cycle.

·           For Acne: Wash the face twice daily with medicated face wash and can spill water during day time.

·         Adapalene is UV, light sensitive, to be used at night. And wash your face before stepping out. It should be instructed that cold temperatures or wind may also increase skin irritation during drug therapy.

·         Benzoyl peroxide : May cause dry skin and peeling, so avoid using higher concentration. Should be advised to use a test dose for over-the-counter (OTC) products due to potential hypersensitivity reactions. Apply small amount to skin for 3 days and if no discomfort occurs, use product as directed.

·         Clomiphene citrate: Drug may cause decreased visual acuity. Patient should be instructed to report visual symptoms, such as blurred vision. Advise patient to take drug exactly as ordered, as administration is timing-sensitive.

Case Study - 25 Answers

1.     What should be the further investigation for this patient ?              Case Study – 25

Ans : The following further investigation should be suggested for her:

                                    i.             Thyroid profile

                                  ii.              HbA1C test

                               iii.              Diurnal epinephrine test

2.     Give the provisional diagnosis for this case.                                             Case Study – 25

     Ans : Provisional Diagnosis: Menopausal syndrome

3.     How, this patient would be managed ?                                Case Study – 25

Ans : Following management plan is recommended for the patient:

Estradiol low dose transdermal patch is to be started with min possible dose

of : 0.025 mg with size 5 cm ² patch. (Duration : 3 weeks of therapy + 1 week off)

An oral progestin is also added for only 10-14 days every month this will promote sleep.

( Dose : 10mg dydrogesterone OR Micronised progesterone capsule 100mg Frequency : at night 1 tab)

For glycine and glutamate:  glutathione 20mg is recommended Or NAC(n-Acetyl cysteine) if needed

For PEA:  vit B6 is advised daily, at lunch time.. 1tab, MVBC most preferably

4.     What Patient counselling points should be included in this case?                            Case Study – 25

Ans:    About disease:

  •     Give information to menopausal women and their family members (as appropriate) that includes:
  •     That a change in their menstrual cycle they may experience a variety of symptoms associated with menopause, including:

·         Vasomotor symptoms (for example, hot flushes and sweats)

·         Musculoskeletal symptoms (for example, joint and muscle pain)

·         Effects on mood (for example, low mood)

·         Urogenital symptoms (for example, vaginal dryness)

·         Sexual difficulties (for example, low sexual desire)

·         Benefits and risks of treatments for menopausal symptoms

·         Long-term health implications of menopause

·          Hormone replacement therapy (HRT) aids for Psychological symptoms , Consider HRT to alleviate low mood that arises as a result of the menopause

·         Non-pharmaceutical, for example cognitive behavioral therapy (CBT),  Consider CBT to alleviate low mood or anxiety that arise as a result of the menopause

About drugs: (general points)

Transdermal patch:

Application : It is to be applied to non hairy skin, below waist on upper quadrant of abdomen.

Replace the patch in every 3-4 days using a different site.

·         Gradually reducing HRT may limit recurrence of symptoms in the short term

·         Gradually reducing or immediately stopping HRT makes no difference to their symptoms in the longer term

**Long-term benefits and risks of hormone replacement therapy–

·          Venous thromboembolism:The risk of venous thromboembolism (VTE) is increased by oral HRT compared with baseline population risk

·          The risk of VTE associated with HRT is greater for oral than transdermal preparations

·          Consider transdermal rather than oral HRT for menopausal women who are at increased risk of VTE, including those with a BMI over 30 kg/m ²

·          Consider referring menopausal women at high risk of VTE (for example, those with a strong family history of VTE or a hereditary thrombophilia) to a haematologist for assessment before considering HRT

Lifestyle changes:

·         She should be suggested to exercise regularly

·         She can take cold water bath as required.

·         She should be counselled to avoid stress and enjoy day to day activity.

·         Advise her to deep her feet in mild-salted, tolerable cold water, to ease her in hot flashes.

·         Consult your gynecologist and ask her about vaginal wash products to prevent dryness, if needed.

·         isoflavones or black cohosh may relieve vasomotor symptoms advice to consume soy products, nuts raisins, legumes, sesame, peanuts, Meat, pork, chicken, egg , banana.

Note: Ensure that menopausal women and healthcare professionals involved in their care understand that there is no clear evidence for SSRIs or SNRIs to ease low mood in menopausal women who have not been diagnosed with depression. (NICE Guidelines)

1.        Indian Menopause Society ( https://indianmenopausesociety.org/prescription-writing-module-for-hormone-therapy/ )

2.        Essentials of medical Pharmacology, Tripathi.KD, 6 th edition.

3.        https://www.acc.org/latest-in-cardiology/ten-points-to-remember/2020/02/10/12/13/hormone-therapy-for-postmenopausal-women

4.        https://dailymed.nlm.nih.gov/dailymed/lookup.cfm?setid=c714974b-766f-42f2-a846-b0c1f5a60560

5.        https://www.jmidlifehealth.org/article.asp?issn=0976-7800;year=2013;volume=4;issue=2;spage=77;epage=106;aulast=Meeta%2C

Case Study -26 Answers

1.      How should the ovarian hormones she lacks be replaced?            Case Study- 26 

Ans: The patient should be advised to start daily transdermal estradiol (100

mcg/day) along with oral natural progesterone (200mg/day) for the last 12 days of each 28 day cycle. On this regimen, her symptoms should disappear and normal monthly uterine bleeding resume.

2.      What extra measures should she take for her osteoporosis while receiving treatment?              Case Study- 26 

Ans: She should  also be advised to get adequate exercise and increase her calcium

and vitamin D intake as treatment for her osteoporosis.

Clinical evidence:

Non-pharmacological approaches–

·         A balanced diet, adequate calcium and vitamin D intake, weight-bearing exercise, maintaining a healthy body weight and cessation of smoking and moderation of alcohol intake are primary goals in reducing fracture risk.

·         Calcium is essential for bone health, and there is evidence that calcium supplementation in older women reduces the risk of fracture.

·         The recommended nutritional intake (RNI) for calcium is 1000mg/day, and for vitamin D 800 IU/day 

**Higher calcium intake during growth and early adulthood is associated with higher peak bone mass.

**Based on recent concerns of a potential association between calcium supplement use and increased risk of myocardial infarction, calcium supplements should not be prescribed when dietary calcium intake is adequate (1000 – 1200 mg/day).

Case Study - 27 Answers

1.      What all investigations are needed for the further management?

·         Her 10-year risk of fracture FRAX is the risk assessment tools validated for use

·         She has been taking multivitamin with calcium so

·         Consider assessing serum 25-OH-D

·         dual-energy X-ray absorptiometry (DXA) and repeat DXA every 1 to 2 years until findings are stable

·         Additional tests for Clinical Identification of Vertebral Fracture :

2.      How this patient would be managed ?

·         Counsel patients to maintain adequate dietary intake

of calcium, to a total intake (including diet plus supplement, if needed) of 1,200 mg/day for women age ≥50 years

·         Pharmacologic therapy is strongly recommended for Patients with osteopenia or low bone mass and a history Of fragility fracture of the hip or spine

·          Pharmacologic therapy is strongly recommended for Patients with a T-score of −2.5 or lower in the spine, femoral neck, total hip, or 1/3 radius

Calcium: 1200mg/day is recommended so as per the patients need either 500mg twice daily OR 1000mg once daily is given with food items rich in calcium.. 

Vit D3: If serum25[OH]D deficiency is seen then according to individualize patient therapy is given as per this case: vit D3 5000IU / day for 8-12 weeks is recommended…. With the maintenance therapy of 1000-2000 IU/day

3.      How will you do the Patient counselling  in this case?

·         Counsel patients to maintain adequate dietary intake of calcium rich products.

·         Counsel patients to avoid or stop smoking.

·          Counsel patients to maintain an active lifestyle, including weight-bearing, balance, and resistance exercises

·          Provide counseling on reducing risk of falls, particularly among the elderly.

·         Daily sun bath is advised.

·          Consider referral for physical therapy, which may reduce discomfort, prevent falls, and improve quality of life.

·          avoiding use of tobacco And excessive use of alcohol;

·         This “bone healthy” lifestyle is Important for everyone, not only patients with osteopenia And osteoporosis.

·         Weight-bearing exercise includes walking, jogging, Tai Chi, stair climbing, and dancing, among other activities.

·         Muscle-strengthening exercise includes weight training and other resistive exercises.

·         Before initiating an exercise program in an individual with osteoporosis, a clinician’s evaluation is recommended.

·         Physical therapy plays an important role in the effort to mitigate sarcopenia and reduce risk of falls.

·         Measures for Prevention of Falls

·         Anchor rugs

·         Minimize clutter

·         Remove loose wires

·         Use nonskid mats

·         Install handrails in bathrooms, halls, and long stairways

·         Light hallways, stairwells, and entrances

·         Encourage patient to wear sturdy, low-heeled shoes

4.      Mention the indications for BMD testing.

·         All women 65 years of age or older

·         All postmenopausal women

·         With a history of fracture(s) without major trauma

·         With osteopenia identified radiographically

·         Starting or taking long-term systemic glucocorticoid therapy (≥3 months)

·         Other perimenopausal or postmenopausal women with risk factors for Osteoporosis if willing to consider pharmacologic interventions

·         Low body weight (<127 lb or body mass index <20 kg/m2)

·         Long-term systemic glucocorticoid therapy (≥3 months)

·         Family history of osteoporotic fracture

·         Early menopause

·         Current smoking

·         Excessive consumption of alcohol

·         Secondary osteoporosis

Case Study-28 Answers

1. what is the significance of t- score            case study – 28.

• The T-score on your bone density report shows how much your bone mass differs from the bone mass of an average healthy adult.

• DEXA accomplishes with only one-tenth of the radiation exposure of a standard chest x-ray and is considered the gold standard for osteoporosis screening

• Standard X-rays may show weakened bones. But at the point when bone weakness can be seen on standard X-rays, it may be too far advanced to treat. Bone densitometry testing can find decreasing bone density and strength at a much earlier stage when treatment can be beneficial

• The World Health Organization has established the following classification system for bone density:

1. If your T-score is –1 or greater: your bone density is considered normal.

2. If your T-score is between –1 and –2.5: you have low bone density, known as osteopenia, but not osteoporosis.

3. If your T-score is –2.5 or less: you have osteoporosis, even if you haven’t yet broken a bone.

2. How this patient would be managed ?                Case Study – 28

• A daily calcium intake of 1000 mg for men under 70 years is advised(only of dietary calcium is not sufficient enough). So 500 mg twice daily once on morning and other at night is ideal.

• Vit D starting from 800 IU/day (once daily OR 400 IU twice daily)

3. How will you do the Patient counselling in this case?       Case Study – 28

• to not involve yourself in smoking again

• Quit drinking alcohol

• Regular exercise helps make your bones stronger. • physical activity is reviewed and prescribed by an exercise professional, such as a physiotherapist or exercise physiologist. This is because some activities like jumping, running and twisting can be hazardous to weaker bones, particularly if you have had a fracture. • Other forms of exercise such as strength training can actually benefit the bones

• Foods high in calcium include milk products, leafy green vegetables, sardines, salmon, tofu, and almonds.

• You can get enough vitamin D from being in direct sunlight for 10 to 15 minutes, two or three times weekly.

Falls prevention program –

• falls are responsible for the majority of hip and spine fractures in older people.

• A falls prevention program can provide strategies to help you prevent falls occurring

Measures for Prevention of Falls

• Anchor rugs

• Minimize clutter

• Remove loose wires

• Use nonskid mats

• Install handrails in bathrooms, halls, and long stairways

• Light hallways, stairwells, and entrances

Sources Of Calcium :

• Milk & milk products -cheese, yogurt, ice cream, buttermilk

• Turnip greens, Spinach, Kale, Radish, Okra( lady Finger),bottle gourd

• Dry Beans, such as rajma, chole, chana, lobia, other kidney beans, black-eyed peas, kidney beans, black beans

• Kamal gatta(makhana)

• Water chestnuts/cresnuts(singhada)

• Nuts like peanuts, groundnuts, walnuts, cashew nuts ,almonds , and fruit seeds.

• Seeds like melon seeds, watermelon seeds etc

• Fruits like Custard Apple, Guavas, Banana, Jackfruits, Figs, Oragnges & chiku

Sources of Vit D :

1. Sunlight

2. Food – fatty fish (examples are mackerel, salmon and tuna), egg yolks and liver.

Calcium: take the tablet with meals… and better at night…

Vit D: with meals.. Together with calcium supplements.

Case Study - 29 Answers

1.     What findings are needed for the diagnosis of POAG?            Case Study  – 29

§   enlarged cup-to-disc ratio

§   Visual acuity loss

§   Family history of glaucoma

§   Race/ Ethnicity (African American)

2.     Explain the result of the above mentioned investigations of the ocular exam?    Case Study  – 29

1.      evidence of increased IOP ,

2.      optic nerve head abnormality

3.      open anterior chamber angle,

4.      visual field deficits and no history to suggest a secondary glaucoma (glaucoma due to an identifiable cause.

·         Visual field examination shows defects that are consistent with the state of the optic nerve.

·         Measurements of the nerve fiber layer over the optic nerve can confirm an abnormally thin nerve.

3.     How this patient would be managed ?        Case Study  – 29

Ans : **Damage is permanent to eyes—it cannot be reversed. But medicine and surgery help to stop further damage.

However, Latanoprost 0.005% eyedrops                    only once at bedtime

And, If further required,then,

                   Pilocarpine 1-4% eyedrops thrice a day    OR

                   Brimonidine tartarate 0.2% twice daily     OR

                   Dorzolamide 2% eyedrops   2-3 times a day can be advised.

**If patient is not controlled on 2 topical drugs, then consider alternative treatment options with either laser trabeculoplasty or glaucoma filtering surgery.

4.     Enlist the Patient counselling points in this case?        Case Study  – 29

Patient should be counselled as-

Counselling regarding administration of drops

·         It is extremely important to use your glaucoma eye drops exactly as your ophthalmologist tells you to. That includes taking every dose, every day.

·         But remember, glaucoma eye drops won’t cure glaucoma or improve your vision. They prevent your vision from getting worse. If you don’t use them as prescribed, you could lose your vision.

How to put drops In eyes:

Follow these steps to put in your eye drops:

·         Tilt your head back and look up

·         With one hand, pull your lower eyelid down and away from your eyeball — this makes a “pocket” for the drops

·         With the other hand, hold the eye drop bottle upside down with the tip just above the pocket

·         Squeeze the prescribed number of eye drops into the pocket 

·         For at least 1 minute, close your eye and press your finger lightly on your tear duct (small hole in the inner corner of your eye) — this keeps the eye drop from draining into your nose

·         to use more than 1 type of eye drop, like different drops for different eye conditions, wait at least 5 minutes between each type.

Follow these tips to protect your eyes from infection:

·         Wash your hands with soap and water before you use eye drops

·         Don’t touch the tip of the eye drop bottle with your hands

·         Don’t let the tip of the eye drop bottle touch your eye or eyelid

Storage of drops:

·         All eye drops have an expiration date, which refers to the shelf life of a drop which has not been opened.

·         opened Eye Drops can lose potency and even become contaminated.

·         Most eye drops are stored in a cool dry place and should not be used longer than one month after the bottle is opened, unless otherwise stated on the label.

·         Pt should be advised to mention the date of opening of drops on the label itself.

General points:

·         Do not drive or operate machinery if your glaucoma eye drops make you feel tired or drowsy.

·         Blurry vision, stinging, and redness may improve with time. But if the side effects still bother you, call your ophthalmologist.

·         Never suddenly quit taking your medicine unless your doctor tells you to.

Case Study -30 Answers

1. what is the diagnosis in this case          case study – 30.

Ans • Suspected acute angle closure glaucoma R eye

         • Narrow angle L eye

2. What is the mechanism of angle closure in an episode of acute angle closure glaucoma resulting from a pupillary block?

• The apposition of the pupil border against the lens obstructs aqueous humor flow through the pupil and creates a pressure gradient with increased pressure behind the iris. This moves the iris forward with subsequent apposition of the peripheral iris with the trabecular meshwork. • Decreased drainage by the trabecular meshwork causes increased pressure in the anterior chamber and pushes the iris against the lens. • A portion of vitreous humor moves anteriorly and around the lens, blocking the trabecular meshwork. • Increased aqueous humor drainage through the trabecular meshwork causes a decreased pressure in the anterior chamber, causing a pressure gradient that presses the iris forward and blocks the angle.

3. Mention in brief the surgical options for the disease?          Case Study – 30

Ans: laser iridotomy Is recommended as the first line treatment for all patients Laser peripheral iridotomy/ Surgical iridectomy Preferably, a laser peripheral iridotomy (LPI) is done to alleviate pupillary block. It allows the aqueous to bypass the pupil, providing an alternative route for outflow from posterior to anterior chambers of the eye. Surgical iridectomy may rarely be needed in case of failures of laser iridotomy. Laser peripheral iridotomy: Technique: The role and limitations and possible complications of laser iridotomy are explained to the patient. To reduce the risk of post laser IOP spike and inflammation, apraclonidine 1% or brimonidine 0.15/ 0.2% can be used either before or after the procedure. Alternatively, oral/ topical carbonic anhydrase inhibitors or topical glycerine (in case of corneal epithelial edema secondary to raised IOP) can be used in selected patients.          • It is preferable to reduce IOP to a safe level prior to the procedure. To reduce the risk of bleeding, selected patients on oral anticoagulants for systemic diseases should be counseled and may be asked      to stop their anticoagulants for a few days prior to the procedure.         • Usually iridotomy is recommended between 11-1 o’ clock beneath theeyelids avoiding the 12 o’ clock position. However, others prefer 3 and 9 o’ clock positions.         • PI is avoided at lid margins to reduce symptoms of glare formed by tear meniscus. Successful penetration is seen with a gush of pigments in anterior chamber with a visible deepening of anterior chamber. A minimum opening of 150-200 microns is aimed to ensure patency.        • Surgery is usually considered in case of failure of medical/ laser .        • Management for IOP control or progression of glaucoma despite maximum medical Management.        • Trabeculectomy alone or combined with cataract surgery.

Case Study - 31 Answers

Questions/Answers:

1.      What are the patient counselling points in these case?          Case Study-31

With one hand, pull your lower eyelid down and away from your eyeball — this makes a “pocket” for the drops

To use more than 1 type of eye drop, like different drops for different eye conditions, wait at least 5 minutes between each type.

Follow these tips to protect your eyes from infection :

·         Opened Eye Drops can lose potency and even become contaminated.

·         Wash your face 3-4 times per day with plain water

·         Soak your eyes with cotton filled of water if having any discomfort , it will ease in your painful eyes.

·         Don’t touch your eyes with bare hands

·         If accidently touching the infected eye then wash hands .. OR least possibly don’t touch the eyes which is non infected.

·         Avoid any extra strain on eyes with limited activity of gadgets

·         Do not drive or operate machinery if your  eye drops make you feel tired or drowsy.

2.      What is not a typical exam finding of conjunctivitis?                Case Study-31

·         Eyelid erythema

·         Red conjunctiva

·         Subepithelial corneal infiltrates

·         Anterior chamber cell

·         Mucous in the canthus

Case Study - 32 Answers

1. what are the possible differential diagnosis in this case      case study – 32.

Ans: Differential Diagnosis • Viral Conjunctivitis • Hyperacute Bacterial Conjunctivitis • Chlamydial Conjunctivitis • Allergic Conjunctivitis • Superficial Keratitis • Blepharitis • Episcleritis • Scleritis • Acute Angle-closure Glaucoma • Acute Anterior Uveitis

2. What is the role of Ofloxacin in this Patient’s case?

Ans: Ofloxacin was prescribed to replace tobramycin due to suspecting toxic reaction to Tobramycin. Patient was asked to instil one drop of ofloxacin into the left Eye q2h on the day, to reduce the dosage to qid the next day if the condition Improves, the right eye is to be treated with ofloxacin qid. A review appointment was Scheduled in one week time.

3. Why Tobramycin wasn’t readvised after the PCR result, is there any evidence suggestive for the change?   

Case study – 32.

Ans:        1. the patient had an adverse Reaction to the aminoglycoside, this was evident when she noted that the Discontinuation of tobramycin had reduced ocular hyperaemia, but resulted an Increase in ocular discharge.      2. PCR was performed to rule possible viral and Chlamydia infection. This was deemed necessary as her condition was unresponsive To prescribed therapy.      3. The ophthalmologist had a strong suspicion that the condition Was caused by a gram-negative bacteria due to the nature of ocular discharge, Therefore prescribed ofloxacin as it has a relatively strong antibacterial activity Against gram-negative organisms.

Case Study - 33 Answers

1.What should Leanne’s GP consider as a possible diagnosis?       Case Study- 33

Ans: Following points should be considered for the diagnostic purpose-

·         Take a detailed history from Leanne and her parents and explore the ‘trances’ because they have experienced and witnessed them. This should determine whether an epileptic seizure is likely to have occurred.

·         Diagnosis should not be based on the presence or absence of single features.

·         Consider a history of absence seizures. Leanne is the right age

and gender for this relatively common childhood epilepsy syndrome.

·         The frequent occurrence of the daydreams and the fact that they interrupt her activities are suspicious features of childhood-onset absence epilepsy.

·         The positive family history is supportive but not diagnostic of this epilepsy syndrome. Childhood-onset absence epilepsy is classified as an idiopathic (presumed genetic) generalised epilepsy.

·         Confirm the diagnosis in the surgery. Children with typical absence seizures will often experience one of their absences during hyperventilation (over-breathing). However, hyperventilation usually has to be performed well and for at least 3 minutes to induce an absence.

·         Refer Leanne to a general paediatrician with an interest in epilepsy or a paediatric neurologist to establish the diagnosis.

Ans: First, information should be given to Leanne and her parents about the reasons For further tests, and they should be carried out in a child-centred environment.

·         An electroencephalogram (EEG) should be arranged and Leanne should have This test soon after it has been requested. Because the paediatrician or Paediatric neurologist suspects that her seizures are epileptic in origin, the EEG Should be performed to support a diagnosis of epilepsy.

·         It should not be used in Isolation to make a diagnosis of epilepsy.

·         The healthcare professionals carrying Out the EEG should encourage Leanne to hyperventilate, because this is one of The provocation techniques always undertaken during an EEG.

·         The EEG is abnormal, as in the vast majority of children with childhood-onset Absence epilepsy. It ‘captures’ an absence seizure, particularly during Hyperventilation.

3. What should the discussion around medication include, and what medications may be Prescribed?   Case Study- 33

·         The discussion on anti-epileptic medication should include the different . Medications that are available, and specifically ethosuximide, sodium valproate And lamotrigine, the evidence base for using these medications.

·         Their Common and potentially unwanted side effects. Discussion should also include The likely outcome or prognosis of the epilepsy and specifically that it will go into Spontaneous remission (that is, it will ‘go away’).

·         The family should also be Referred to a paediatric epilepsy nurse who can provide information and Guidance on lifestyle and other non-medical issues.

·         Ethosuximide or sodium valproate should be offered as a first-line treatment.

Issue Cover

  • Previous Article
  • Next Article

Case Presentation

Suggested readings, case study: a patient with type 2 diabetes working with an advanced practice pharmacist to address interacting comorbidities.

  • Split-Screen
  • Article contents
  • Figures & tables
  • Supplementary Data
  • Peer Review
  • Open the PDF for in another window
  • Cite Icon Cite
  • Get Permissions

Peggy Yarborough; Case Study: A Patient With Type 2 Diabetes Working With an Advanced Practice Pharmacist to Address Interacting Comorbidities. Diabetes Spectr 1 January 2003; 16 (1): 41–48. https://doi.org/10.2337/diaspect.16.1.41

Download citation file:

  • Ris (Zotero)
  • Reference Manager

Advanced practice pharmacists in the field of diabetes work collaboratively with patients’ medical providers, often in primary care settings or in close proximity to the providers’ practices. They help to integrate the pharmaceutical, medical, education/ counseling, and direct patient care activities necessary to meet patients’ individual self-management and diabetes care needs.

Patient education and self-management behavioral change are underpinnings of pharmaceutical care, and not only as they directly relate to the use of medications. Pharmacists, especially those who are certified diabetes educators (CDEs), frequently provide diabetes patients with education not only on medications, but also on the overall disease state, nutrition, physical activity, decision-making skills, psychosocial adaptation, complication prevention, goal setting, barrier resolution, and cost issues.

In addition to these substantial education responsibilities, advanced practice pharmacists who are Board Certified–Advanced Diabetes Managers (BC-ADMs) play an expanded role that encompasses disease state management. This includes performing clinical assessments and limited physical examinations; recognizing the need for additional care; making referrals as needed; ordering and interpreting specific laboratory tests; integrating their pharmacy patient care plans into patients’ total medical care plans; and entering notes on patient charts or carrying out other forms of written communication with patients’ medical care providers. Depending on state regulations and physician-based protocols, some advanced practice pharmacists can prescribe and adjust medications independently or after consultation with prescribing clinicians.

The clinical activities of BC-ADM pharmacists are not carried out independent of referring, collaborative practitioners. Rather, they are complementary to and serve to enhance the diagnostic, complex physical assessment, and management skills of medical providers.

The following case study illustrates the pharmacotherapeutic challenges of diabetes with other comorbidities, which can lead to potential drug-drug and drug-disease interactions. Although it does not offer detailed solutions to such problems, this case does describe the process of patient care and problem resolution as approached by advanced practice pharmacists.

B.L. is a 58-year-old white woman who has been referred to the pharmacist clinician for pharmacotherapy assessment and diabetes management. Her multiple medical conditions include type 2 diabetes diagnosed in 1995, hypertension, hyperlipidemia, asthma, coronary artery disease, persistent peripheral edema, and longstanding musculoskeletal pain secondary to a motor vehicle accident. Her medical history includes atrial fibrillation with cardioversion, anemia, knee replacement, and multiple emergency room (ER) admissions for asthma.

B.L.’s diabetes is currently being treated with a premixed preparation of 75% insulin lispro protamine suspension with 25% insulin lispro preparation (Humalog 75/25), 33 units before breakfast and 23 units before supper. She says she occasionally “takes a little more” insulin when she notes high blood glucose readings, but she has not been instructed on the use of an insulin adjustment algorithm.

Her other routine medications include the fluticasone metered dose inhaler (Flovent MDI), two puffs twice a day; salmeterol MDI (Serevent MDI), two puffs twice a day; naproxen (Naprosyn), 375 mg twice a day; enteric-coated aspirin, 325 mg daily; rosiglitazone (Avandia), 4 mg daily; furosemide (Lasix), 80 mg every morning; diltiazem (Cardizem CD), 180 mg daily (per cardiologist consult); lanoxin (Digoxin), 0.25 mg daily (per cardiologist consult); potassium chloride, 20 mEq daily; and fluvastatin (Lescol), 20 mg at bedtime. Medications she has been prescribed to take “as needed” include sublingual nitroglycerin for chest pain (has not been needed in the past month); furosemide, additional 40 mg later in the day if needed for swelling (on most days the additional dose is needed); and albuterol MDI (Proventil, Ventolin), two to four puffs every 4–6 hours for shortness of breath. She denies use of nicotine, alcohol, or recreational drugs; has no known drug allergies; and is up to date on her immunizations.

B.L.’s chief complaint now is increasing exacerbations of asthma and the need for prednisone tapers. She reports that during her last round of prednisone therapy, her blood glucose readings increased to the range of 300–400 mg/dl despite large decreases in her carbohydrate intake. She reports that she increases the frequency of her fluticasone MDI, salmeterol MDI, and albuterol MDI to four to five times/day when she has a flare-up. However, her husband has been out of work for more than a year, and their only source of income is her Social Security check. Therefore, she has been unable to purchase the fluticasone or salmeterol and so has only been taking prednisone and albuterol for recent acute asthma exacerbations.

B.L. reports eating three meals a day with a snack between supper and bedtime. Her largest meal is supper. She states that she counts her carbohydrate servings at each meal and is “watching what she eats.” She has not been able to exercise routinely for several weeks because of bad weather and her asthma.

The memory printout from her blood glucose meter for the past 30 days shows a total of 53 tests with a mean blood glucose of 241 mg/dl (SD 74). With a premeal glucose target set at 70–140 mg/dl, there were no readings below target, 8% within target, and 91% above target. By comparison, her results from the same month 1 year ago averaged 112 mg/dl, with a high of 146 mg/dl and a low of 78 mg/dl.

Physical Exam

B.L. is well-appearing but obese and is in no acute distress. A limited physical exam reveals:

Weight: 302 lb; height 5′1″

Blood pressure: 130/78 mmHg using a large adult cuff

Pulse 88 bpm; respirations 22 per minute

Lungs: clear to auscultation bilaterally without wheezing, rales, or rhonchi

Lower extremities +1 pitting edema bilaterally; pulses good

B.L. reports that on the days her feet swell the most, she is active and in an upright position throughout the day. Swelling worsens throughout the day, but by the next morning they are “skinny again.” She states that she makes the decision to take an extra furosemide tablet if her swelling is excessive and painful around lunch time; taking the diuretic later in the day prevents her from sleeping because of nocturnal urination.

Lab Results

For the sake of brevity, only abnormal or relevant labs within the past year are listed below.

Hemoglobin A 1c (A1C) measured 6 months ago: 7.0% (normal range: <5.9%; target: <7%)

Creatinine: 0.7 mg/dl (normal range: 0.7–1.4 mg/dl)

Blood urea nitrogen: 16 mg/dl (normal range: 7–21 mg/dl)

Sodium: 140 mEq/l (normal range: 135–145 mEq/l)

Potassium: 3.4 mg/dl (normal range: 3.5–5.3 mg/dl)

Calcium: 8.2 mg/dl (normal range: 8.3–10.2 mg/dl)

Lipid panel

    • Total cholesterol: 211 mg/dl (normal range <200 mg/dl)

    • HDL cholesterol: 52 mg/dl (normal range: 35–86 mg/dl; target: >55 mg/dl, female)

    • LDL cholesterol (calculated): 128 mg/dl (normal range: <130 mg/dl; target: <100 mg/dl) Initial LDL was 164 mg/dl.

    • Triglycerides: 154 mg/dl (normal range: <150 mg/dl; target: <150 mg/dl)

Liver function panel: within normal limits

Urinary albumin: <30 μg/mg (normal range: <30 μg/mg)

Poorly controlled, severe, persistent asthma

Diabetes; control recently worsened by asthma exacerbations and treatment

Dyslipidemia, elevated LDL cholesterol despite statin therapy

Persistent lower-extremity edema despite diuretic therapy

Hypokalemia, most likely drug-induced

Hypertension JNC-VI Risk Group C, blood pressure within target and stable

Coronary artery disease, stable

Obesity, stable

Chronic pain secondary to previous injury, stable

Status post–atrial fibrillation with cardioversion

Status post–knee replacement

Financial constraints affecting medication behaviors

Insufficient patient education regarding purposes and role of specific medications

Wellness, preventive, and routine monitoring issues: calcium/vitamin D supplement, magnesium supplement, depression screening, osteoporosis screening, dosage for daily aspirin

Strand et al. 1 proposed a systematic method for evaluation of and intervention for a patient’s pharmacotherapy, using a process called the Pharmacist’s Work-Up of Drug Therapy (PWDT). The PWDT has been modified by subsequent authors, 2 – 4 but the process remains grounded in the following five questions:

What are reasonable outcomes for this patient?

Based on current guidelines and literature, pharmacology, and pathophysiology, what therapeutic endpoints would be needed to achieve these outcomes?

Are there potential medication-related problems that prevent these endpoints from being achieved?

What patient self-care behaviors and medication changes are needed to address the medication-related problems? What patient education interventions are needed to enhance achievement of these changes?

What monitoring parameters are needed to verify achievement of goals and detect side effects and toxicity, and how often should these parameters be monitored?

Outcomes and Endpoints

Clinical outcomes are distinctly different from therapeutic or interventional endpoints. The former refers to the impact of treatment on patients’ overall medical status and quality of life and should emphasize patient-oriented evidence that matters (POEMs) rather than disease-oriented evidence (DOEs).

Therapeutic endpoints include the anticipated and desired clinical effects from drug therapy that are expected, ultimately, to achieve the desired outcome(s). As such, therapeutic endpoints are used as surrogate markers for achievement of outcomes. Commonly, more than one endpoint will be needed to achieve an outcome. For example, near-normal glycemic control and normalization of blood pressure (endpoints) would be necessary to significantly reduce the risk of end-stage renal disease (outcome).

Therapeutic endpoints should be specific, measurable, and achievable within a short period of time. Achievement of clinical outcomes usually cannot be determined except by long-term observation or retrospective analysis.

Outcomes and endpoints for any given patient should be determined collaboratively between patient and provider before selecting or initiating pharmacotherapy or nonpharmacological interventions. Taking the time to identify these components up front (and periodically revise them later on) helps ensure that subsequent medications or strategies are appropriately directed. It further ensures a common vision and commitment for ongoing patient care and self-management among the care team (including the patient), thus maximizing the potential for optimal disease control and patient satisfaction.

The outcomes and endpoints for a patient such as B.L. are numerous and obviously would not be addressed or attained in a single session. Therefore, after desired outcomes and endpoints are determined, they should be prioritized according to medical urgency and patient preference. Implementation and goal setting related to these priorities can then be undertaken, thus establishing a treatment plan for the eventual attainment of the full list.

During ongoing and follow-up visits, this care plan should be reviewed and modified as indicated by changes in patient status, preferences, and medical findings. Examples of desired outcomes and endpoints for B.L. are given in Tables 1 and 2 . For the sake of brevity, these tables are not intended to be inclusive.

Medication-Related Problems and Proposed Interventions

With agreement between patient and clinician concerning desired outcomes and endpoints, the next logical step is to evaluate whether the current treatment plan is likely to achieve those goals, or, if treatment is to be initiated, which therapies or interventions should be selected.

According to Strand et al., 1 a medication-related problem is any aspect of a patient’s drug therapy that is interfering with a desired, positive patient (therapeutic) outcome or endpoint. The PWDT proposes a systematic and comprehensive method to identify, resolve, or prevent medication-related problems based on the following major categories:

No indication for a current drug

Indication for a drug (or device or intervention) but none prescribed

Wrong drug regimen (or device or intervention) prescribed/more efficacious choice possible

Too much of the correct drug

Too little of the correct drug

Adverse drug reaction/drug allergy

Drug-drug, drug-disease, drug-food interactions

Patient not receiving a prescribed drug

Routine monitoring (labs, screenings, exams) missing

Other problems, such as potential for overlap of adverse effects

Once problems are identified, resolutions must be developed, prioritized, and implemented. Patient or caregiver input is especially helpful at this stage because the individual can describe subjective as well as objective data, expectations or concerns that may be affecting drug therapy, and deficits in drug knowledge, understanding, or administration.

Resolutions may result from numerous strategies, including dose alteration, addition or discontinuation of medication, adjunct medications, regimen adjustment, complementary therapies, instruction on medication administration or devices, disease or medication education, development of “cues” as compliance reminders (e.g., pill boxes), and identification of ways to avoid, detect, or manage side effects or toxicities. Needless to say, the involvement of patients and family or caregivers is critical for successful implementation of most resolution strategies and for optimal disease management.

Because of the extent of B.L.’s medication-related problems and potential interventions ( Tables 3 and 4 ), it was agreed to tackle first her asthma exacerbations and high blood glucose levels. To this end, B.L. was counseled about the role of maintenance asthma medications versus rescue drugs. The root of her confusion between these agents was easy to understand—because the prednisone and fluticasone were both called steroids, it seemed likely that the tablets were a cheaper and easier way to take the medicine. Likewise, since albuterol and salmeterol were both called bronchodilators, it seemed that the albuterol was the cheaper way to take the medicine.

Having grasped the concept of asthma prevention, she was willing to convert to a product combining fluticasone and salmeterol (Advair Diskus) for maintenance/prevention and to reserve the albuterol for quick relief of acute symptoms. Free samples of the new product were dispensed, and B.L. was enrolled in the manufacturer’s indigent drug program for subsequent supplies.

She was further instructed on the use of a peak flow meter and advised to monitor her readings and symptoms. At the next visit, these data will be used to determine her maximal expiratory effort (“personal best”) and to construct an asthma action plan.

B.L.’s insulin was changed to a basal-based regimen utilizing bedtime glargine (Lantus) insulin and premeal lispro (Humalog) insulin. Education was provided on this dosing concept. She and the pharmacist discussed how this regimen can give greater flexibility in dosing, especially for responding to changes in diet, exercise, and disease exacerbations or medications. She was also given an initial supplementary adjustment algorithm (sliding scale) to correct for any temporary elevation of blood glucose. She agreed to test four times daily and to record her blood glucose results, carbohydrate intake, and insulin doses. At the next visit, these data will be used to modify the adjustment algorithm and to construct a prospective algorithm for matching premeal bolus insulin to the anticipated carbohydrate intake (insulin-to-carbohydrate ratio).

The final interventions for this visit were to increase the dose of potassium chloride and change the fluvastatin to atorvastatin (Lipitor) to further reduce B.L.’s LDL cholesterol. Medication education for atorvastatin was provided, and patient questions were answered.

Other medication-related problems and interventions identified for B.L. are listed and briefly discussed in Tables 3 and 4 . For the sake of brevity, these lists are not inclusive nor are all pharmacotherapy issues discussed.

Monitoring for Effectiveness, Side Effects, and Toxicity

The last step in the PWDT process is to develop a plan to evaluate the patient’s progress in attaining desired outcomes, therapeutic endpoints, and behavior changes; to assess effectiveness of pharmacotherapy; and to identify side effects, drug interactions, or toxicity issues that need to be addressed.

The monitoring/follow-up component is the most tedious aspect of the PWDT. For each medication or intervention, key parameters must be identified as markers for effectiveness, for side effects, for drug interactions, and for toxicity. In addition, the time frame and process for assessing those parameters must be determined. Finally, the desired range for the parameter must be listed or a “decision point” must be identified to signal that additional action will be required.

It should be noted that only a limited number of parameters are selected for a given patient. For example, it is not necessary to list and monitor for every possible side effect with equal intensity and frequency. Selection of the monitoring parameters is based on the positive effects (efficacy) that are most important to the care of that patient, as well as the adverse effects (side effects, toxicity, or drug interactions) that are most important to avoid for the safety of that patient or to which that patient is most prone.

Because the monitoring component is usually extensive, examples listed for B.L. in Table 5 have been limited to three of the medication or regimen changes that were made at the first pharmacist visit: switching from fluvastatin to atorvastatin; switching from two shots of premixed 75/25 lispro to bedtime glargine with premeal lispro; and substituting the combination inhaler product for her fluticasone and salmeterol MDI prescriptions. Because atorvastatin and fluvastatin differ chemically, the monitoring parameters for this change are similar to those for initiation of a new medication. Monitoring for the new insulin regimen (basal insulin with premeal bolus) focuses primarily on glycemic control patterns and hypoglycemic episodes. Because B.L. has previously used the two ingredients of her new inhaler product (fluticasone and salmeterol) without adverse effect, monitoring of her new asthma therapy is focused on effectiveness, tolerance of inhalation of its dry powder formula, and use of the administration device.

Diabetes patients with multiple co-morbidities have concerns about all of their problems, not just the diabetes; therefore, BC-ADM pharmacists must comprehensively explore all the ramifications of comorbidities as well as patients’ feelings, expectations, and concerns for total health. B.L. is a good example of this; even though her referral was for “diabetes management,” her greatest concern at this visit was her asthma exacerbations.

As can be seen in this case, each coexisting disease or coprescribed drug has a domino effect, affecting other diseases or drugs and ultimately affecting quality of life. With input from B.L., the pharmacist clinician was able to develop a PWDT that addresses her diabetes as well as her other health care needs.

B.L. was able to leave the health center with a few achievable self-care goals and medication changes that address her acute concerns and with the knowledge and confidence that, at each subsequent visit, additional progress will be made toward her personalized health status goals.

Examples of B.L.’s Desired Outcomes

Examples of B.L.’s Desired Outcomes

Examples of B.L.’s Therapeutic Endpoints

Examples of B.L.’s Therapeutic Endpoints

Examples of B.L.’s Medication-Related Problems

Examples of B.L.’s Medication-Related Problems

Examples of B.L.’s Interventions (Prioritized and to be Implemented Accordingly)

Examples of B.L.’s Interventions (Prioritized and to be Implemented Accordingly)

Examples of B.L.’s Monitoring Plans

Examples of B.L.’s Monitoring Plans

Peggy Yarborough, PharmD, MS, BC-ADM, CDE, FAPP, FASHP, NAP, is a professor at Campbell University School of Pharmacy in Buies Creek, N.C., and a pharmacist clinician at Wilson Community Health Center in Wilson, N.C.

For information concerning POEMs and DOEs: a multitude of literature on this topic is available through Internet sources. Search for “patient oriented evidence that matters” using a medical topic browser.

Email alerts

  • Advanced Practice Care: Advanced Practice Care in Diabetes: Epilogue
  • Advanced Practice Care: Advanced Practice Care in Diabetes: Preface
  • Online ISSN 1944-7353
  • Print ISSN 1040-9165
  • Diabetes Care
  • Clinical Diabetes
  • Diabetes Spectrum
  • Standards of Medical Care in Diabetes
  • Scientific Sessions Abstracts
  • BMJ Open Diabetes Research & Care
  • ShopDiabetes.org
  • ADA Professional Books

Clinical Compendia

  • Clinical Compendia Home
  • Latest News
  • DiabetesPro SmartBrief
  • Special Collections
  • DiabetesPro®
  • Diabetes Food Hub™
  • Insulin Affordability
  • Know Diabetes By Heart™
  • About the ADA
  • Journal Policies
  • For Reviewers
  • Advertising in ADA Journals
  • Reprints and Permission for Reuse
  • Copyright Notice/Public Access Policy
  • ADA Professional Membership
  • ADA Member Directory
  • Diabetes.org
  • X (Twitter)
  • Cookie Policy
  • Accessibility
  • Terms & Conditions
  • Get Adobe Acrobat Reader
  • © Copyright American Diabetes Association

This Feature Is Available To Subscribers Only

Sign In or Create an Account

We have a new app!

Take the Access library with you wherever you go—easy access to books, videos, images, podcasts, personalized features, and more.

Download the Access App here: iOS and Android . Learn more here!

  • Remote Access
  • Save figures into PowerPoint
  • Download tables as PDFs
  • Top 125 Drug Card Case Quiz
  • Katzung Pharmacology Cases
  • Pharmacotherapy Casebook and Care Plans
  • Standardized Patient Cases
  • Pharmacogenomics: A Primer for Clinicians
  • Transitions of Care in Pharmacy Casebook
  • Pharmacy Practice and Tort Law
  • Case Files®: Pharmacology
  • G&G Pharm Cases
  • Pathophysiology Cases
  • Infectious Diseases: A Case Study Approach
  • Harrison’s Visual Case Challenge

Book Cover

Pharmacotherapy Casebook: A Patient-Focused Approach 12th Edition

Author(s): Terry L. Schwinghammer; Julia M. Koehler; Douglas Slain; Jill S. Borchert; Sharon K. Park

Principles of Patient-Focused Therapy

  • 11 Chemical Threat Agent Exposure
  • 9 Clinical Toxicology: Acetaminophen Toxicity
  • 10 Cyanide Exposure
  • 7 Geriatrics
  • 8 Palliative Care
  • 6 Pediatrics

Cardiovascular Disorders

  • 16 Acute Coronary Syndrome: ST-Elevation Myocardial Infarction
  • 20 Acutely Decompensated Heart Failure
  • 26 Atrial Fibrillation
  • 27 Cardiac Arrest
  • 23 Chronic Anticoagulation
  • 21 Deep Vein Thrombosis
  • 14 Dyslipidemia
  • 19 Heart Failure with Preserved Ejection Fraction
  • 18 Heart Failure with Reduced Ejection Fraction
  • 12 Hypertension
  • 13 Hypertensive Crisis
  • 28 Hypovolemic Shock
  • 17 Peripheral Arterial Disease
  • 22 Pulmonary Embolism
  • 15 Stable Ischemic Heart Disease
  • 25 Ventricular Arrhythmia

Respiratory Disorders

  • 29 Acute Asthma
  • 30 Chronic Asthma
  • 31 Chronic Obstructive Pulmonary Disease
  • 33 Cystic Fibrosis
  • 32 Pulmonary Artery Hypertension

Gastrointestinal Disorders

  • 48 Acute Pancreatitis
  • 45 Ascites Management in Portal Hypertension and Cirrhosis
  • 49 Chronic Pancreatitis
  • 42 Constipation
  • 38 Crohn Disease
  • 41 Diarrhea
  • 46 Esophageal Varices
  • 34 Gastroesophageal Reflux Disease
  • 47 Hepatic Encephalopathy
  • 43 Irritable Bowel Syndrome
  • 40 Nausea and Vomiting
  • 36 NSAID-Induced Ulcer Disease
  • 44 Pediatric Gastroenteritis
  • 35 Peptic Ulcer Disease
  • 37 Stress Ulcer Prophylaxis
  • 39 Ulcerative Colitis
  • 50 Viral Hepatitis A
  • 51 Viral Hepatitis B
  • 52 Viral Hepatitis C

Renal Disorders

  • 54 Acute Kidney Injury
  • 53 Drug-Induced Acute Kidney Injury
  • 58 Electrolyte Abnormalities in Chronic Kidney Disease
  • 56 End-Stage Kidney Disease
  • 59 Hypercalcemia of Malignancy
  • 60 Hypokalemia and Hypomagnesemia
  • 61 Metabolic Acidosis
  • 62 Metabolic Alkalosis
  • 55 Progressive Kidney Disease
  • 57 Syndrome of Inappropriate Antidiuretic Hormone Release

Neurologic Disorders

  • 68 Acute Management of the Traumatic Brain Injury Patient
  • 71 Acute Pain Management
  • 63 Alzheimer Disease
  • 70 Chronic Pain Management
  • 65 Complex Partial Seizures
  • 66 Generalized Tonic–Clonic Seizures
  • 73 Migraine Headache
  • 64 Multiple Sclerosis
  • 72 Pain, Agitation, and Delirium in Critical Care
  • 69 Parkinson Disease
  • 67 Status Epilepticus

Psychiatric Disorders

  • 77 Alcohol Withdrawal
  • 74 Attention-Deficit Hyperactivity Disorder
  • 81 Bipolar Disorder
  • 75 Bulimia Nervosa
  • 82 Generalized Anxiety Disorder
  • 84 Insomnia
  • 80 Major Depression
  • 78 Nicotine Dependence
  • 83 Obsessive–Compulsive Disorder
  • 76 Opioid Use Disorder
  • 79 Schizophrenia

Endocrinologic Disorders

  • 91 Addison Disease
  • 90 Cushing Syndrome
  • 92 Hyperprolactinemia
  • 88 Hyperthyroidism: Graves Disease
  • 89 Hypothyroidism
  • 85 Type 1 Diabetes Mellitus and Ketoacidosis
  • 87 Type 2 Diabetes Mellitus: Existing Disease
  • 86 Type 2 Diabetes Mellitus: New Onset

Urologic Disorders

  • 100 Benign Prostatic Hyperplasia
  • 99 Erectile Dysfunction
  • 101 Urinary Incontinence

Immunologic Disorders

  • 103 Allergic Drug Reaction
  • 104 Solid Organ Transplantation
  • 102 Systemic Lupus Erythematosus

Bone and Joint Disorders

  • 108 Gout and Hyperuricemia
  • 105 Osteoarthritis
  • 107 Osteoporosis
  • 106 Rheumatoid Arthritis

Dermatologic Disorders

  • 111 Acne Vulgaris
  • 113 Atopic Dermatitis
  • 114 Dermatologic Drug Reaction
  • 112 Psoriasis

Hematologic Disorders

  • 117 Folic Acid Deficiency
  • 115 Iron Deficiency Anemia
  • 118 Sickle Cell Anemia
  • 116 Vitamin B12 Deficiency

Infectious Diseases

  • 123 Acute Bronchitis
  • 126 Acute Pharyngitis
  • 149 Adult Immunization
  • 147 Antimicrobial Prophylaxis for Surgery
  • 120 Bacterial Meningitis
  • 143 Bacterial Vaginosis
  • 144 Candida Vaginitis
  • 133 Clostridioides Difficile Infection
  • 121 Community-Acquired Pneumonia
  • 128 Coronavirus Vaccination and Treatment
  • 142 Dermatophytosis
  • 130 Diabetic Foot Infection
  • 139 Genital Herpes, Gonococcal, and Chlamydial Infections
  • 150 HIV Infection
  • 122 Hospital-Acquired Pneumonia
  • 146 Infection in an Immunocompromised Patient
  • 131 Infective Endocarditis
  • 127 Influenza
  • 134 Intra-Abdominal Infection
  • 145 Invasive Fungal Infections
  • 135 Lower Urinary Tract Infection
  • 140 Osteomyelitis and Septic Arthritis
  • 124 Otitis Media
  • 148 Pediatric Immunization
  • 137 Pelvic Inflammatory Disease and Other Sexually Transmitted Infections
  • 136 Pyelonephritis
  • 125 Rhinosinusitis
  • 129 Skin and Soft Tissue Infection
  • 138 Syphilis
  • 132 Tuberculosis
  • 119 Using Laboratory Tests in Infectious Diseases

Oncologic Disorders

  • 158 Acute Lymphocytic Leukemia
  • 151 Breast Cancer
  • 159 Chronic Myeloid Leukemia
  • 153 Colon Cancer
  • 156 Hodgkin Lymphoma
  • 160 Kidney Cancer
  • 161 Melanoma
  • 155 Non-Hodgkin Lymphoma
  • 152 Non–Small Cell Lung Cancer
  • 157 Ovarian Cancer
  • 154 Prostate Cancer
  • 162 Stem Cell Transplantation

Nutrition and Nutritional Disorders

  • 164 Adult Enteral Nutrition
  • 165 Obesity
  • 163 Parenteral Nutrition

Gynecologic and Obstetric Disorders

  • 94 Contraception
  • 95 Emergency Contraception
  • 97 Endometriosis
  • 98 Managing Menopausal Symptoms
  • 93 Pregnancy and Lactation
  • 96 Premenstrual Dysphoric Disorder

Eyes, Ears, Nose, and Throat Disorders

  • 110 Allergic Rhinitis
  • 109 Glaucoma

Pharmacy Case Studies for Pharmacists & Medical Sciences Students

Pharmacists and healthcare practitioners are required to demonstrate knowledge and understanding of the application of therapeutics in clinical practice. Pharmacists must ensure patient safety and achieve desired health outcomes through effective decision-making. The idea of designing these case studies is to meet the needs and challenges of a modern pharmacy undergraduate curriculum. Case studies are increasingly used in pharmacy undergraduate as well as postgraduate education.

Each chapter contains five case studies, increasing in complexity from those we would expect first-year students to complete (Level 1) through to cases designed for fourth-year/pre-registration students (Level M). The chapters have been designed to follow approximately the British National Formulary chapters for ease of use. Case study scenarios include both community and hospital pharmacy situations as suited to the disease and pharmaceutical care provision.

This section is only for Bangladeshi Pharmacy/Medical Students & Professionals !

Cardiovascular case studies by Narinder Bhalla

Case study level 1 – Angina Case study level 2 – Hypertension Case study level 3 – Atrial fibrillation Case study level Ma – Heart failure Case study level Mb – Myocardial infarction

Respiratory system case studies by Soraya Dhillon and Andrzej Kostrzewski

Case study level 1 – Asthma – community Case study level 2 – Asthma – acute on chronic Case study level 3 – Chronic obstructive pulmonary disease (COPD) with co-morbidity Case study level Ma – COPD Case study level Mb – Brittle asthma

Obstetrics, gynaecology and UTI case studies by Alka Mistry

Case study level 1 – Primary dysmenorrhoea Case study level 2 – Urinary tract infections in pregnancy Case study level 3 – Pelvic inflammatory disease Case study level Ma – Endometriosis management in secondary care Case study level Mb – Management of severe pre-eclampsia/ eclampsia

Liver disease case studies by Caron Weeks and Mark Tomlin

Case study level 1 – Alcoholic cirrhosis; alcohol withdrawal Case study level 2 – Alcoholic cirrhosis; management of bleeding risk and treatment for the maintenance of alcohol abstinence Case study level 3 – Hepatic encephalopathy and ascites Case study level Ma – Pulmonary tuberculosis Case study level Mb – Liver failure  
  • Case study for pharmacist
  • Pharmacy case study

You might be interested in

pharma case studies

Liver disease case studies: Case study level 1 – Alcoholic cirrhosis; alcohol withdrawal

Cardiovascular case studies: case study level mb – myocardial infarction, cardiovascular case studies : case study level 3 – atrial fibrillation.

Nice sources

How to read all case studies?

Leave a Reply Cancel reply

Your email address will not be published.

{{#message}}{{{message}}}{{/message}}{{^message}}Your submission failed. The server responded with {{status_text}} (code {{status_code}}). Please contact the developer of this form processor to improve this message. Learn More {{/message}}

{{#message}}{{{message}}}{{/message}}{{^message}}It appears your submission was successful. Even though the server responded OK, it is possible the submission was not processed. Please contact the developer of this form processor to improve this message. Learn More {{/message}}

Submitting…

This site uses Akismet to reduce spam. Learn how your comment data is processed .

BioPharma Asia Convention Terrapinn

Press Release: 4 Symposiums at BioPharma Asia Convention 2013, Singapore

bioinformatics notes

Notes on Bioinformatics Basic

Case-based learning: constipation in adults

It is important that pharmacists understand how to appropriately and safely manage patients with constipation in light of new Medicines and Healthcare products Regulatory Agency guidance announced in August 2020.

Care logo

Shutterstock.com

Constipation is one of the most common gastrointestinal (GI) symptoms patients may experience [1] , [2] . As a symptom-based disorder that has no single definition, patients will often have their own opinion of what constipation means to them [3] . It is therefore necessary to ascertain what a ‘normal’ frequency of defecation is for each patient, as frequency can vary between three times per day to once every three days [2] , [3] .

Constipation affects 1 in 7 adults and 1 in 3 children at any given time; with this symptom being so common, it is not surprising that 66,287 patients in the UK were admitted to hospital with constipation as the main condition (equivalent to 182 people per day) in 2014–2015 [4] .

The incidence of constipation is two to three times higher in women than in men, and is more common with increasing age [5] , [6] . It affects 40% of women during pregnancy; however, this may be owing to the physiological, biochemical and dietary changes that occur during pregnancy [5] , [6] .

This article outlines the symptoms, diagnosis and management options for short-term, occasional constipation (< 5 days), including the appropriate use of stimulant laxatives in line with new safety measures announced by the Medicines and Healthcare products Regulatory Agency (MHRA) in August 2020 [7] .

During constipation, the lower GI tract becomes dysfunctional, giving rise to symptoms including:

  • Fewer bowel movements than normal;
  • Abdominal pain;
  • Straining during bowel movements [8] , [9] .

Some patients may report a sense of incomplete bowel evacuation, or excessive time spent on the toilet owing to unsuccessful defecation [3] . Patients experiencing any red flag symptoms (including rectal bleeding, mucus in stools, unexplained weight loss, sudden and severe abdominal pain, abdominal or rectal mass, fever or a persistent change in bowel habit for more than four weeks), as well as those who have a family history of colon cancer, ovarian cancer or inflammatory bowel disease, or who are taking clozapine, should be referred to urgent care immediately [10] , [11] .

Establishing the nature and texture of the stool, with reference to the Bristol stool chart (also known as the Meyers scale, see Figure), will help determine how constipated the patient may be [12] , [13] . Pain and discomfort from chronic constipation (i.e. having symptoms for more than 12 weeks in the past 6 months) may have an impact on the patient’s quality of life, particularly in older people [10] . Guidance from the Royal Pharmaceutical Society outlines how pharmacy can support these patients; however, the management of chronic constipation is outside the scope of this article [14] .

pharmacy patient case study examples

Figure: Bristol stool chart

Source: Shutterstock.com

Constipation is often multifactorial in adults and can result from systemic or neurological disorders, medications or other organic causes (see Table 1) [10] . Other contributory factors may include pain, fever, dehydration, lack of dietary fibre and fluid intake, little or no physical activity, psychological issues, toilet training in children and familial history of constipation [10] .

There are a range of risk factors for constipation, including:

  • Low fibre diet or low-calorie intake;
  • Difficulty in accessing the toilet, or changes in normal routine or lifestyle;
  • Lack of exercise or reduced mobility;
  • Limited privacy when using the toilet [10] .
  • Anxiety and/or depression; 
  • Somatization disorders;
  • Eating disorders;
  • History of sexual abuse [10] .
  • Female sex;
  • Older people;
  • Pyrexia, dehydration, immobility;
  • Sitting position on a toilet compared with the squatting position for defecation [10] .

Taking a thorough history from the patient can rule out many secondary causes of constipation [15] . An assessment of stool form (see Figure) can be used to estimate the extremes of colonic transit time, since very hard stools or loose stools correlate with slow or rapid colonic transit, respectively [16] .

Most patients will describe constipation with one or more symptoms. Symptoms commonly include hard stools, infrequent stools (typically fewer than three times per week), a sense of an incomplete bowel evacuation, the need for excessive straining or an excessive time spent on the toilet [17] . It is important to understand the patient’s perspective and how they feel about their symptoms when considering management.

Differential diagnosis

It is crucial that an assessment rules out other conditions that may warrant further investigation. For example, irritable bowel syndrome (IBS) should be excluded. IBS is usually associated with pain during defecation, change in stool frequency and/or change in stool form [9] .

In general practice, a physical abdominal or internal rectal examination may also be performed, which is often the most revealing part of the clinical evaluation [10] , [16] . When a clinician performs an abdominal or internal rectal examination, they are trying to determine if other issues (e.g. lesions, masses, excoriation, fissures or haemorrhoids) may be causing symptoms [3] .

The aim of management is to resolve symptoms and help prevent future occurrence of constipation. This can typically be achieved by non-pharmacological measures, but treatment can be recommended if these are ineffective. A recent MHRA update has outlined precautions that need to be taken when advising patients about over-the-counter measures for managing constipation (see Box) [7] , [14] , [18] , [19] .

Box: Changes to practice as a result of the Medicines and Healthcare products Regulatory Agency safety update

Stimulant laxatives have an acceptable safety profile, have been widely used for many years and are generally used responsibly. However, concerns have been raised regarding the abuse, overuse and misuse of over-the-counter (OTC) stimulant laxatives (e.g. in people with eating disorders, owing to the misconception that they can help them lose weight, and long-term use in older people) [7] , [19] . Following advice from the Commission on Human Medicines (CHM), the Medicines and Healthcare products Regulatory Agency (MHRA) introduced a package of risk minimisation measures for OTC stimulant laxatives in August 2020 to support correct use and minimise risk of misuse [7] .

The following summarises the risk minimisation measures relating to stimulant laxatives introduced by the MHRA.

General sales list stimulant laxatives

  • New posology: licensed only for people aged 18 years and over;
  • Revised indication: for the short-term relief of occasional constipation;
  • Reduced pack sizes: standard strength tablets in a pack size of 20, maximum strength tablets in a pack size of 10 and syrups in a pack size of 100ml;
  • More prominent warnings included, which state that stimulant laxatives do not aid weight loss [14] .

P stimulant laxatives

  • New posology: licensed only for people aged 12 years and over;
  • Revised indication: removal of indications not appropriate for self-care;

Updated stimulant laxative products are already available, while existing packs may continue to be available for sale until early autumn 2020. The individual summary of product characteristics should be consulted for more information [7] .

The Royal Pharmaceutical Society has produced detailed guidance for pharmacists and pharmacy teams to support the correct use of OTC stimulant laxatives, to prevent misuse in light of these changes and to assist in how to advise at-risk groups (e.g. people with a potential eating disorder, older people and children) [14] .

First-line management: non-pharmacological

In many cases, occasional constipation is a result of poor diet and lack of exercise. The National Institute for Health and Care Excellence (NICE) recommends first line that underlying causes should be managed and adult patients should be given advice on appropriate dietary and lifestyle measures (see Case study 1 ) [10] .

A diet that is high in fibre helps normalise bowel movements by increasing the weight and size of the stool, softening it and helping it to ‘bulk up’, making it easier to pass through the colon [20] . Fibre also contributes to the maintenance of a healthy gut microbiota [21] .

Second-line management: pharmacological

If non-pharmacological intervention for the treatment of short-term constipation in adult patients is ineffective, or patients do not experience the desired response, NICE recommends that oral laxatives can be offered using a stepwise approach:

1. Offer a bulk-forming laxative if the patient can drink adequate fluids;

2. If the patient cannot drink adequate fluids and/or if stools remain hard or are difficult to pass, consider adding or switching to an osmotic laxative;

3. If stools are soft but remain difficult to pass, then consider adding a stimulant laxative (see Box 1 and Table 2).

Alternatively, pharmacological management can be used based on symptoms and combination therapies considered as appropriate (see Table 2) [10] , [22] , [23] .

Children aged over 12 years who experience occasional constipation should be advised on maintaining a balanced diet, with enough fluids and behavioural interventions, in combination with macrogol as first-line treatment and stimulant laxatives as second line. Children aged under 12 years with constipation should be referred to a prescriber [14] .

When to stop treatment

Patients should be advised to gradually reduce and stop the use of laxatives once soft, formed stools are produced without straining at least three times per week. Pharmacists may want to arrange to review the patient regularly and should use clinical judgement to determine whether the condition is resolved or whether further treatment or referral is necessary. The importance of adhering to dietary and lifestyle recommendations should be reinforced to prevent reoccurrence.

Case study 1: an older patient presents with recurrent constipation

A man aged 74 years* presents to the pharmacy asking for something to help him manage his constipation. He explains that it “comes and goes” but he was hoping to “speed it along this time”.

It is important to discuss the following with the patient:

  • What are the symptoms?
  • When did the symptoms start?
  • How often has this occurred in the past?
  • Has he had a change of diet, routine/lifestyle or medicine?
  • Is there any pain when he uses the toilet?
  • Does he ever have frequent bowel movements or diarrhoea?
  • Is there any blood or mucus when he does defecate?
  • Has he tried anything before to manage his symptoms?

After the discussion, it is clear that the patient experiences these symptoms a couple of times per year — usually during the summer months when it is hotter — but other than that, he is “fairly regular”. He indicates that his diet and lifestyle are consistent and that other than some stomach pain and “struggling to go”, he has no other symptoms. The constipation started five days ago but as it usually resolves itself, he says he did not want to “bother the pharmacist” for the first few days. However, owing to the pain he has been experiencing, he decided it was best to come to the pharmacy for treatment.

The patient explains that he spends a lot of time in his garden, goes for regular long walks with his wife and rarely drinks, except for a pint of beer on a Sunday afternoon.

The patient’s medication record reveals that he uses an inhaler (ipratropium) to manage his COPD and takes 100mg sertraline daily. Both have been prescribed for the past year and he takes them as prescribed.

Since the patient has no change in lifestyle or new medication, it is likely that their advanced age, combined with dehydration caused by the summer heat, has contributed to the patient becoming constipated [10] .

Advice and recommendations

In this case it is important to explain that the symptoms are potentially a result of dehydration because of warmer weather. The pharmacist should explain that, owing to his age, he is more likely to suffer from constipation but he can help prevent and treat his current symptoms by drinking plenty of water and following the National Institute for Health and Care Excellence guidance on appropriate dietary and lifestyle measures. These include:

  • Eat plenty of high-fibre foods (30g per day; e.g. beans, vegetables, fruits, whole grain cereals and bran);
  • Eat fewer foods with low amounts of fibre (e.g. processed foods, dairy and meat products);
  • Drink plenty of fluids;
  • Avoid caffeine;
  • Reduce alcohol intake and do not consume more than 14 units per week;
  • Stay as active as possible and try to get regular exercise;
  • Do pelvic floor exercises (refer to  Bladder and bowel community );
  • Try to manage stress;
  • Do not ignore the urge to pass a stool;
  • Try to create a regular schedule for bowel movements, especially after a meal [24] .

If these measures do not help, advise the patient to return to the pharmacy to discuss other treatment options. However, caveat that if the patient has sudden or severe abdominal pain they should seek urgent care immediately.

Case study 2: a young patient with abdominal pain

A patient aged 16 years* asks to speak to the pharmacist. She would like something to help her go to the toilet as it has been four days and she is experiencing stomach pain.

It is important to discuss the following points during the consultation:

  • Can she describe the nature of the pain?
  • Is it the first time she has had constipation?
  • How does this vary from her ‘normal’?
  • What is her diet like and what are her activity levels?
  • Is she experiencing any stress?
  • Does she have any conditions or take any medicines or supplements (prescribed or bought over-the-counter)?
  • Has she tried anything to treat it?
  • Does she have any medicines in mind?

Some people, in particular young women, may intend to use laxatives to help them lose weight [7] , [19] . Owing to the potential for misuse, abuse and overuse, it is necessary to ensure that appropriate questions are asked to rule this out — for example, “Have you used this medicine before?” or “Are you experiencing other symptoms (e.g. vomiting)?”. It is important to offer opportunities for them to discuss any issues with you — for example, by asking “What other issues would you like to discuss today?”.

The patient describes the pain as cramping but explains that she does not think it is from menstruation as her period was more than a week ago. She cannot remember having constipation before and usually goes to the toilet once per day. She is not currently on any prescribed or over-the-counter medication, except moisturiser for her eczema. The patient regularly runs and has a good diet (i.e. plenty of fibre and fluids). She does not drink alcohol and only has a cup of tea in the morning. The patient explains that she is not stressed and that her routine is “the same as always”. She has not tried anything yet but says that her dad recommended senna as he had used it in the past; however, she does not have a preference.

Based on the conversation with the patient, it appears that she has uncomplicated constipation. Owing to her age and the mention of senna, it is appropriate to consider abuse; however, the description of symptoms and the lack of product preference indicates that the patient does not intend to abuse laxatives.

Recommendations

As the patient has an active lifestyle and suitable diet, it would be advisable to reinforce these practices, but also suggest that she could take a bulk-forming laxative (e.g. ispaghula husk). As the patient previously mentioned senna, explain to her that this is a stimulant laxative and that, although it can be beneficial to some patients, it is not recommended as the initial treatment for constipation.

Advise the patient that it could take up to three days for the bulk-forming laxative to have an effect. If the bulk-forming laxative fails, advise the patient that they could then try an osmotic laxative, such as macrogol and electrolytes (e.g. Movicol; Norgine Limited). Clarify the directions for use (e.g. reconstitution process) and explain that she should not take it immediately before bed and that she should avoid drinking tea and other diuretics while the symptoms are present as it may worsen them.

If the osmotic laxative fails, the patient may need to trial a stimulant laxative; however, if this is also ineffective after five days, then referral to her GP is recommended.

Case study 3: symptomatic during pregnancy

A pregnant woman aged 32 years* presents to the pharmacy explaining that she has had problems “going to the toilet” and was hoping she could purchase something that would help.

During the consultation it is important to ask the following questions:

  • How long has she had the symptoms?
  • What is regular for her?
  • What other symptoms does she have?
  • Is this the first time during her pregnancy that she has been constipated?
  • Does she take any other medicines (prescribed or over-the-counter) or have any other medical conditions?
  • Have there been any recent lifestyle changes?

The patient explains that she is usually regular but has been struggling to go, is passing fewer stools and experiencing irregular bowel movements. She explains her frustration as she has recently got over some reflux. Further questioning on her reflux reveals that she is currently using an aluminium-containing antacid to help relieve her symptoms. She explains that it is a bit painful but more frustrating. The patient adds that her diet is good, but she sometimes has cravings that result in her eating more dairy products. Owing to her pregnancy, she is avoiding coffee and alcohol, but explains she didn’t drink much of either prior to becoming pregnant. The patient is not on any prescribed medicine and, other than the antacid she is currently using, is only taking one other over-the-counter medicine: paracetamol (for back pain that has resulted from her pregnancy).

Although constipation is common during pregnancy, this tends to be related to difficulty in passing stools. However, this patient appears to be having difficulties passing stools and bowel movements appear to be less frequent than normal. The patient appears to have medicine-induced constipation; while the aluminium containing antacid may be effective at managing her reflux, it may be contributing to her constipation [10] .

Advise the patient to stop using the aluminium-based antacid and to try an alternative (e.g. a product containing a combination of calcium carbonate/sodium alginate/sodium bicarbonate). Owing to the discomfort caused and the patient’s existing, suitable diet, it would be appropriate to recommend a bulk-forming laxative (e.g. ispaghula husk). This is safe for use in pregnant and breastfeeding patients. Explain that this product can take two to three days to work and that she needs to make up the solution using sachets. Importantly, the patient needs to understand that once the sachet is made up in water, she must drink it as soon as possible as the drink can become ‘set’ and undrinkable as a result. The patient should be encouraged to also increase her fluid intake while taking the bulk-forming medicine. Explain to the patient that a common side effect is flatulence and abdominal bloating; however, with cessation of treatment upon resolution of constipation, this will dissipate.

If the patient continues to struggle with reflux and constipation, she should speak to her GP.

*All cases are fictional.

Mikin Patel is lead pharmacist, gastroenterology at Imperial College Healthcare NHS Trust

[1] Keshav S & Bailey A. The Gastrointestinal system at a Glance . 2 nd ed. Chichester: Wiley–Blackwell; 2012.

[2] Rutter P. Constipation and diarrhoea. In: Walker R & Whittlesea C (eds.)  Clinical Pharmacy and Therapeutics. 5 th ed. London: Elsevier; 2012. p. 209–221. 

[3] Lembo A & Camilleri M. Chronic Constipation. N Eng J Med  2003;349(14):1360–1368. doi: 10.1056/NEJMra020995

[4] Coloplast. The cost of constipation report. Available at: https://www.coloplast.co.uk/Global/UK/Continence/Cost_of_Constipation_Report_FINAL.pdf (accessed September 2020)

[5] Suares NC & Ford AC. Prevalence of, and risk factors for, chronic idiopathic constipation in the community: systematic review and meta-analysis. Am J Gastroenterol 2011;106(9):1582–1591. doi: 10.1038/ajg.2011.164

[6] Shafe AC, Lee S, Dalrymple DS & Whorwell PJ. The LUCK study: laxative usage in patients with GP-diagnosed constipation in the UK, within the general population and in pregnancy. An epidemiological study using the General Practice Research Database (GPRD). Therap Adv Gastroenterol 2011;4(6):343–363. doi: 10.1177/1756283X11417483

[7] Medicines and Healthcare products Regulatory Agency. Stimulant laxatives (bisacodyl, senna and sennosides, sodium picosulfate) available over-the-counter: new measures to support safe use. 2020. Available at: https://www.gov.uk/drug-safety-update/stimulant-laxatives-bisacodyl-senna-and-sennosides-sodium-picosulfate-available-over-the-counter-new-measures-to-support-safe-use (accessed September 2020)

[8] Cleveland Clinic. Gastrointestinal disorders. 2016. Available at: https://my.clevelandclinic.org/health/articles/7040-gastrointestinal-disorders (accessed September 2020)

[9] MD Calc. Rome IV Criteria – Constipation. 2020. Available at: https://www.mdcalc.com/rome-iv-diagnostic-criteria-constipation (accessed September 2020)

[10] National Institute for Health and Care Excellence. NICE Clinical Knowledge Summary. Constipation. 2019. Available at: https://cks.nice.org.uk/constipation#!prescribingInfoSub:2 (accessed September 2020)

[11] National Institute for Health and Care Excellence. NICE Clinical Knowledge Summary. Colorectal Cancer. 2017. Available at: https://cks.nice.org.uk/gastrointestinal-tract-lower-cancers-recognition-and-referral (accessed September 2020)

[12] Lewis SJ & Heaton KW. Stool form scale as a useful guide to intestinal transit time. Scand J Gastroenterol 1997;32(9):920–924.  doi: 10.3109/00365529709011203

[13] National Institute for Health and Care Excellence. Constipation in children and young people: diagnosis and management. NICE [CG99]. 2017. Available at:  https://www.nice.org.uk/guidance/cg99/resources/cg99-constipation-in-children-and-young-people-bristol-stool-chart-2 (accessed September 2020)

[14] Royal Pharmaceutical Society. Dealing with over-the-counter stimulant laxatives in community pharmacy. 2020. Available at:  https://www.rpharms.com/resources/pharmacy-guides/dealing-with-over-the-counter-stimulant-laxatives-in-community-pharmacy#Long%20term (accessed September 2020)

[15] Tack J, Müller-Lissner S, Stanghellini V et al . Diagnosis and treatment of chronic constipation–a European perspective. Neurogastroenterol Motil 2011;23(8):697–710. doi:10.1111/j.1365-2982.2011.01709.x

[16] Jamshed N, Lee Z-E & Olden KW. Diagnostic Approach to Chronic Constipation in Adults. Am Fam Physician 2011;84(3):299–306. Available at: https://www.aafp.org/afp/2011/0801/p299.html (accessed September 2020)

[17] Andrews CN & Storr M. The pathophysiology of chronic constipation. Can J Gastroenterol 2011;25 Suppl B(Suppl B):16B–21B.  PMID: 22114753

[18] Robinson J. Medicines regulator to publish review into safety of OTC laxatives. Pharm J online. 2020. Available at:  https://www.pharmaceutical-journal.com/news-and-analysis/news/medicines-regulator-to-publish-review-into-safety-of-otc-laxatives/20207696.article?firstPass=false (accessed September 2020)

[19] Medicines and Healthcare products Regulatory Agency. Public Assessment Report of over-the-counter stimulant laxatives: benefit-risk review. 2020. Available at: https://www.gov.uk/government/publications/public-assessment-report-of-over-the-counter-stimulant-laxatives-benefit-risk-review (accessed September 2020)

[20] Forootan M, Bagheri N & Darvishi M. Chronic Constipation: a literature review. Medicine 2018;97(20):1–9.  doi: 10.1097/MD.0000000000010631

[21] Shah A, Morisson M & Holtmann G. A novel treatment for patients with constipation: Dawn of a new age for translational microbiome research? Indian J Gastroenterol 2018;37(5):388–391.  doi: 10.1007/s12664-018-0912-3

[22] Electronic medicines compendium. Resolor 1mg film-coated tablets. 2019. Available at: https://www.medicines.org.uk/emc/product/584/smpc (accessed September 2020)

[23] Kumar L, Barker C & Emmanuel A. Opioid-induced constipation: pathophysiology, clinical consequences, and management. Gastroenterology Res Pract  2014;2014:141737. doi: 10.1155/2014/141737

[24] NHS Eat well. How to get more fibre into your diet. 2018. Available at: https://www.nhs.uk/live-well/eat-well/how-to-get-more-fibre-into-your-diet/ (accessed September 2020)

You might also be interested in…

The importance of diverse clinical imagery within health education.

injection being held above skin

Regulator approves first injectable treatment for prevention of HIV

Photo of Dame Andrea Leadsom, current Parliamentary Under-Secretary of State for Public Health, Start for Life and Primary Care

Andrea Leadsom: ‘Pharmacy First is such a big win’

  • Search Menu
  • Browse content in Arts and Humanities
  • Browse content in Archaeology
  • Anglo-Saxon and Medieval Archaeology
  • Archaeological Methodology and Techniques
  • Archaeology by Region
  • Archaeology of Religion
  • Archaeology of Trade and Exchange
  • Biblical Archaeology
  • Contemporary and Public Archaeology
  • Environmental Archaeology
  • Historical Archaeology
  • History and Theory of Archaeology
  • Industrial Archaeology
  • Landscape Archaeology
  • Mortuary Archaeology
  • Prehistoric Archaeology
  • Underwater Archaeology
  • Urban Archaeology
  • Zooarchaeology
  • Browse content in Architecture
  • Architectural Structure and Design
  • History of Architecture
  • Residential and Domestic Buildings
  • Theory of Architecture
  • Browse content in Art
  • Art Subjects and Themes
  • History of Art
  • Industrial and Commercial Art
  • Theory of Art
  • Biographical Studies
  • Byzantine Studies
  • Browse content in Classical Studies
  • Classical History
  • Classical Philosophy
  • Classical Mythology
  • Classical Literature
  • Classical Reception
  • Classical Art and Architecture
  • Classical Oratory and Rhetoric
  • Greek and Roman Epigraphy
  • Greek and Roman Law
  • Greek and Roman Papyrology
  • Greek and Roman Archaeology
  • Late Antiquity
  • Religion in the Ancient World
  • Digital Humanities
  • Browse content in History
  • Colonialism and Imperialism
  • Diplomatic History
  • Environmental History
  • Genealogy, Heraldry, Names, and Honours
  • Genocide and Ethnic Cleansing
  • Historical Geography
  • History by Period
  • History of Emotions
  • History of Agriculture
  • History of Education
  • History of Gender and Sexuality
  • Industrial History
  • Intellectual History
  • International History
  • Labour History
  • Legal and Constitutional History
  • Local and Family History
  • Maritime History
  • Military History
  • National Liberation and Post-Colonialism
  • Oral History
  • Political History
  • Public History
  • Regional and National History
  • Revolutions and Rebellions
  • Slavery and Abolition of Slavery
  • Social and Cultural History
  • Theory, Methods, and Historiography
  • Urban History
  • World History
  • Browse content in Language Teaching and Learning
  • Language Learning (Specific Skills)
  • Language Teaching Theory and Methods
  • Browse content in Linguistics
  • Applied Linguistics
  • Cognitive Linguistics
  • Computational Linguistics
  • Forensic Linguistics
  • Grammar, Syntax and Morphology
  • Historical and Diachronic Linguistics
  • History of English
  • Language Acquisition
  • Language Evolution
  • Language Reference
  • Language Variation
  • Language Families
  • Lexicography
  • Linguistic Anthropology
  • Linguistic Theories
  • Linguistic Typology
  • Phonetics and Phonology
  • Psycholinguistics
  • Sociolinguistics
  • Translation and Interpretation
  • Writing Systems
  • Browse content in Literature
  • Bibliography
  • Children's Literature Studies
  • Literary Studies (Asian)
  • Literary Studies (European)
  • Literary Studies (Eco-criticism)
  • Literary Studies (Romanticism)
  • Literary Studies (American)
  • Literary Studies (Modernism)
  • Literary Studies - World
  • Literary Studies (1500 to 1800)
  • Literary Studies (19th Century)
  • Literary Studies (20th Century onwards)
  • Literary Studies (African American Literature)
  • Literary Studies (British and Irish)
  • Literary Studies (Early and Medieval)
  • Literary Studies (Fiction, Novelists, and Prose Writers)
  • Literary Studies (Gender Studies)
  • Literary Studies (Graphic Novels)
  • Literary Studies (History of the Book)
  • Literary Studies (Plays and Playwrights)
  • Literary Studies (Poetry and Poets)
  • Literary Studies (Postcolonial Literature)
  • Literary Studies (Queer Studies)
  • Literary Studies (Science Fiction)
  • Literary Studies (Travel Literature)
  • Literary Studies (War Literature)
  • Literary Studies (Women's Writing)
  • Literary Theory and Cultural Studies
  • Mythology and Folklore
  • Shakespeare Studies and Criticism
  • Browse content in Media Studies
  • Browse content in Music
  • Applied Music
  • Dance and Music
  • Ethics in Music
  • Ethnomusicology
  • Gender and Sexuality in Music
  • Medicine and Music
  • Music Cultures
  • Music and Religion
  • Music and Media
  • Music and Culture
  • Music Education and Pedagogy
  • Music Theory and Analysis
  • Musical Scores, Lyrics, and Libretti
  • Musical Structures, Styles, and Techniques
  • Musicology and Music History
  • Performance Practice and Studies
  • Race and Ethnicity in Music
  • Sound Studies
  • Browse content in Performing Arts
  • Browse content in Philosophy
  • Aesthetics and Philosophy of Art
  • Epistemology
  • Feminist Philosophy
  • History of Western Philosophy
  • Metaphysics
  • Moral Philosophy
  • Non-Western Philosophy
  • Philosophy of Science
  • Philosophy of Language
  • Philosophy of Mind
  • Philosophy of Perception
  • Philosophy of Action
  • Philosophy of Law
  • Philosophy of Religion
  • Philosophy of Mathematics and Logic
  • Practical Ethics
  • Social and Political Philosophy
  • Browse content in Religion
  • Biblical Studies
  • Christianity
  • East Asian Religions
  • History of Religion
  • Judaism and Jewish Studies
  • Qumran Studies
  • Religion and Education
  • Religion and Health
  • Religion and Politics
  • Religion and Science
  • Religion and Law
  • Religion and Art, Literature, and Music
  • Religious Studies
  • Browse content in Society and Culture
  • Cookery, Food, and Drink
  • Cultural Studies
  • Customs and Traditions
  • Ethical Issues and Debates
  • Hobbies, Games, Arts and Crafts
  • Lifestyle, Home, and Garden
  • Natural world, Country Life, and Pets
  • Popular Beliefs and Controversial Knowledge
  • Sports and Outdoor Recreation
  • Technology and Society
  • Travel and Holiday
  • Visual Culture
  • Browse content in Law
  • Arbitration
  • Browse content in Company and Commercial Law
  • Commercial Law
  • Company Law
  • Browse content in Comparative Law
  • Systems of Law
  • Competition Law
  • Browse content in Constitutional and Administrative Law
  • Government Powers
  • Judicial Review
  • Local Government Law
  • Military and Defence Law
  • Parliamentary and Legislative Practice
  • Construction Law
  • Contract Law
  • Browse content in Criminal Law
  • Criminal Procedure
  • Criminal Evidence Law
  • Sentencing and Punishment
  • Employment and Labour Law
  • Environment and Energy Law
  • Browse content in Financial Law
  • Banking Law
  • Insolvency Law
  • History of Law
  • Human Rights and Immigration
  • Intellectual Property Law
  • Browse content in International Law
  • Private International Law and Conflict of Laws
  • Public International Law
  • IT and Communications Law
  • Jurisprudence and Philosophy of Law
  • Law and Politics
  • Law and Society
  • Browse content in Legal System and Practice
  • Courts and Procedure
  • Legal Skills and Practice
  • Primary Sources of Law
  • Regulation of Legal Profession
  • Medical and Healthcare Law
  • Browse content in Policing
  • Criminal Investigation and Detection
  • Police and Security Services
  • Police Procedure and Law
  • Police Regional Planning
  • Browse content in Property Law
  • Personal Property Law
  • Study and Revision
  • Terrorism and National Security Law
  • Browse content in Trusts Law
  • Wills and Probate or Succession
  • Browse content in Medicine and Health
  • Browse content in Allied Health Professions
  • Arts Therapies
  • Clinical Science
  • Dietetics and Nutrition
  • Occupational Therapy
  • Operating Department Practice
  • Physiotherapy
  • Radiography
  • Speech and Language Therapy
  • Browse content in Anaesthetics
  • General Anaesthesia
  • Neuroanaesthesia
  • Browse content in Clinical Medicine
  • Acute Medicine
  • Cardiovascular Medicine
  • Clinical Genetics
  • Clinical Pharmacology and Therapeutics
  • Dermatology
  • Endocrinology and Diabetes
  • Gastroenterology
  • Genito-urinary Medicine
  • Geriatric Medicine
  • Infectious Diseases
  • Medical Toxicology
  • Medical Oncology
  • Pain Medicine
  • Palliative Medicine
  • Rehabilitation Medicine
  • Respiratory Medicine and Pulmonology
  • Rheumatology
  • Sleep Medicine
  • Sports and Exercise Medicine
  • Clinical Neuroscience
  • Community Medical Services
  • Critical Care
  • Emergency Medicine
  • Forensic Medicine
  • Haematology
  • History of Medicine
  • Browse content in Medical Dentistry
  • Oral and Maxillofacial Surgery
  • Paediatric Dentistry
  • Restorative Dentistry and Orthodontics
  • Surgical Dentistry
  • Browse content in Medical Skills
  • Clinical Skills
  • Communication Skills
  • Nursing Skills
  • Surgical Skills
  • Medical Ethics
  • Medical Statistics and Methodology
  • Browse content in Neurology
  • Clinical Neurophysiology
  • Neuropathology
  • Nursing Studies
  • Browse content in Obstetrics and Gynaecology
  • Gynaecology
  • Occupational Medicine
  • Ophthalmology
  • Otolaryngology (ENT)
  • Browse content in Paediatrics
  • Neonatology
  • Browse content in Pathology
  • Chemical Pathology
  • Clinical Cytogenetics and Molecular Genetics
  • Histopathology
  • Medical Microbiology and Virology
  • Patient Education and Information
  • Browse content in Pharmacology
  • Psychopharmacology
  • Browse content in Popular Health
  • Caring for Others
  • Complementary and Alternative Medicine
  • Self-help and Personal Development
  • Browse content in Preclinical Medicine
  • Cell Biology
  • Molecular Biology and Genetics
  • Reproduction, Growth and Development
  • Primary Care
  • Professional Development in Medicine
  • Browse content in Psychiatry
  • Addiction Medicine
  • Child and Adolescent Psychiatry
  • Forensic Psychiatry
  • Learning Disabilities
  • Old Age Psychiatry
  • Psychotherapy
  • Browse content in Public Health and Epidemiology
  • Epidemiology
  • Public Health
  • Browse content in Radiology
  • Clinical Radiology
  • Interventional Radiology
  • Nuclear Medicine
  • Radiation Oncology
  • Reproductive Medicine
  • Browse content in Surgery
  • Cardiothoracic Surgery
  • Gastro-intestinal and Colorectal Surgery
  • General Surgery
  • Neurosurgery
  • Paediatric Surgery
  • Peri-operative Care
  • Plastic and Reconstructive Surgery
  • Surgical Oncology
  • Transplant Surgery
  • Trauma and Orthopaedic Surgery
  • Vascular Surgery
  • Browse content in Science and Mathematics
  • Browse content in Biological Sciences
  • Aquatic Biology
  • Biochemistry
  • Bioinformatics and Computational Biology
  • Developmental Biology
  • Ecology and Conservation
  • Evolutionary Biology
  • Genetics and Genomics
  • Microbiology
  • Molecular and Cell Biology
  • Natural History
  • Plant Sciences and Forestry
  • Research Methods in Life Sciences
  • Structural Biology
  • Systems Biology
  • Zoology and Animal Sciences
  • Browse content in Chemistry
  • Analytical Chemistry
  • Computational Chemistry
  • Crystallography
  • Environmental Chemistry
  • Industrial Chemistry
  • Inorganic Chemistry
  • Materials Chemistry
  • Medicinal Chemistry
  • Mineralogy and Gems
  • Organic Chemistry
  • Physical Chemistry
  • Polymer Chemistry
  • Study and Communication Skills in Chemistry
  • Theoretical Chemistry
  • Browse content in Computer Science
  • Artificial Intelligence
  • Computer Architecture and Logic Design
  • Game Studies
  • Human-Computer Interaction
  • Mathematical Theory of Computation
  • Programming Languages
  • Software Engineering
  • Systems Analysis and Design
  • Virtual Reality
  • Browse content in Computing
  • Business Applications
  • Computer Security
  • Computer Games
  • Computer Networking and Communications
  • Digital Lifestyle
  • Graphical and Digital Media Applications
  • Operating Systems
  • Browse content in Earth Sciences and Geography
  • Atmospheric Sciences
  • Environmental Geography
  • Geology and the Lithosphere
  • Maps and Map-making
  • Meteorology and Climatology
  • Oceanography and Hydrology
  • Palaeontology
  • Physical Geography and Topography
  • Regional Geography
  • Soil Science
  • Urban Geography
  • Browse content in Engineering and Technology
  • Agriculture and Farming
  • Biological Engineering
  • Civil Engineering, Surveying, and Building
  • Electronics and Communications Engineering
  • Energy Technology
  • Engineering (General)
  • Environmental Science, Engineering, and Technology
  • History of Engineering and Technology
  • Mechanical Engineering and Materials
  • Technology of Industrial Chemistry
  • Transport Technology and Trades
  • Browse content in Environmental Science
  • Applied Ecology (Environmental Science)
  • Conservation of the Environment (Environmental Science)
  • Environmental Sustainability
  • Environmentalist Thought and Ideology (Environmental Science)
  • Management of Land and Natural Resources (Environmental Science)
  • Natural Disasters (Environmental Science)
  • Nuclear Issues (Environmental Science)
  • Pollution and Threats to the Environment (Environmental Science)
  • Social Impact of Environmental Issues (Environmental Science)
  • History of Science and Technology
  • Browse content in Materials Science
  • Ceramics and Glasses
  • Composite Materials
  • Metals, Alloying, and Corrosion
  • Nanotechnology
  • Browse content in Mathematics
  • Applied Mathematics
  • Biomathematics and Statistics
  • History of Mathematics
  • Mathematical Education
  • Mathematical Finance
  • Mathematical Analysis
  • Numerical and Computational Mathematics
  • Probability and Statistics
  • Pure Mathematics
  • Browse content in Neuroscience
  • Cognition and Behavioural Neuroscience
  • Development of the Nervous System
  • Disorders of the Nervous System
  • History of Neuroscience
  • Invertebrate Neurobiology
  • Molecular and Cellular Systems
  • Neuroendocrinology and Autonomic Nervous System
  • Neuroscientific Techniques
  • Sensory and Motor Systems
  • Browse content in Physics
  • Astronomy and Astrophysics
  • Atomic, Molecular, and Optical Physics
  • Biological and Medical Physics
  • Classical Mechanics
  • Computational Physics
  • Condensed Matter Physics
  • Electromagnetism, Optics, and Acoustics
  • History of Physics
  • Mathematical and Statistical Physics
  • Measurement Science
  • Nuclear Physics
  • Particles and Fields
  • Plasma Physics
  • Quantum Physics
  • Relativity and Gravitation
  • Semiconductor and Mesoscopic Physics
  • Browse content in Psychology
  • Affective Sciences
  • Clinical Psychology
  • Cognitive Psychology
  • Cognitive Neuroscience
  • Criminal and Forensic Psychology
  • Developmental Psychology
  • Educational Psychology
  • Evolutionary Psychology
  • Health Psychology
  • History and Systems in Psychology
  • Music Psychology
  • Neuropsychology
  • Organizational Psychology
  • Psychological Assessment and Testing
  • Psychology of Human-Technology Interaction
  • Psychology Professional Development and Training
  • Research Methods in Psychology
  • Social Psychology
  • Browse content in Social Sciences
  • Browse content in Anthropology
  • Anthropology of Religion
  • Human Evolution
  • Medical Anthropology
  • Physical Anthropology
  • Regional Anthropology
  • Social and Cultural Anthropology
  • Theory and Practice of Anthropology
  • Browse content in Business and Management
  • Business Strategy
  • Business Ethics
  • Business History
  • Business and Government
  • Business and Technology
  • Business and the Environment
  • Comparative Management
  • Corporate Governance
  • Corporate Social Responsibility
  • Entrepreneurship
  • Health Management
  • Human Resource Management
  • Industrial and Employment Relations
  • Industry Studies
  • Information and Communication Technologies
  • International Business
  • Knowledge Management
  • Management and Management Techniques
  • Operations Management
  • Organizational Theory and Behaviour
  • Pensions and Pension Management
  • Public and Nonprofit Management
  • Strategic Management
  • Supply Chain Management
  • Browse content in Criminology and Criminal Justice
  • Criminal Justice
  • Criminology
  • Forms of Crime
  • International and Comparative Criminology
  • Youth Violence and Juvenile Justice
  • Development Studies
  • Browse content in Economics
  • Agricultural, Environmental, and Natural Resource Economics
  • Asian Economics
  • Behavioural Finance
  • Behavioural Economics and Neuroeconomics
  • Econometrics and Mathematical Economics
  • Economic Systems
  • Economic History
  • Economic Methodology
  • Economic Development and Growth
  • Financial Markets
  • Financial Institutions and Services
  • General Economics and Teaching
  • Health, Education, and Welfare
  • History of Economic Thought
  • International Economics
  • Labour and Demographic Economics
  • Law and Economics
  • Macroeconomics and Monetary Economics
  • Microeconomics
  • Public Economics
  • Urban, Rural, and Regional Economics
  • Welfare Economics
  • Browse content in Education
  • Adult Education and Continuous Learning
  • Care and Counselling of Students
  • Early Childhood and Elementary Education
  • Educational Equipment and Technology
  • Educational Strategies and Policy
  • Higher and Further Education
  • Organization and Management of Education
  • Philosophy and Theory of Education
  • Schools Studies
  • Secondary Education
  • Teaching of a Specific Subject
  • Teaching of Specific Groups and Special Educational Needs
  • Teaching Skills and Techniques
  • Browse content in Environment
  • Applied Ecology (Social Science)
  • Climate Change
  • Conservation of the Environment (Social Science)
  • Environmentalist Thought and Ideology (Social Science)
  • Natural Disasters (Environment)
  • Social Impact of Environmental Issues (Social Science)
  • Browse content in Human Geography
  • Cultural Geography
  • Economic Geography
  • Political Geography
  • Browse content in Interdisciplinary Studies
  • Communication Studies
  • Museums, Libraries, and Information Sciences
  • Browse content in Politics
  • African Politics
  • Asian Politics
  • Chinese Politics
  • Comparative Politics
  • Conflict Politics
  • Elections and Electoral Studies
  • Environmental Politics
  • European Union
  • Foreign Policy
  • Gender and Politics
  • Human Rights and Politics
  • Indian Politics
  • International Relations
  • International Organization (Politics)
  • International Political Economy
  • Irish Politics
  • Latin American Politics
  • Middle Eastern Politics
  • Political Methodology
  • Political Communication
  • Political Philosophy
  • Political Sociology
  • Political Behaviour
  • Political Economy
  • Political Institutions
  • Political Theory
  • Politics and Law
  • Public Administration
  • Public Policy
  • Quantitative Political Methodology
  • Regional Political Studies
  • Russian Politics
  • Security Studies
  • State and Local Government
  • UK Politics
  • US Politics
  • Browse content in Regional and Area Studies
  • African Studies
  • Asian Studies
  • East Asian Studies
  • Japanese Studies
  • Latin American Studies
  • Middle Eastern Studies
  • Native American Studies
  • Scottish Studies
  • Browse content in Research and Information
  • Research Methods
  • Browse content in Social Work
  • Addictions and Substance Misuse
  • Adoption and Fostering
  • Care of the Elderly
  • Child and Adolescent Social Work
  • Couple and Family Social Work
  • Developmental and Physical Disabilities Social Work
  • Direct Practice and Clinical Social Work
  • Emergency Services
  • Human Behaviour and the Social Environment
  • International and Global Issues in Social Work
  • Mental and Behavioural Health
  • Social Justice and Human Rights
  • Social Policy and Advocacy
  • Social Work and Crime and Justice
  • Social Work Macro Practice
  • Social Work Practice Settings
  • Social Work Research and Evidence-based Practice
  • Welfare and Benefit Systems
  • Browse content in Sociology
  • Childhood Studies
  • Community Development
  • Comparative and Historical Sociology
  • Economic Sociology
  • Gender and Sexuality
  • Gerontology and Ageing
  • Health, Illness, and Medicine
  • Marriage and the Family
  • Migration Studies
  • Occupations, Professions, and Work
  • Organizations
  • Population and Demography
  • Race and Ethnicity
  • Social Theory
  • Social Movements and Social Change
  • Social Research and Statistics
  • Social Stratification, Inequality, and Mobility
  • Sociology of Religion
  • Sociology of Education
  • Sport and Leisure
  • Urban and Rural Studies
  • Browse content in Warfare and Defence
  • Defence Strategy, Planning, and Research
  • Land Forces and Warfare
  • Military Administration
  • Military Life and Institutions
  • Naval Forces and Warfare
  • Other Warfare and Defence Issues
  • Peace Studies and Conflict Resolution
  • Weapons and Equipment

Case Studies in Pharmacy Ethics (3 edn)

Case Studies in Pharmacy Ethics (3 edn)

Case Studies in Pharmacy Ethics (3 edn)

Professor of Medical Ethics Emeritus and Senior Research Scholar

Director, Center for Health Policy and Ethics and Dr. C.C. and Mabel L. Criss Endowed Chair in Health Sciences

Medical Safety Clinical Pharmacist

  • Cite Icon Cite
  • Permissions Icon Permissions

The third edition of Case Studies in Pharmacy Ethics presents a comprehensive series of cases faced by pharmacists that raise ethical issues, with chapters arranged in a manner that simultaneously presents the topics that would be covered in a course on ethical theory. After an introduction, the book is divided into three parts. The introduction takes up four basic issues in ethical theory: the source, meaning, and justification of ethical claims; the two major ways of determining if acts are morally right; how moral rules apply to specific situations; and what ought to be done in specific cases. Part I deals with conceptual issues. Chapter 1 presents a five-step model the pharmacist can use for ethical problem solving. Chapter 2 addresses identification of value judgments in pharmacy and separation of ethical from nonethical value judgments. Chapter 3 looks at where the pharmacist should turn to find the source of ethical judgments. Part II presents cases organized around the major principles of ethics: beneficence and nonmaleficence, justice and the allocation of resources, autonomy, veracity (dealing honestly with patients), fidelity (including confidentiality), and avoidance of killing. Part III presents cases organized around topics that present ethical controversy: abortion, sterilization, and contraception; genetics and birth technologies; and mental health and behavior control. The remaining chapters cover drug formularies and drug distribution systems; health insurance, health system planning, and rationing; pharmaceutical research; consent to drug therapies; and terminally ill patients. The book includes links to professional codes of ethics and a glossary.

Signed in as

Institutional accounts.

  • GoogleCrawler [DO NOT DELETE]
  • Google Scholar Indexing

Personal account

  • Sign in with email/username & password
  • Get email alerts
  • Save searches
  • Purchase content
  • Activate your purchase/trial code

Institutional access

  • Sign in with a library card Sign in with username/password Recommend to your librarian
  • Institutional account management
  • Get help with access

Access to content on Oxford Academic is often provided through institutional subscriptions and purchases. If you are a member of an institution with an active account, you may be able to access content in one of the following ways:

IP based access

Typically, access is provided across an institutional network to a range of IP addresses. This authentication occurs automatically, and it is not possible to sign out of an IP authenticated account.

Sign in through your institution

Choose this option to get remote access when outside your institution. Shibboleth/Open Athens technology is used to provide single sign-on between your institution’s website and Oxford Academic.

  • Click Sign in through your institution.
  • Select your institution from the list provided, which will take you to your institution's website to sign in.
  • When on the institution site, please use the credentials provided by your institution. Do not use an Oxford Academic personal account.
  • Following successful sign in, you will be returned to Oxford Academic.

If your institution is not listed or you cannot sign in to your institution’s website, please contact your librarian or administrator.

Sign in with a library card

Enter your library card number to sign in. If you cannot sign in, please contact your librarian.

Society Members

Society member access to a journal is achieved in one of the following ways:

Sign in through society site

Many societies offer single sign-on between the society website and Oxford Academic. If you see ‘Sign in through society site’ in the sign in pane within a journal:

  • Click Sign in through society site.
  • When on the society site, please use the credentials provided by that society. Do not use an Oxford Academic personal account.

If you do not have a society account or have forgotten your username or password, please contact your society.

Sign in using a personal account

Some societies use Oxford Academic personal accounts to provide access to their members. See below.

A personal account can be used to get email alerts, save searches, purchase content, and activate subscriptions.

Some societies use Oxford Academic personal accounts to provide access to their members.

Viewing your signed in accounts

Click the account icon in the top right to:

  • View your signed in personal account and access account management features.
  • View the institutional accounts that are providing access.

Signed in but can't access content

Oxford Academic is home to a wide variety of products. The institutional subscription may not cover the content that you are trying to access. If you believe you should have access to that content, please contact your librarian.

For librarians and administrators, your personal account also provides access to institutional account management. Here you will find options to view and activate subscriptions, manage institutional settings and access options, access usage statistics, and more.

Our books are available by subscription or purchase to libraries and institutions.

  • About Oxford Academic
  • Publish journals with us
  • University press partners
  • What we publish
  • New features  
  • Open access
  • Rights and permissions
  • Accessibility
  • Advertising
  • Media enquiries
  • Oxford University Press
  • Oxford Languages
  • University of Oxford

Oxford University Press is a department of the University of Oxford. It furthers the University's objective of excellence in research, scholarship, and education by publishing worldwide

  • Copyright © 2024 Oxford University Press
  • Cookie settings
  • Cookie policy
  • Privacy policy
  • Legal notice

This Feature Is Available To Subscribers Only

Sign In or Create an Account

This PDF is available to Subscribers Only

For full access to this pdf, sign in to an existing account, or purchase an annual subscription.

  • ⏩ healthcare
  • ⏩ AI in Healthcare: Examples, Use Cases, and Benefits

AI in Healthcare: Examples, Use Cases, and Benefits

AI in healthcare examples, use cases, and benefits

Key Takeaways

  • Technologies like data analytics, deep learning, predictive analytics, and content analytics support early diagnosis and care services.
  • AI-driven predictive analytics optimize processes, reduce errors, and identify high-risk patients for early intervention, lowering healthcare costs and enabling institutions to handle more patients efficiently.
  • AI in healthcare extends beyond data processing, offering benefits like enhanced diagnostic accuracy, personalized patient care, and efficient administrative task automation.
  • Valued at $20.9B in 2024, the global AI medical market is forecasted to reach $148.4B by 2029, growing at a CAGR of 48.1%.

The increasing use of digital technology and reliance on data in the medical field presents impressive benefits of AI in healthcare.

As the number of patients grows, it generates vast amounts of health data that need to be processed and turned into actionable insights. This demand, coupled with the need for personalized medicine and digital health records, unleashes the transformative role of AI in healthcare.

Integrating AI and machine learning enables early disease detection and improved patient care. At Acropolium, we have diverse expertise in AI/ML consulting and solutions development. Serving the healthcare sector for 15+ years, we witnessed how the power of AI automation transformed medical business operations.

Today, we will guide you through global AI applications in healthcare, showing how artificial intelligence is used in healthcare businesses to save lives and cut operational costs.

What is Artificial Intelligence in Healthcare?

Widely adopted in industries globally , AI in healthcare encompasses the application of ML algorithms and cognitive technologies in medical settings. As the most prominent healthcare tech trend , AI analyzes medical data to predict outcomes and assist in diagnosis and treatment planning.

By using AI in the healthcare industry for patient data analysis, medical professionals can deliver more precise diagnoses and treatment strategies. Furthermore, AI enhances healthcare by analyzing big data to develop proactive preventive care recommendations for patients.

benefits of AI in healthcare with global market predictions

The medical sector generates vast amounts of data requiring automatic analysis to optimize hospital workloads and reduce costs. For that reason, artificial intelligence and healthcare go hand in hand, with the global AI medical market projected to reach $148.4B by 2029. In 2024, it’s valued at $20.9B with an expected CAGR of 48.1 toward the next four years.

Benefits of AI in Healthcare Operations

what is AI in healthcare and what are its benefits

As medical organizations employ AL and ML algorithms to deal with big chunks of data, the number of AI benefits in healthcare is not limited to data processing alone.

Enhanced Diagnostic Accuracy

AI algorithms can analyze large datasets of medical records, images, and diagnostic tests to assist medical clinics in more accurate and timely diagnosis. This can lead to preventive disease detection and improved treatment outcomes.

Personalized Patient Care

By analyzing patient data, including medical history, genetics, and lifestyle factors, AI enables personalized treatment plans. Such AI solutions in healthcare are usually cloud-based , where the data is being collected on a medical platform or web application. Personalization can lead to higher patient satisfaction, contributing to improved diagnoses and outcomes.

Efficient Administrative Tasks

As part of a hospital management system , AI streamlines administrative tasks by automating scheduling appointments, managing medical records, and processing billing. This efficiency frees up healthcare professionals’ time, reduces administrative burdens, and minimizes errors, allowing them to focus more on patient care.

Cost Reduction through Predictive Analytics

By optimizing processes, reducing medical errors, and facilitating preventive care, AI can help lower healthcare costs for both providers and patients. By digitizing healthcare routines and switching to remote care models with telehealth apps , institutions can handle more patients a day with fewer resources needed.

Predictive analytics can also identify high-risk patients and intervene early to prevent costly medical complications.

Use Cases and AI Applications in Healthcare

examples of AI in healthcare operations

The ways how one can employ AI for analysis and automation are defined by the complexity of their business and their core objectives. Here, we explore some of AI in healthcare examples from real life and applications that help medical businesses globally get the most out of their operations with advanced algorithms.

Medical Diagnosis and Treatment

AI aids in diagnosing diseases and recommending treatment plans. For example, IBM’s Watson for Oncology analyzes patient data to suggest personalized cancer treatments, improving clinical decision-making.

WFO has cataloged and organized literature, protocols, and patient charts. It has acquired knowledge from Memorial Sloan Kettering Cancer Center (MSKCC) test cases and experts. This allows them to employ computational reasoning methods to address specific cases.

Patient Data Processing

One of the core artificial intelligence use cases in healthcare is that it processes large volumes of patient data efficiently. For instance, Google’s DeepMind Health applies AI algorithms to analyze patient records, assisting clinicians in identifying patients at risk of deterioration.

In a specific case, their tailored AI system lowered false positives by 25% in a sizable UK mammography dataset compared to standard clinical workflows. With constant learning and use, it achieved this without overlooking any true positives.

Medical Imaging Analysis

Medical imaging is one of the most progressive AI healthcare applications. AI analyzes medical images, aiding in disease diagnosis and treatment. For instance, it identifies brain tumors in MRI scans, assisting surgical planning.

It detects Alzheimer’s and dementia in brain scans by assessing structural changes. Also, AI analyzes retinal images, detecting early-stage diabetic retinopathy, a cause of blindness in diabetics. GE Healthcare’s AI-powered Edison platform automates the analysis of MRI and CT scans, aiding radiologists in detecting abnormalities more quickly.

Electronic Health Records (EHRs)

AI technology is increasingly indispensable in EHR management. It enables the analysis of vast medical data to spot patterns that aid disease prevention and treatment. AI healthcare technology scrutinizes EHR data, identifying high-risk patients and devising personalized prevention strategies.

AI predicts disease risks based on medical history, lifestyle, and genetics, like diabetes or heart disease. It also flags medication patterns to prevent adverse drug reactions, suggesting alternative treatments. Oracle Health employs AI in its EHR systems to enhance data accuracy, streamline workflows, and improve patient care coordination.

Remote Patient Assistance

AI enables remote monitoring and patient assistance. This method emphasizes convenience, accessibility, and timely interventions, enhancing patient outcomes and reducing healthcare costs.

Patients access real-time medical support, share health data, and receive remote treatment guidance. Businesses utilize AI in the healthcare industry to analyze this data, identifying trends and potential issues early. This proactive and remote approach reduces hospital visits and fosters efficient healthcare delivery.

Thus, Philips’ HealthSuite Digital Platform uses AI to monitor patients’ vital signs remotely, allowing early intervention and reducing hospital readmissions.

Virtual Assistants and Chatbots

AI-powered virtual assistants and healthcare chatbots are vital in remote patient care, allowing medical staff to focus on complex issues. These tools provide personalized health advice, offer pharmacy suggestions , and aid patients in seeking medication information promptly. By offering swift and accurate responses, they reduce the need for patients to wait for professional guidance.

Patients can also use chatbots to schedule appointments and clarify upcoming visits. AI-driven virtual assistants provide medical advice and support. Healthily’s chatbot offers symptom checking and medical advice, improving patients’ access to healthcare services.

Drug Discovery & Vaccine Research

The use of artificial intelligence in healthcare and medicine accelerates drug discovery and vaccine development. It identifies potential drug candidates or vaccine targets more efficiently than traditional methods. In fact, AI algorithms can predict the effectiveness and safety of compounds, leading to faster development and testing processes.

Atomwise applies AI to the virtual screening of molecular compounds for drug discovery, expediting the identification of potential treatments.

Challenges and Considerations in Implementing AI in the Healthcare Industry

artificial intelligence use cases in healthcare and implementation challenges

The rise of AI in the healthcare industry is propelled by several facilitating factors like abundant healthcare data and a trend of patients becoming more proactive consumers. Nevertheless, the landscape of the healthcare AI market isn’t without its challenges and obstacles, presenting hurdles that need to be addressed.

Data Privacy and Security Concerns

Organizations must ensure that patient data used for AI applications in healthcare is secure and compliant with regulations like HIPAA . To safeguard sensitive patient information, implement robust cybersecurity encryption methods and access controls. Regular audits and training programs can also ensure staff compliance with privacy regulations.

Interoperability and Data Integration

Healthcare systems often use disparate data sources and formats, making it challenging to integrate data for AI applications seamlessly. Those who plan to utilize AI solutions in healthcare should invest in interoperable systems and standardized data formats to facilitate data integration.

Application programming interfaces (APIs) can help streamline data exchange between different healthcare systems and platforms.

Ethical and Regulatory Compliance

AI applications in healthcare raise ethical concerns related to patient consent, bias in algorithms, and accountability for decision-making. Make sure to develop clear ethical guidelines and governance frameworks for AI use in your organization.

If you’re cooperating with a tech vendor, they must regularly audit AI algorithms for bias and ensure transparency in decision-making processes.

Lack of Skilled Workforce

To fully experience the benefits of AI in healthcare, a skilled workforce with expertise in data science, machine learning, and healthcare domain knowledge is required.

At this point, you’ll need to find a third-party partner to provide training programs and resources to upskill existing staff.

Why Choose Acropolium?

Acropolium is a seasoned web developer with a long track record of successfully delivering secure projects to the healthcare industry. Crafting GDPR-compliant software, we follow ISO-certified processes to guarantee unparalleled data security and regulatory adherence.

From on-demand applications and custom software development to legacy system modernization , our dedicated teams will elevate your operations with smart algorithms.

Acropolium Case Studies

As our company has been delivering AI/ML solutions to global industries, from healthcare and hospitality to logistics and fintech, we have some success stories to share. Let’s start with one of our most valuable AI in healthcare examples!

AI-based Data Quality & Data Profiling Tool

A top fintech firm sought automated data profiling, commissioning a new tool for data quality assurance and categorization. The tool was supposed to automatically gather, categorize, and allocate data while ensuring its integrity for valuable insights.

AI healthcare applications and case studies by Acropolium

  • The development team established standardized processes and frameworks for real-time data quality monitoring, identifying and tracking issues via dashboards and configuring alerts for immediate notification of changes.
  • The system design was optimized for future growth and fluctuating data inflow and structured for horizontal scalability to manage increasing volumes without performance degradation.
  • The real-time data quality monitoring tool promptly identifies and flags data quality issues.
  • We used a proactive ML-based approach to ensure swift corrective actions to prevent inaccurate data spread.
  • The scalable data quality tool and profiling software architecture include customizable features for modifying incorrect data detection settings.
  • Data errors decreased by 40%, achieving a final data quality rate of 95%.
  • Additionally, data processing time was reduced by 30%.
  • The client now processes up to 30 terabytes of data daily, showing a 200% improvement in scalability.

Bioscience Cloud-Based Big Data Processing App

A research-focused proteomics company specializing in discovering new biomarkers contacted us to seek the integration of AI and ML into its biomaterial analysis processes. Apart from employing advanced algorithms, we had to optimize their software’s back-end.

  • We opted to structure the entire app development using the Scrum methodology, establishing a unified approach for managing task lifecycles.
  • Daily stand-ups facilitated seamless communication among team members and external back-end developers.
  • Our dedicated team ensured regular demos, which provided clients with insights into the project’s progress, enabling them to suggest product priority changes more effectively.
  • The project launch time was decreased by 30%.
  • The client enhanced analysis processing accuracy by 40%
  • Overall, we reduced analysis processing time by 38%.

Final Thoughts

Cutting-edge processing has revolutionized the medical industry, offering unparalleled benefits of AI in healthcare with improved patient outcomes and streamlined operations.

From personalized treatment plans and enhanced diagnostic accuracy to efficient administrative tasks and predictive analytics, AI continues to reshape the healthcare landscape. And Acropolium is here to help you discover the endless growth opportunities for your business!

From complex custom software development to consultation, we offer a flexible, subscription-based cooperation model that caters to any budget without compromising quality.

Sources of Information

  • Artificial Intelligence Healthcare Market (2024-2029) — Markets and Markets

Let's start a new project together!

Leave us your details and explore the full potential of our future collaboration.

image

What are AI use cases in healthcare?

AI applications in healthcare encompass diagnosis, treatment optimization, and patient monitoring.AI algorithms can analyze medical data to predict diseases, recommend personalized treatment plans, and monitor patient vitals in real-time.

How does AI improve medical imaging analysis?

Applications of AI in healthcare that are used for medical imaging enable swift and accurate identification of anomalies in X-rays, MRIs, and CT scans. Using deep learning algorithms, AI can detect subtle abnormalities that may be missed by human eyes, enabling earlier diagnosis and treatment.

What are some examples of AI applications in personalized treatment?

In personalized treatment, AI healthcare solutions analyze patient data like genetic information, medical history, and lifestyle factors, to tailor treatment plans. For instance, AI-powered platforms can recommend optimal medication dosages or suggest targeted therapies based on individual patient profiles.

What role do AI virtual health assistants play in healthcare?

AI virtual health assistants serve as digital interfaces between patients and healthcare providers. These healthcare AI applications, usually chatbots, can provide basic medical information, schedule appointments, and offer remote consultations.

What are the benefits of integrating AI into healthcare?

The benefits of AI in healthcare operations include improved efficiency, enhanced diagnostic accuracy, and personalized patient care. Furthermore, AI can streamline administrative tasks, reduce medical errors, and facilitate preventive care, ultimately leading to better patient outcomes and reduced healthcare costs.

How can healthcare professionals and organizations adopt AI technologies?

Healthcare professionals and organizations can adopt AI technologies by investing in training programs, collaborating with AI vendors, and implementing AI solutions in healthcare workflows. By fostering a culture of learning the emerging technologies, healthcare providers can harness AI's full potential.

Articles you may also like

mHealth App Development Explained: Why MVPs Fail & How to Build Yours

mHealth App Development Explained: Why MVPs Fail & How to Build Yours

Cloud Computing in Healthcare [5 Real Use Cases Included]

Cloud Computing in Healthcare [Real Use Cases Included]

Developing a Feature-Rich On-Demand Veterinary App: Practices to Follow

Developing a Feature-Rich On-Demand Veterinary App: Practices to Follow

Custom Hospital Management Software [2024 Guide]

How to Choose the Best Hospital Management Software for Healthcare Business

Guide to ☑️ Health Information Management Systems Development

Guide to Healthcare Management System Development

Chatbots in Healthcare [10 Use Cases] + Development Guide

Chatbots in Healthcare: Development and Use Cases

We use cookies on our website to enhance your experience. If you want to see the complete overview of the cookies used, please see our privacy policy. Read more

Strictly necessary cookies

These cookies are essential for your use of different parts of our website and its features. Without them, services that you want to get can’t be provided.

Visitor preferences

These cookies help us personalize the Acropolium website and provide you with relevant content by remembering your preferences and settings.

Analytics cookies

These cookies allow us to measure your use of the website so that we can improve it later. They collect information in a way that does not directly identify anyone.

  • Share full article

Advertisement

Supported by

Study Suggests Genetics as a Cause, Not Just a Risk, for Some Alzheimer’s

People with two copies of the gene variant APOE4 are almost certain to get Alzheimer’s, say researchers, who proposed a framework under which such patients could be diagnosed years before symptoms.

A colorized C.T. scan showing a cross-section of a person's brain with Alzheimer's disease. The colors are red, green and yellow.

By Pam Belluck

Scientists are proposing a new way of understanding the genetics of Alzheimer’s that would mean that up to a fifth of patients would be considered to have a genetically caused form of the disease.

Currently, the vast majority of Alzheimer’s cases do not have a clearly identified cause. The new designation, proposed in a study published Monday, could broaden the scope of efforts to develop treatments, including gene therapy, and affect the design of clinical trials.

It could also mean that hundreds of thousands of people in the United States alone could, if they chose, receive a diagnosis of Alzheimer’s before developing any symptoms of cognitive decline, although there currently are no treatments for people at that stage.

The new classification would make this type of Alzheimer’s one of the most common genetic disorders in the world, medical experts said.

“This reconceptualization that we’re proposing affects not a small minority of people,” said Dr. Juan Fortea, an author of the study and the director of the Sant Pau Memory Unit in Barcelona, Spain. “Sometimes we say that we don’t know the cause of Alzheimer’s disease,” but, he said, this would mean that about 15 to 20 percent of cases “can be tracked back to a cause, and the cause is in the genes.”

The idea involves a gene variant called APOE4. Scientists have long known that inheriting one copy of the variant increases the risk of developing Alzheimer’s, and that people with two copies, inherited from each parent, have vastly increased risk.

The new study , published in the journal Nature Medicine, analyzed data from over 500 people with two copies of APOE4, a significantly larger pool than in previous studies. The researchers found that almost all of those patients developed the biological pathology of Alzheimer’s, and the authors say that two copies of APOE4 should now be considered a cause of Alzheimer’s — not simply a risk factor.

The patients also developed Alzheimer’s pathology relatively young, the study found. By age 55, over 95 percent had biological markers associated with the disease. By 65, almost all had abnormal levels of a protein called amyloid that forms plaques in the brain, a hallmark of Alzheimer’s. And many started developing symptoms of cognitive decline at age 65, younger than most people without the APOE4 variant.

“The critical thing is that these individuals are often symptomatic 10 years earlier than other forms of Alzheimer’s disease,” said Dr. Reisa Sperling, a neurologist at Mass General Brigham in Boston and an author of the study.

She added, “By the time they are picked up and clinically diagnosed, because they’re often younger, they have more pathology.”

People with two copies, known as APOE4 homozygotes, make up 2 to 3 percent of the general population, but are an estimated 15 to 20 percent of people with Alzheimer’s dementia, experts said. People with one copy make up about 15 to 25 percent of the general population, and about 50 percent of Alzheimer’s dementia patients.

The most common variant is called APOE3, which seems to have a neutral effect on Alzheimer’s risk. About 75 percent of the general population has one copy of APOE3, and more than half of the general population has two copies.

Alzheimer’s experts not involved in the study said classifying the two-copy condition as genetically determined Alzheimer’s could have significant implications, including encouraging drug development beyond the field’s recent major focus on treatments that target and reduce amyloid.

Dr. Samuel Gandy, an Alzheimer’s researcher at Mount Sinai in New York, who was not involved in the study, said that patients with two copies of APOE4 faced much higher safety risks from anti-amyloid drugs.

When the Food and Drug Administration approved the anti-amyloid drug Leqembi last year, it required a black-box warning on the label saying that the medication can cause “serious and life-threatening events” such as swelling and bleeding in the brain, especially for people with two copies of APOE4. Some treatment centers decided not to offer Leqembi, an intravenous infusion, to such patients.

Dr. Gandy and other experts said that classifying these patients as having a distinct genetic form of Alzheimer’s would galvanize interest in developing drugs that are safe and effective for them and add urgency to current efforts to prevent cognitive decline in people who do not yet have symptoms.

“Rather than say we have nothing for you, let’s look for a trial,” Dr. Gandy said, adding that such patients should be included in trials at younger ages, given how early their pathology starts.

Besides trying to develop drugs, some researchers are exploring gene editing to transform APOE4 into a variant called APOE2, which appears to protect against Alzheimer’s. Another gene-therapy approach being studied involves injecting APOE2 into patients’ brains.

The new study had some limitations, including a lack of diversity that might make the findings less generalizable. Most patients in the study had European ancestry. While two copies of APOE4 also greatly increase Alzheimer’s risk in other ethnicities, the risk levels differ, said Dr. Michael Greicius, a neurologist at Stanford University School of Medicine who was not involved in the research.

“One important argument against their interpretation is that the risk of Alzheimer’s disease in APOE4 homozygotes varies substantially across different genetic ancestries,” said Dr. Greicius, who cowrote a study that found that white people with two copies of APOE4 had 13 times the risk of white people with two copies of APOE3, while Black people with two copies of APOE4 had 6.5 times the risk of Black people with two copies of APOE3.

“This has critical implications when counseling patients about their ancestry-informed genetic risk for Alzheimer’s disease,” he said, “and it also speaks to some yet-to-be-discovered genetics and biology that presumably drive this massive difference in risk.”

Under the current genetic understanding of Alzheimer’s, less than 2 percent of cases are considered genetically caused. Some of those patients inherited a mutation in one of three genes and can develop symptoms as early as their 30s or 40s. Others are people with Down syndrome, who have three copies of a chromosome containing a protein that often leads to what is called Down syndrome-associated Alzheimer’s disease .

Dr. Sperling said the genetic alterations in those cases are believed to fuel buildup of amyloid, while APOE4 is believed to interfere with clearing amyloid buildup.

Under the researchers’ proposal, having one copy of APOE4 would continue to be considered a risk factor, not enough to cause Alzheimer’s, Dr. Fortea said. It is unusual for diseases to follow that genetic pattern, called “semidominance,” with two copies of a variant causing the disease, but one copy only increasing risk, experts said.

The new recommendation will prompt questions about whether people should get tested to determine if they have the APOE4 variant.

Dr. Greicius said that until there were treatments for people with two copies of APOE4 or trials of therapies to prevent them from developing dementia, “My recommendation is if you don’t have symptoms, you should definitely not figure out your APOE status.”

He added, “It will only cause grief at this point.”

Finding ways to help these patients cannot come soon enough, Dr. Sperling said, adding, “These individuals are desperate, they’ve seen it in both of their parents often and really need therapies.”

Pam Belluck is a health and science reporter, covering a range of subjects, including reproductive health, long Covid, brain science, neurological disorders, mental health and genetics. More about Pam Belluck

The Fight Against Alzheimer’s Disease

Alzheimer’s is the most common form of dementia, but much remains unknown about this daunting disease..

How is Alzheimer’s diagnosed? What causes Alzheimer’s? We answered some common questions .

A study suggests that genetics can be a cause of Alzheimer’s , not just a risk, raising the prospect of diagnosis years before symptoms appear.

Determining whether someone has Alzheimer’s usually requires an extended diagnostic process . But new criteria could lead to a diagnosis on the basis of a simple blood test .

The F.D.A. has given full approval to the Alzheimer’s drug Leqembi. Here is what to know about i t.

Alzheimer’s can make communicating difficult. We asked experts for tips on how to talk to someone with the disease .

logo

Case Studies

GB is a 42-year-old man who comes to the clinic for a scheduled visit. GB is overweight and admits to exercising very infrequently. He has a medical history of hypertension and hyperlipidemia for which he takes hydrochlorothiazide and rosuvastatin, respectively. Recent laboratory results show GB has a fasting blood glucose of 172 mg/dL, an A1C of 7.6%, and a serum creatinine of 1.1 mg/dL. He does not complain of polyuria, polydipsia, or other common signs of diabetes. Following repeat laboratory results a few weeks later showing similar glucose abnormalities, GB’s physician diagnoses him with type 2 diabetes and decides to initiate pharmacologic treatment.

What is the most appropriate initial pharmacologic treatment for GB?

CK is a 23-year-old woman who presents to her physician with a positive result (15 mm induration) on her purified protein derivative (PPD) tuberculin skin test. She denies signs or symptoms such as cough, blood-tinged sputum, fever, or weight loss, and her chest x-ray is unremarkable. CK’s physician diagnoses her with latent tuberculosis (TB). CK states she is enrolled in classes at a local college and admits to having a very busy schedule and difficulty managing stress. She is concerned about her ability to be adherent to an extended duration treatment regimen. The physician asks you for your recommendation on an appropriate regimen to treat CK’s latent TB. CK has no other medical conditions and is not currently taking any prescription medications.

Which latent TB treatment regimen would you recommend for CK?

Case 1: Case One Answer: C The 2012 American Diabetes Association guidelines now recommend that most newly diagnosed type 2 diabetes patients be initiated on metformin therapy along with lifestyle modifications. - Exceptions include the presence of a contraindication to metformin, patients that are highly symp tomatic, or those who have severe glucose abnormalities, in which case insulin therapy (with or without additional agents) should be considered.

Case 2: Answer: Case Two Latent TB occurs when an infected individual harbors live bacteria ( does not present with any of the signs or symptoms consistent with active infection. At any time, however, an individual with latent TB can experience activation of the bacteria, exhibit symptoms indicative of TB, and become contagious. Although the standard of care for treating patients with latent TB consists of isoniazid daily for 9 months, the Centers for Disease Control and Prevention (CDC) issued new guidelines in December 2011 stating that isoniazid and rifapentine taken once weekly for 3 months (12 once-weekly doses) is as effective for treating latent TB as isoniazid for 9 months. Moreover, in clinical trials comparing the 2 regimens, more patients completed the 3-month isoniazid and rifapentine regimen. The CDC suggests this shorter regimen be considered for all patients who are 12 years or older (use in children aged 2-11 years should be considered on a case-by-case basis) and otherwise healthy, including patients with HIV not taking antiretrovirals. Based on CK’s concerns about being adherent to an extended duration regimen, isoniazid and rifapentine once weekly for 12 weeks seems appropriate.As GB has no contraindications to metformin, is asymptomatic, and does not have a severe glu cose abnormality, metformin should be initiated at a daily dose of 500 mg taken with the largest meal of the day. This dose could then be titrated up by 500 mg every week, up to 2000 mg daily or the maximum tolerated dose. GB’s physician and pharmacist should also stress the importance of lifestyle modifications. This should include a diet low in carbohydrates, calories, and fat, and enriched with fruits, vegetables, and fiber. Furthermore, it is recommended that patients engage in moderately intense aerobic physical activity for at least 150 minutes per week.

Read the answers

Dr. Coleman is associate professor of pharmacy practice and director of the pharmacoeconomics and outcomes studies group at the University of Connecticut School of Pharmacy. Mr. Caranfa is a PharmD candidate from the University of Connecticut School of Pharmacy.

function showAnswer() {document.getElementById("answer").style.display = 'block';document.getElementById("link").style.display = 'none';}

References:

1. Kirkwood CK, Melton ST. Insomnia, drowsiness, and fatigue. In: Krinsky DL, Berardi RR, Ferreri SP, et al, eds. Handbook of Nonprescription Drugs: An Interactive Approach to Self-Care. 17th ed. Washington, DC: American Pharmacists Association; 2012:867-883.

2. Morin AK, Jarvis CI, Lynch AM. Therapeutic options for sleep maintenance and sleep-onset insomnia. Pharmacotherapy. 2007;27:89-110.

3. Shimp LA. Disorders related to menstruation. In: Krinsky DL, Berardi RR, Ferreri SP, et al, eds. Handbook of Nonprescription Drugs: An Interactive Approach to Self-Care. 17th ed. Washington, DC: American Pharmacists Association; 2012:147-158.

4. St. John’s wort and depression. National Institutes of Health/National Center for Complementary and Alternative Medicine website. http://nccam.nih.gov/health/stjohnswort/sjw-and-depression.htm. Accessed February 12, 2012.

5. Yanni EA. Jet lag. Centers for Disease Control and Prevention website. http://wwwnc.cdc.gov/travel/yellowbook/2012/chapter-2-the-pre-travel-consultation/jet-lag.htm. New York, NY: Oxford University Press; 2012.

6. Herxheimer A, Petrie KJ. Melatonin for the prevention and treatment of jet lag. Cochrane Database Syst Rev. 2002;(2):CD001520.

7. Dennehy CE, Tsourounis C. Botanicals (“herbal medications”) and nutritional supplements. In: Katzung BG, ed. Basic and Clinical Pharmacology. 10th ed. New York, NY: Lange/McGraw Hill; 2007:1060-1062.

pharmacy patient case study examples

Celebrate Women’s Health Month

pharmacist and woman at counter with medicine or prescription drugs sales at drug store. | Image Credit: Clayton D/peopleimages.com - stock.adobe.com

Culturally Competent Care Creates a Safe Space for a Better Patient Experience

Gene-editing technology -- Image credit: vchalup | stock.adobe.com

Commercializing CRISPR/Cas9 Therapy: FDA Approval of Exa-Cel Marks Milestone in Treatment Development

Modern pharmacy building facade with large window showcasing the interior, as seen from the street view, promoting a welcoming atmosphere for customers. Frontal view. Generative AI - Image credit: Karrrtinki | stock.adobe.com

It’s Crunch Time for Cash Flow and Closures. What’s on the Other Side?

Medical students listening to a lecture in the lab | Image Credit: luckybusiness - stock.adobe.com

Hello, Generation Z: Welcome to Provider Status

Gene therapy-- Image credit: nobeastsofierce | stock.adobe.com

Considering Financial Perspectives With Gene Therapy Breakthroughs

2 Commerce Drive Cranbury, NJ 08512

609-716-7777

pharmacy patient case study examples

IMAGES

  1. Case Studies Clinical pharmacy

    pharmacy patient case study examples

  2. Case Studies Clinical pharmacy

    pharmacy patient case study examples

  3. Case Studies Clinical pharmacy

    pharmacy patient case study examples

  4. 10+ Patient Case Study Examples in PDF

    pharmacy patient case study examples

  5. FREE 10+ Patient Case Study Samples & Templates in MS Word

    pharmacy patient case study examples

  6. Case Studies Clinical pharmacy

    pharmacy patient case study examples

VIDEO

  1. Hospital and Clinical Pharmacy

  2. A day in my life as a hospital Pharmacist #hospital

  3. Case study discussion for a pharmaceutical product launch marketing plan

  4. Case presentation on Hypertension, Pharm D

  5. Patient Medication History Interview / L-5 UNIT-2 Pharmacy Practice 7th sem / HCP 2nd Year D.Pharm

  6. Quick Case Review: Guess the Disease? Ep. 4 #medical #academicmedicine #casestudy #gastroenterology

COMMENTS

  1. 50+ Pharmacy Case Studies for Students!

    As a qualifying pharmacist, case studies bring together the threads of study over the past four years. This includes your study of subjects such as: Pharmacology. Pharmaceutical chemistry. Pharmaceutics. Clinical pharmacy practice. In practice, pharmacists are expected to draw on this knowledge and clinically apply it where necessary.

  2. PDF How to Present a Patient Case

    Summarize the major points of the case. Provide a limited number (e.g. 3) of takeaway points for the audience. Tailor summary and takeaway points to your audience. Critical Thinking Skills. Successful patient case presentations: Integrate disease and drug knowledge, clinical evidence, and patient factors.

  3. Clinical Case Studies with Answers

    c) Endocrine System Case Studies with answers: Diabetes: Case Study 17 and Case Study - 18. Hypothyroidism : Case Study -19 and Case Study -20. Other Thyroid Disorders : Case Study -21 and Case Study -22. Oral Contraceptive Use: Case Study- 23 and Case Study- 24. Hormone Replacement Therapy: Case Study- 25 and Case Study- 26.

  4. PDF Presenting a Patient Case update

    Ms. Lewis is an 83 yo Caucasian F, weighing 65kg and measuring 5'3", who came to clinic today with complaints of a cough after discontinuing lisinopril 1 month ago due to angioedema and starting losartan 50 mg po daily. Her PMH is significant for HTN, type II DM, and CKD (stage IV). The patient's SH and FH are non-contributory.

  5. Cases

    013 Case 013-Chronic Heart Failure Case # 2. 014 Case 014-Venous Thromboembolism-Pulmonary Embolism Case # 1. 015 Case 015-Venous Thromboembolism-Pulmonary Embolism Case # 2. 016 Case 016-Stroke. 017 Case 017-Arrhythmias. 018 Case 018-Cardiac Arrest, Resuscitation.

  6. Case Studies

    Case Studies. CASE 1. JC is a 60-year-old woman with rheumatoid arthritis (RA) for the past 2 years. She presents to the clinic with worsening disease activity, complaining of increased joint pain and inflammation for the past 2 months that limits her ability to do daily activities. JC has been self-managing the pain/inflammation with naproxen ...

  7. Interactive Case Studies (August 2020)

    CASE 1: NS is a 55-year-old man who recently visited his primary-care provider (PCP) for a follow up.His medical history includes hypothyroidism, hypertension, and type 2 diabetes (T2D). He takes levothyroxine 75 mcg daily, lisinopril 20 mg daily, and metformin 1 gram twice a day.

  8. Case study: assessing and counselling a patient with insomnia

    It is now three days since the pharmacist dispensed his prescription and Mr AB has returned to the pharmacist because he claims he is still not able to sleep properly. Pharmacy Case Studies by Soraya Dhillon and Rebekah Raymond. Pp 471 £29.99. London: Pharmaceutical Press; 2009. ISBN 978 0 85369 724 4.

  9. Interactives: Case Studies (April 2021)

    Case 1. EP is a patient with epilepsy. He has been taking phenytoin at a therapeutic dose for 4 years. EP was recently prescribed valproic acid because he has had breakthrough seizures over the past few months. After taking valproic acid for several days, he calls the pharmacy expressing concern about some new adverse effects.

  10. Case Study: A Patient With Type 2 Diabetes Working With an Advanced

    B.L. is a 58-year-old white woman who has been referred to the pharmacist clinician for pharmacotherapy assessment and diabetes management. Her multiple medical conditions include type 2 diabetes diagnosed in 1995, hypertension, hyperlipidemia, asthma, coronary artery disease, persistent peripheral edema, and longstanding musculoskeletal pain secondary to a motor vehicle accident.

  11. Cases

    16 Acute Coronary Syndrome: ST-Elevation Myocardial Infarction. 20 Acutely Decompensated Heart Failure. 26 Atrial Fibrillation. 27 Cardiac Arrest. 23 Chronic Anticoagulation. 21 Deep Vein Thrombosis. 14 Dyslipidemia. 19 Heart Failure with Preserved Ejection Fraction. 18 Heart Failure with Reduced Ejection Fraction.

  12. PDF Example Case Studies

    Example Case Study 1a: BRAJFEC . Patient D.N. (BC Cancer ID #20-45678) is a 48-year-old female in good health, ... Doses administered must be documented on the patient chart as well as on the pharmacy record. The maximum cumulative dose for epirubicin is 720 - 1000 mg/m. 2.

  13. Pharmacy Case Studies for Pharmacists & Medical Sciences Students

    This section is only for Bangladeshi Pharmacy/Medical Students & Professionals ! Cardiovascular case studies by Narinder Bhalla. Case study level 1 - Angina. Case study level 2 - Hypertension. Case study level 3 - Atrial fibrillation. Case study level Ma - Heart failure. Case study level Mb - Myocardial infarction.

  14. Pharmacy guide to joint pain: case studies

    In the ' Pharmacy guide to joint pain: diagnosis and assessment ' and ' Pharmacy guide to joint pain: management ', best practice for pharmacy teams was outlined for patients who present in community pharmacy with new onset joint pain. In this article, case studies provide further context for the appropriate diagnosis and management of ...

  15. Case-based learning: constipation in adults

    Case study 1: an older patient presents with recurrent constipation. A man aged 74 years* presents to the pharmacy asking for something to help him manage his constipation. He explains that it "comes and goes" but he was hoping to "speed it along this time". Assessment. It is important to discuss the following with the patient:

  16. Veracity: Dealing Honestly with Patients

    Veatch, Robert M., and others, 'Veracity: Dealing Honestly with Patients', Case Studies in Pharmacy Ethics, 3 edn (New York, 2017; ... traditional health professional ethics looks only to the consequences for the patient. 4 For example, in past eras when pharmacists were expected to be paternalistic, if asked by a patient the purpose of a ...

  17. Case Studies (January 2018)

    Case 2: Although the mechanism underlying dopaminergic augmentation is not well understood, it is thought that dopamine-related medications, including pramipexole, may overstimulate the brain and cause a change in dopamine receptors and/or dopamine levels, leading to an overall decrease in natural dopamine activity. The rate of new-onset augmentation is estimated to be approximately 3% to 9% ...

  18. PDF Case Studies in Drug Interactions and Polypharmacy Issues

    The incidence of a clinical drug interactions 3-5% in pts taking 3 or fewer medication, but 20% in pts taking 10-20 drugs. 20% of hospital admissions result from an adverse drug event or a Drug-Drug Interaction. 60-80% of computerized DDI alerts are overridden. To prevent a SERIOUS drug interaction, you would have to review over 2,700 alerts.

  19. (Management Case Study) Antimicrobial Stewardship: Small ...

    •Vancomycin 15mg/kg IV Q12h (Pharmacy to dose) ••Duration of treatment 5-7 days. 1. Clostridium Difficile (C. Diff) 3 loose stools within 24hr w/symptoms Consider alternative cause of diarrhea No solid stool samples tested Do not test patients with history of C. Diff if loose stools and symptoms are not

  20. Case Studies in Pharmacy Ethics

    Abstract. The third edition of Case Studies in Pharmacy Ethics presents a comprehensive series of cases faced by pharmacists that raise ethical issues, with chapters arranged in a manner that simultaneously presents the topics that would be covered in a course on ethical theory. After an introduction, the book is divided into three parts. The introduction takes up four basic issues in ethical ...

  21. PDF Case studies for pharmacists and GP surgeries

    Case study four. The practice-based pharmacists undertakes a variety of roles and brings many benefits to the practice: Pharmacist-led clinics: NSAID reduction clinics; persuading patients to move back to simple analgesia, i.e. paracetamol. Benzodiazepine withdrawal clinics; dose reduction schedules with GP prescribing support.

  22. 5 Interesting Medication Case Reports

    This article is part 5 of a 6-part series on interesting and unusual medication-related case reports. For part 4 click here. Case reports are defined as the scientific documentation of an individual patient. These reports are often written to document an unusual clinical presentation, treatment approach, side effect, or response to treatment.

  23. AI in Healthcare: Examples, Use Cases & Benefits [2024 Guide]

    AI-powered virtual assistants and healthcare chatbots are vital in remote patient care, allowing medical staff to focus on complex issues. These tools provide personalized health advice, offer pharmacy suggestions, and aid patients in seeking medication information promptly. By offering swift and accurate responses, they reduce the need for ...

  24. Study Suggests Genetics as a Cause, Not Just a Risk, for Some Alzheimer

    Most patients in the study had European ancestry. While two copies of APOE4 also greatly increase Alzheimer's risk in other ethnicities, the risk levels differ, said Dr. Michael Greicius, a ...

  25. Case Studies

    Case 1: Case One Answer: C The 2012 American Diabetes Association guidelines now recommend that most newly diagnosed type 2 diabetes patients be initiated on metformin therapy along with lifestyle modifications. - Exceptions include the presence of a contraindication to metformin, patients that are highly symp tomatic, or those who have severe ...